«Детская школа искусств» Мошенского муниципального района

Ответы на вопросы информатика 9 класс: Ответы к учебнику по информатике 9 класс Босова ФГОС с машинкой

Содержание

Ответы на тест 2 по Информатике 9 класс (Босова Л.Л. учебник)

Ответы на тест 2 по Информатике 9 класс

«Ответы на тест 2 по Информатике 9 класс» — это пособие для родителей для проверки правильности ответов обучающихся детей (ГДЗ) на «Тестовые вопросы для самоконтроля», указанные в учебнике Информатики в конце 2-й главы. Как утверждают авторы учебника (Л.Л.Босова, А.Ю.Босова) в конце каждой главы приведены тестовые задания, которые помогут оценить, хорошо ли учащиеся освоили теоретический материал и могут ли они применять свои знания для решения возникающих проблем.

Ответы на вопросы помогут родителям оперативно проверить выполнение указанных заданий.


  1. Выберите верное утверждение:
  • а) Один объект может иметь только одну модель
  • б) Разные объекты не могут описываться одной моделью
  • в) Электрическая схема — это модель электрической цепи
  • г) Модель полностью повторяет изучаемый объект

ПРАВИЛЬНЫЙ ОТВЕТ: в)

 


  1. Выберите неверное утверждение:
  • а) Натурные модели — реальные объекты, в уменьшенном или увеличенном виде воспроизводящие внешний вид, структуру или поведение моделируемого объекта
  • б) Информационные модели описывают объект-оригинал на одном из языков кодирования информации
  • в) Динамические модели отражают процессы изменения и развития объектов во времени
  • г) За основу классификации моделей может быть взята только предметная область, к которой они относятся

ПРАВИЛЬНЫЙ ОТВЕТ: г)

 


  1. Какие признаки объекта должны быть отражены в информационной модели ученика, позволяющей получать следующие сведения: возраст учеников, увлекающихся плаванием; количество девочек, занимающихся танцами; фамилии и имена учеников старше 14 лет?
  • а) имя, фамилия, увлечение
  • б) имя, фамилия, пол, пение, плавание, возраст
  • в) имя, увлечение, пол, возраст
  • г) имя, фамилия, пол, увлечение, возраст

ПРАВИЛЬНЫЙ ОТВЕТ: г)

 


  1. Выберите элемент информационной модели учащегося, существенный для выставления ему оценки за контрольную работу по информатике:
  • а) наличие домашнего компьютера
  • б) количество правильно выполненных заданий
  • в) время, затраченное на выполнение контрольной работы
  • г) средний балл за предшествующие уроки информатики

ПРАВИЛЬНЫЙ ОТВЕТ: б)

 


  1. Замена реального объекта его формальным описанием — это:
  • а) анализ
  • б) моделирование
  • в) формализация
  • г) алгоритмизация

ПРАВИЛЬНЫЙ ОТВЕТ: в)

 


  1. Выберите знаковую модель:
  • а) рисунок
  • б) схема
  • в) таблица
  • г) формула

ПРАВИЛЬНЫЙ ОТВЕТ: г)

 


  1. Выберите образную модель:
  • а) фотография
  • б) схема
  • в) текст
  • г) формула

ПРАВИЛЬНЫЙ ОТВЕТ: а)

 


  1. Выберите смешанную модель:
  • а) фотография
  • б) схема
  • в) текст
  • г) формула

ПРАВИЛЬНЫЙ ОТВЕТ: б)

 


  1. Описания предметов, ситуаций, событий, процессов на естественных языках — это:
  • а) словесные модели
  • б) логические модели
  • в) геометрические модели
  • г) алгебраические модели

ПРАВИЛЬНЫЙ ОТВЕТ: а)

 


  1. Модели, реализованные с помощью систем программирования, электронных таблиц, специализированных математических пакетов и программных средств для моделирования, называются:
  • а) математическими моделями
  • б) компьютерными моделями
  • в) имитационными моделями
  • г) экономическими моделями

ПРАВИЛЬНЫЙ ОТВЕТ: б)

 


  1. Файловая система персонального компьютера наиболее адекватно может быть описана в виде:
  • а) математической модели
  • б) табличной модели
  • в) натурной модели
  • г) иерархической модели

ПРАВИЛЬНЫЙ ОТВЕТ: г)

 


  1. Графической моделью иерархической системы является:
  • а) цепь
  • б) сеть
  • в) генеалогическое дерево
  • г) дерево

ПРАВИЛЬНЫЙ ОТВЕТ: г)

 


  1. Расписание движения электропоездов может рассматриваться как пример:
  • а) табличной модели
  • б) графической модели
  • в) имитационной модели
  • г) натурной модели

ПРАВИЛЬНЫЙ ОТВЕТ: а)

 


  1. Какая тройка понятий находится в отношении «объект — натурная модель — информационная модель»?
  • а) человек — анатомический скелет — манекен
  • б) человек — медицинская карта — фотография
  • в) автомобиль — рекламный буклет с техническими характеристиками автомобиля — атлас автомобильных дорог
  • г) автомобиль — игрушечный автомобиль — техническое описание автомобиля

ПРАВИЛЬНЫЙ ОТВЕТ: г)

 


  1. На схеме изображены дороги между населёнными пунктами A, B, С, D и указаны протяжённости этих дорог.
     Определите, какие два пункта наиболее удалены друг от друга. Укажите длину кратчайшего пути между ними.
  • а) 17
  • б) 15
  • в) 13
  • г) 9

ПРАВИЛЬНЫЙ ОТВЕТ: в)

 


  1. Населённые пункты А, Б, С, D соединены дорогами. Время проезда на автомобиле из города в город по соответствующим дорогам указано в таблице:

Турист, выезжающий из пункта А, хочет посетить все города за кратчайшее время. Укажите соответствующий маршрут.

  • а) ABCD
  • б) ACBD
  • в) ADCB
  • г) ABDC

ПРАВИЛЬНЫЙ ОТВЕТ: г)

 


  1. В школе учатся четыре ученика — Андреев, Иванов, Петров, Сидоров, имеющие разные увлечения.
    Один из них увлекается теннисом, другой — бальными танцами, третий — живописью, четвёртый — пением. О них известно:
  • Иванов и Сидоров присутствовали на концерте хора, когда пел их товарищ;
  • Петров и теннисист позировали художнику;
  • теннисист дружит с Андреевым и хочет познакомиться с Ивановым.

Чем увлекается Андреев?

  • а) теннисом
  • б) живописью
  • в) танцами
  • г) пением

ПРАВИЛЬНЫЙ ОТВЕТ: б)

 


  1. Два игрока играют в следующую игру. Перед ними лежат три кучки камней, в первой из которых 2 камня, во второй — 3 камня, в третьей — 4 камня. У каждого игрока неограниченно много камней. Игроки ходят по очереди. Ход состоит в том, что игрок или удваивает число камней в какой-то куче, или добавляет по два камня в каждую из куч. Выигрывает игрок, после хода которого либо в одной из куч становится не менее 15 камней, либо общее число камней во всех трёх кучах становится не менее 25. Кто выигрывает при безошибочной игре обоих игроков?
  • а) игрок, делающий первый ход
  • б) игрок, делающий второй ход
  • в) каждый игрок имеет одинаковый шанс на победу
  • г) для этой игры нет выигрышной стратегии

ПРАВИЛЬНЫЙ ОТВЕТ: а)

 


  1. База данных — это:
  • а) набор данных, собранных на одной дискете
  • б) таблица, позволяющая хранить и обрабатывать данные и формулы
  • в) прикладная программа для обработки информации пользователя
  • г) совокупность данных, организованных по определённым правилам, предназначенная для хранения во внешней памяти компьютера и постоянного применения

ПРАВИЛЬНЫЙ ОТВЕТ: г)

 


  1. Какая база данных основана на табличном представлении информации об объектах?
  • а) иерархическая
  • б) сетевая
  • в) распределённая
  • г) реляционная

ПРАВИЛЬНЫЙ ОТВЕТ: в)

 


  1. Строка таблицы, содержащая информацию об одном конкретном объекте, — это:
  • а) поле
  • б) запись
  • в) отчёт
  • г) форма

ПРАВИЛЬНЫЙ ОТВЕТ: б)

 


  1. Столбец таблицы, содержащий определённую характеристику объекта, — это:
  • а) поле
  • б) запись
  • в) отчёт
  • г) ключ

ПРАВИЛЬНЫЙ ОТВЕТ: а)

 


  1. Системы управления базами данных используются для:
  • а) создания баз данных, хранения и поиска в них необходимой информации
  • б) сортировки данных
  • в) организации доступа к информации в компьютерной сети
  • г) создания баз данных

ПРАВИЛЬНЫЙ ОТВЕТ: а)

 


  1. Какое из слов НЕ является названием базы данных?
  • а) Microsoft Access
  • б) OpenOffice. org Base
  • в) OpenOffice.org Writer
  • г) FoxPro

ПРАВИЛЬНЫЙ ОТВЕТ: в)

 


  1. Ниже в табличной форме представлен фрагмент базы данных:


На какой позиции окажется товар «Сканер планшетный», если произвести сортировку данной таблицы по возрастанию столбца КОЛИЧЕСТВО?

ПРАВИЛЬНЫЙ ОТВЕТ: в)

 


  1. Ниже в табличной форме представлен фрагмент базы данных «Продажа канцелярских товаров»:


Сколько записей в данном фрагменте удовлетворяет условию ЦЕНА>20 ИЛИ ПРОДАНО
<50 ?

ПРАВИЛЬНЫЙ ОТВЕТ: в)

 


Вы смотрели «Ответы на тест 2 по Информатике 9 класс (Босова, Ответы на вопросы теста главы 2)»

 

Ответы на тест 3 по Информатике 9 класс (Босова Л.Л. учебник)

Ответы на тест 3 по Информатике 9 класс

«Ответы на тест 3 по Информатике 9 класс» — это пособие для родителей для проверки правильности ответов обучающихся детей (ГДЗ) на «Тестовые вопросы для самоконтроля», указанные в учебнике Информатики в конце 3-й главы. Как утверждают авторы учебника (Л.Л.Босова, А.Ю.Босова) в конце каждой главы приведены тестовые задания, которые помогут оценить, хорошо ли учащиеся освоили теоретический материал и могут ли они применять свои знания для решения возникающих проблем.

Ответы на вопросы помогут родителям оперативно проверить выполнение указанных заданий.


  1. Алгоритмом можно считать:
  • а) описание решения квадратного уравнения 
  • б) расписание уроков в школе
  • в) технический паспорт автомобиля
  • г) список класса в журнале

ПРАВИЛЬНЫЙ ОТВЕТ: а)

 

  1. Как называется свойство алгоритма, означающее, что данный алгоритм применим к решению целого класса задач?
  • а) понятность
  • б) определённость
  • в) результативность
  • г) массовость

ПРАВИЛЬНЫЙ ОТВЕТ: г)

 

  1. Как называется свойство алгоритма, означающее, что он всегда приводит к результату через конечное, возможно, очень большое, число шагов?
  • а) дискретность
  • б) понятность
  • в) результативность 
  • г) массовость

ПРАВИЛЬНЫЙ ОТВЕТ: в)

 

  1. Как называется свойство алгоритма, означающее, что он задан с помощью таких предписаний, которые исполнитель может воспринимать и по которым может выполнять требуемые действия?
  • а) дискретность
  • б) понятность 
  • в) определённость
  • г) массовость

ПРАВИЛЬНЫЙ ОТВЕТ: б)

 

  1. Как называется свойство алгоритма, означающее, что путь решения задачи разделён на отдельные шаги?
  • а) дискретность 
  • б) определённость
  • в) результативность
  • г) массовость

ПРАВИЛЬНЫЙ ОТВЕТ: а)

 

  1. Как называется свойство алгоритма, означающее, что путь решения задачи определён вполне однозначно, на любом шаге не допускаются никакие двусмысленности и недомолвки?
  • а) дискретность
  • б) понятность
  • в) определённость 
  • г) результативность

ПРАВИЛЬНЫЙ ОТВЕТ: в)

 

  1. Исполнителю Черепашке был дан для исполнения следующий алгоритм:

Повтори 10 [Вперед 10 Направо 72]

Какая фигура появится на экране?

  • а) незамкнутая ломаная линия
  • б) правильный десятиугольник
  • в) фигура, внутренние углы которой равны 72°
  • г) правильный пятиугольник

ПРАВИЛЬНЫЙ ОТВЕТ: г)

 

  1. Исполнитель Робот передвигается по клетчатому полю, выполняя команды, которым присвоены номера: 1 — на клетку вверх, 2 — на клетку вниз, 3 — на клетку вправо, 4 — на клетку влево. Между соседними клетками поля могут стоять стены. Если при выполнении очередного шага Робот сталкивается со стеной, то он разрушается. В результате выполнения программы 3242332411 Робот успешно прошел из точки А в точку Б. Какую программу необходимо выполнить, чтобы вернуться из точки Б в точку А по кратчайшему пути и не подвергнуться риску разрушения?
  • а) 41
  • б) 4131441322
  • в) 2231441314
  • г) 2 41314
  • д) 14

ПРАВИЛЬНЫЙ ОТВЕТ: д)

 

  1. Система команд исполнителя Вычислитель состоит из двух команд, которым присвоены номера:
  1. — вычти 2
  2. — умножь на 3.

Первая из них уменьшает число на 2, вторая увеличивает число в 3 раза. При записи алгоритмов для краткости указываются лишь номера команд. Запишите алгоритм, содержащий не более пяти команд, с помощью которого из числа 11 будет получено число 13.

ПРАВИЛЬНЫЙ ОТВЕТ: 11121.

 

  1. Некоторый алгоритм строит цепочки символов следующим образом:
  • первая цепочка состоит из одного символа — цифры 1;
  • в начало каждой из последующих цепочек записывается число — номер строки по порядку, далее дважды подряд записывается предыдущая строка.

Вот первые 3 строки, созданные по этому правилу:

Сколько символов будет в седьмой цепочке, созданной по этому алгоритму?

ПРАВИЛЬНЫЙ ОТВЕТ: 127.

 

  1. Наибольшей наглядностью обладают следующие формы записи алгоритмов:
  • а) словесные
  • б) рекурсивные
  • в) графические 
  • г) построчные

ПРАВИЛЬНЫЙ ОТВЕТ: в)

 

  1. Величины, значения которых меняются в процессе исполнения алгоритма, называются:
  • а) постоянными
  • б) константами
  • в) переменными 
  • г) табличными

ПРАВИЛЬНЫЙ ОТВЕТ: в)

 

  1. Величиной целого типа является
  • а) количество мест в зрительном зале 
  • б) рост человека
  • в) марка автомобиля
  • г) площадь государства

ПРАВИЛЬНЫЙ ОТВЕТ: а)

 

  1. Какое логическое выражение истинно, если х е [—10, 10]?
  • а) (х > 10) И (х < -10)
  • б) (х > 10) ИЛИ (х < -10)
  • в) (х < 10) ИЛИ (х >= -10)
  • г) (х >= -10) И (х <= 10)

ПРАВИЛЬНЫЙ ОТВЕТ: г)

 

  1. Укажите правильный вариант записи условия «х — двузначное число»:
  • а) х div 10 <= 9
  • б) (х >= 10) И (х < 100) 
  • в) х div 100 = О
  • г) х mod 100 <= 99

ПРАВИЛЬНЫЙ ОТВЕТ: б)

 

  1. Какая команда присваивания должна следовать за командами А:=А+В и В:=А-В, чтобы последовательное выполнение всех трёх команд вело к обмену значениями переменных А и Б?
  • а) A:=А+В
  • б) А:=А—В 
  • в) В:=А+В
  • г) В:=В—А

ПРАВИЛЬНЫЙ ОТВЕТ: б)

 

  1. К какому виду алгоритмов можно отнести алгоритм, схема которого представлена справа?
  • а) линейный
  • б) разветвляющийся
  • в) циклический
  • г) вспомогательный

ПРАВИЛЬНЫЙ ОТВЕТ: а)

 

  1. К какому виду алгоритмов можно отнести алгоритм, схема которого представлена ниже?
  • а) линейный
  • б) разветвляющийся с неполным ветвлением
  • в) разветвляющийся с полным ветвлением 
  • г) циклический

ПРАВИЛЬНЫЙ ОТВЕТ: в)

 

  1. К какому виду алгоритмов можно отнести алгоритм, схема которого представлена справа?
  • а) цикл с параметром
  • б) цикл с заданным условием продолжения работы 
  • в) цикл с заданным условием окончания работы
  • г) цикл с заданным числом повторений

ПРАВИЛЬНЫЙ ОТВЕТ: б)

 

  1. К какому виду алгоритмов можно отнести алгоритм, схема которого представлена ниже?
  • а) цикл с заданным условием продолжения работы
  • б) цикл с заданным условием окончания работы
  • в) цикл с постусловием
  • г) цикл с заданным числом повторений

ПРАВИЛЬНЫЙ ОТВЕТ: г)

 

  1. К какому виду алгоритмов можно отнести алгоритм, схема которого представлена ниже?
  • а) цикл с заданным условием продолжения работы
  • б) цикл с заданным условием окончания работы 
  • в) цикл с заданным числом повторений
  • г) цикл с предусловием

ПРАВИЛЬНЫЙ ОТВЕТ: б)

 

  1. Сергей, Антон, Таня и Надя, гуляя по лесу, наткнулись на овраг, который можно перейти по шаткому мосту. Сергей может перейти его за минуту, Антон — за две, Таня — за три, Надя — за четыре. Фонарик у группы только один, и он обязательно нужен для перехода по мосту, который выдерживает только двоих человек. Когда два человека вместе идут по мосту, то идут они со скоростью более медлительного из них. Ребята смогли разработать алгоритм перехода на другой берег за минимально возможное время. Какое время она затратили на его исполнение?
  • а) 10 минут
  • б) 11 минут 
  • в) 12 минут
  • г) 13 минут

ПРАВИЛЬНЫЙ ОТВЕТ: б)

 

  1. Дан фрагмент линейного алгоритма,

а:=8
b:=6+3*а
а:=b/3*а

Чему равно значение переменной а после его исполнения?

ПРАВИЛЬНЫЙ ОТВЕТ: 80.

 

  1. Исполните следующий фрагмент линейного алгоритм для а = х и b = у.

а:=а+b
b:=b—а 
а:=а+b 
b:=—b

Какие значения присвоены переменным а и b ?

  • а) y, х 
  • б) х+у, х—у
  • в) х, у
  • г) —у, х

ПРАВИЛЬНЫЙ ОТВЕТ: a)

 

  1. Определите значение целочисленных переменных х и у после выполнения фрагмента программы.

х:=11 
у:=5 
t:=y 
у:=х mod у 
х:=t 
у:=y+2*t

  • а) x=11, у=5
  • б) х=5, у=11 
  • в) х=10, у=5
  • г) x=5, у=10

ПРАВИЛЬНЫЙ ОТВЕТ: б)

 

  1. Среди четырёх монет есть одна фальшивая. Неизвестно, легче она или тяжелее настоящей. Какое минимальное количество взвешиваний необходимо сделать на весах с двумя чашками без гирь, чтобы определить фальшивую монету?
  • а) 2 
  • б) 3
  • в) 4
  • г) 5

ПРАВИЛЬНЫЙ ОТВЕТ: а)

 

  1. Исполните алгоритм при х = 10 и у = 15.


Какие значения будут получены в результате его работы?

  • а) -5, 10
  • б) 5, 20
  • в) 10, 15
  • г) 5, 5 
  • д) -5, 5

ПРАВИЛЬНЫЙ ОТВЕТ: г)

 

  1. Исполните фрагмент алгоритма при а = 2 и b = 0. Определите значение переменной b после выполнения фрагмента алгоритма.

ПРАВИЛЬНЫЙ ОТВЕТ: 25.

 

 

 

  1. Определите значение переменной f после выполнения фрагмента алгоритма.

ПРАВИЛЬНЫЙ ОТВЕТ: 120.

 

  1. Определите значение переменной s после выполнения фрагмента алгоритма.

ПРАВИЛЬНЫЙ ОТВЕТ: 55.

 


Вы смотрели «Ответы на тест 3 по Информатике 9 класс (Босова, Ответы на вопросы теста главы 3)»

 

Электронное приложение к учебнику «Информатика» для 9 класса (УМК Босова Л.Л. и др. 5-9 кл.)

  • Главная
  •  > 
  • Методист
  •  > 
  • Авторские мастерские
  •  > 
  • Информатика
  •  > 
  • Босова Л. Л.

Введение

Презентация «Информатика 9 класс. Введение»

Презентация «Информатика 9 класс. Введение» (Open Document Format)

Плакат «Техника безопасности»

Ссылки на ресурсы ЕК ЦОР

  • демонстрация «Правильная посадка за компьютером» (134882)
  • демонстрация «Информатизация общества» (126797)
  • демонстрация «Информационное общество» (125823)
  • демонстрация «Информационные ресурсы современного общества» (125847)
  • демонстрация «Информационные преступления и информационная безопасность» (125862)
  • демонстрация «Меры обеспечения информационной безопасности» (125858)
  • кроссворд по теме «Социальная информатика» (125813)

Глава 1. Моделирование и формализация

1.1. Моделирование как метод познания

Презентация «Моделирование как метод познания»

Презентация «Моделирование как метод познания» (Open Document Format)

Интерактивные тесты

  • Онлайн тест «Моделирование как метод познания». Вариант 1
  • Онлайн тест «Моделирование как метод познания». Вариант 2

Ссылки на ресурсы ЕК ЦОР:

  • демонстрация «Классификация моделей» (119303)
  • демонстрация «Моделирование натурное и информационное» (119415)
  • анимированная 3D-модель строения «Арсенал» (198257)
  • анимированная 3D-модель строения «Архангельский собор» (198275)
  • анимированная 3D-модель строения «Благовещенская башня» (198271)
  • демонстрация «Типы информационных моделей» (119357)

Ссылки на ресурсы ФЦИОР:

  • информационный модуль «Назначение и виды информационных моделей»;
  • практический модуль «Назначение и виды информационных моделей»;
  • контрольный модуль «Назначение и виды информационных моделей»;

Ресурсы сети Интернет:

Планета Земля

§ 1.2. Знаковые модели

Презентация «Знаковые модели»

Презентация «Знаковые модели» (Open Document Format)

Интерактивные тесты

  • Онлайн тест «Знаковые модели». Вариант 1
  • Онлайн тест «Знаковые модели». Вариант 2

Ссылки на ресурсы ЕК ЦОР

  • демонстрация «Демонстрационная математическая модель» (119324)
  • демонстрация «Демонстрационная имитационная модель» (119425)
  • интерактивное задание «Изучение закона сохранения импульса» (133528)
  • игра «Равноплечий рычаг» (189509)

Ссылки на ресурсы ФЦИОР:

  • информационный модуль «Назначение и виды информационных моделей»;
  • практический модуль «Назначение и виды информационных моделей»;
  • контрольный модуль «Назначение и виды информационных моделей»;

§ 1. 3. Графические информационные модели

Презентация «Графические модели»

Презентация «Графические модели» (Open Document Format)

Интерактивные тесты

  • Онлайн тест «Графические информационные модели». Вариант 1
  • Онлайн тест «Графические информационные модели». Вариант 2

Ссылки на ресурсы ЕК ЦОР

  • интерактивный задачник, раздел «Графические модели» (119308)
  • инструмент разработки и анализа родословных «Живая Родословная» (145555)

§ 1.4. Табличные информационные модели

Презентация «Табличные информационные модели»

Презентация «Табличные информационные модели» (Open Document Format)

Интерактивные тесты

  • Онлайн тест «Табличные информационные модели». Вариант 1
  • Онлайн тест «Табличные информационные модели». Вариант 2

Ссылки на ресурсы ЕК ЦОР

  • демонстрация «Примеры табличных моделей» (119417)
  • кроссворд по теме: «Информационное моделирование» (119349)
  • тренировочный тест к главе 2 «Информационное моделирование» (119338)

§ 1.5. База данных как модель предметной области

Презентация «База данных как модель предметной области»

Презентация «База данных как модель предметной области» (Open Document Format)

Интерактивные тесты

  • Онлайн тест «База данных как модель предметной области». Вариант 1
  • Онлайн тест «База данных как модель предметной области». Вариант 2

Ссылки на ресурсы ЕК ЦОР

  • интерактивный задачник, раздел «Реляционные структуры данных» (119329)

§ 1.6. Система управления базами данных

Презентация «Система управления базами данных»

Презентация «Система управления базами данных» (Open Document Format)

Интерактивные тесты

  • Онлайн тест «Система управления базами данных» Вариант 1
  • Онлайн тест «Система управления базами данных» Вариант 2

Ссылки на ресурсы ЕК ЦОР:

  • кроссворд по теме: «СУБД и базы данных» (119339)
  • тренировочный тест к главе 3 «Хранение и обработка информации в базах данных» (128617)

Интерактивный тест «Моделирование и формализация»

Тест 1

Глава 2. Алгоритмизация и программирование

§ 2.1. Решение задач на компьютере

Презентация «Программирование как этап решения задачи на компьютере»

Презентация «Программирование как этап решения задачи на компьютере» (Open Document Format)

Интерактивные тесты

  • Онлайн тест «Решение задач на компьютере». Вариант 1
  • Онлайн тест «Решение задач на компьютере». Вариант 2

Ссылки на ресурсы ЕК ЦОР

  • демонстрация «Этапы решения расчетных задач» (125855)
  • демонстрация «Назначение и средства программирования» (126138)

§ 2.2. Одномерные массивы целых чисел

Презентация «Одномерные массивы целых чисел»

Презентация «Одномерные массивы целых чисел» (Open Document Format)

Интерактивные тесты

  • Онлайн тест «Одномерные Массивы». Вариант 1
  • Онлайн тест «Одномерные Массивы». Вариант 2

Ссылки на ресурсы ЕК ЦОР

  • демонстрация «Понятие таблицы и массива» (126150)
  • демонстрация «Описание и ввод значений в массив в программе на Паскале» (126153)
  • демонстрация «Цикл с параметром в алгоритме обработки массива» (126791)
  • демонстрация «Датчик случайных чисел на Паскале» (126117)
  • демонстрация «Алгоритм поиска числа в массиве (125817)

Ссылки на ресурсы ФЦИОР:

  • Одномерные массивы. Практическая работа
  • Работа с массивами (на примере языка Pascal). Контрольная работа
  • Подсчет суммы элементов, максимум и минимум, поиск и сортировка элементов в массиве. Контрольная работа
  • Алгоритмы сортировки

Ссылки на свободно распространяемое программное обеспечение:

  • PascalABC
  • анимационная интерактивная демонстрация «Алгоритм поиска максимального элемента в массиве»
  • анимационная интерактивная демонстрация «Сортировка массивов»

§ 2. 3. Конструирование алгоритмов

Презентация «Конструирование алгоритмов»

Презентация «Конструирование алгоритмов» (Open Document Format)

Интерактивные тесты

  • Онлайн тест «Конструирование алгоритмов». Вариант 1
  • Онлайн тест «Конструирование алгоритмов». Вариант 2

Ссылки на ресурсы ЕК ЦОР:

  • демонстрация «Нисходящий и библиотечный методы построения сложных алгоритмов» (128643)
  • демонстрация «Вспомогательные алгоритмы» (128641)
  • интерактивная игра «Ханойские башни» (195747)

Ссылки на свободно распространяемое программное обеспечение:

  • Система КуМир — Комплект учебных миров
  • PascalABC
  • Интерактивный плакат «Фракталы»

§ 2.4. Запись вспомогательных алгоритмов на языке Паскаль

Презентация «Запись вспомогательных алгоритмов на языке Паскаль»

Презентация «Запись вспомогательных алгоритмов на языке Паскаль» (Open Document Format)

Интерактивные тесты

  • Онлайн тест «Запись вспомогательных алгоритмов на языке Паскаль». Вариант 1
  • Онлайн тест «Запись вспомогательных алгоритмов на языке Паскаль». Вариант 2

Ссылки на свободно распространяемое программное обеспечение:

  • PascalABC

§ 2.5. Алгоритмы управления

Презентация «Алгоритмы управления»

Презентация «Алгоритмы управления» (Open Document Format)

Интерактивные тесты

  • Онлайн тест «Алгоритмы управления». Вариант 1
  • Онлайн тест «Алгоритмы управления». Вариант 2

Ссылки на ресурсы ЕК ЦОР:

  • демонстрация «Зарождение и предмет кибернетики» (128608)
  • демонстрация «Компьютер и управление» (128613)

Интерактивный тест «Алгоритмизация и программирование»

Тест 2

Глава 3. Обработка числовой информации в электронных таблицах

§ 3. 1. Электронные таблицы

Презентация «Электронные таблицы»

Презентация «Электронные таблицы» (Open Document Format)

Интерактивные тесты

  • Онлайн тест «Электронные таблицы». Вариант 1
  • Онлайн тест «Электронные таблицы». Вариант 2

Ссылки на ресурсы ЕК ЦОР:

  • демонстрация к лекции «Назначение и возможности электронных таблиц» (119365)
  • демонстрация «Структура электронной таблицы» (119354)
  • демонстрация «Интерфейс MS Excel» (119441)
  • демонстрация «Диапазон (блок) электронной таблицы» (127438)
  • демонстрация «Ввод и редактирование данных в MS Excel» (119345)
  • демонстрация «Режимы отображения электронной таблицы» (119363)
  • демонстрация «Подготовка электронной таблицы к расчетам» (119320)
  • демонстрация «Манипулирование фрагментами таблицы (очистка и удаление ячеек, добавление строк и столбцов, перемещение, копирование, автозаполнение) MS Excel» (119325)
  • демонстрация «Перемещение по таблице MS Excel» (119296)
  • демонстрация «Форматирование таблицы MS Excel» (119301)
  • демонстрация «Формулы в MS Excel» (119359)
  • интерактивный задачник, раздел «Электронные таблицы. Запись формул» (119384)

§ 3.2. Организация вычислений в электронных таблицах

Презентация «Организация вычислений в электронных таблицах»

Презентация «Организация вычислений в электронных таблицах» (Open Document Format)

Интерактивные тесты

  • Онлайн тест «Организация вычислений в электронных таблицах». Вариант 1
  • Онлайн тест «Организация вычислений в электронных таблицах». Вариант 2

Ссылки на ресурсы ЕК ЦОР:

  • демонстрация «Операции манипулирования с диапазонами ЭТ» (119389)
  • интерактивный задачник, раздел «Электронные таблицы. Запись формул» (119384)
  • интерактивное задание «Тренировочный тест N4» (119442)
  • интерактивное задание «Статистические функции в электронных таблицах» (119341)
  • демонстрация к лекции «Элементарные логические операции» (128620)
  • демонстрация к лекции «Вычисление логических выражений» (128658)
  • демонстрация к лекции «Условная функция» (119322)
  • интерактивный задачник, раздел «Логические формулы в электронных таблицах» (119424)

§ 3. 3. Средства анализа и визуализации данных

Презентация «Средства анализа и визуализации данных»

Презентация «Средства анализа и визуализации данных» (Open Document Format)

Интерактивные тесты

  • Онлайн тест «Средства анализа и визуализации данных». Вариант 1
  • Онлайн тест «Средства анализа и визуализации данных». Вариант 2

Ссылки на ресурсы ЕК ЦОР:

  • демонстрация «Сортировка таблицы» (119323)
  • демонстрация «Сортировка данных в таблице MS Excel» (119408)
  • демонстрация «Деловая графика. Типы диаграмм» (119383)
  • демонстрация «Демонстрационная таблица с диаграммами» (119317)
  • демонстрация «Создание диаграмм MS Excel» (119327)
  • кроссворд по теме: «Электронные таблицы» (119360)
  • тренировочный тест к главе 4 «Табличные вычисления на компьютере» (119423)
  • итоговый тест к главе 4 «Табличные вычисления на компьютере» (119432)

Ссылки на ресурсы ФЦИОР:

  • Основные программные средства для редактирования таблиц и работы с цифровыми данными
Интерактивный тест «Обработка числовой информации в электронных таблицах»

Тест 3

Глава 4. Коммуникационные технологии

§ 4.1. Локальные и глобальные компьютерные сети

Презентация «Локальные и глобальные компьютерные сети»

Презентация «Локальные и глобальные компьютерные сети» (Open Document Format)

Интерактивные тесты

  • Онлайн тест «Локальные и глобальные компьютерные сети». Вариант 1
  • Онлайн тест «Локальные и глобальные компьютерные сети». Вариант 2

Ссылки на ресурсы ЕК ЦОР:

  • демонстрация «Локальные сети» (119353)
  • демонстрация «Модели различных конфигураций локальной сети» (119373)
  • демонстрация «Глобальные сети» (119347)
  • демонстрация «Аппаратное и программное обеспечение сетей» (119316)
  • демонстрация «Программное обеспечение сетевых услуг» (119391)
  • демонстрация к лекции «Технические средства глобальной сети» (119356)

Ссылки на ресурсы ФЦИОР:

  • практическое задание по теме «Глобальные компьютерные сети»
  • контрольное задание по теме «Глобальные компьютерные сети»

§ 4. 2. Всемирная компьютерная сеть Интернет

Презентация «Всемирная компьютерная сеть Интернет»

Презентация «Всемирная компьютерная сеть Интернет» (Open Document Format)

Интерактивные тесты

  • Онлайн тест «Всемирная компьютерная сеть Интернет». Вариант 1
  • Онлайн тест «Всемирная компьютерная сеть Интернет». Вариант 2

Ссылки на ресурсы ЕК ЦОР:

  • демонстрация «Что такое Интернет» (119328)
  • анимация «Демонстрация IP-адресации» (192564)
  • анимация «Организация пространства имен» (192876)
  • анимация «Протокол .IP» (192655)
  • анимация «Сетевой уровень. IP-маршрутизация» (192947)
  • анимация «Демонстрация протокола TCP» (192744)
  • демонстрационный имитатор «Пакетная передачи данных в Интернете» (119376)

§ 4.3. Информационные ресурсы и сервисы Интернета

Презентация «Информационные ресурсы и сервисы Интернета»

Презентация «Информационные ресурсы и сервисы Интернета» (Open Document Format)

Интерактивные тесты

  • Онлайн тест «Информационные ресурсы и сервисы Интернета». Вариант 1
  • Онлайн тест «Информационные ресурсы и сервисы Интернета». Вариант 2

Ссылки на ресурсы ЕК ЦОР:

  • демонстрация «Услуги компьютерных сетей (119300)
  • демонстрационный имитатор «Работа поисковой системы в Интернете» (119393)
  • демонстрация «Язык запросов поисковой системы» (119305)
  • демонстрация «Элементарные логические операции» (128620)
  • демонстрация «Организация поиска информации» (119302)
  • демонстрация «Электронная почта» (119401)
  • демонстрация «Телеконференции» (119420)
  • кроссворд по теме: «Компьютерные сети» (119377)
  • логическая схема понятий по теме: «Компьютерные сети» (119419)
  • тренировочный тест к главе 1 «Передача информации в компьютерных сетях» (119396)
  • итоговый тест к главе 1 «Передача информации в компьютерных сетях» (119412)

§ 4. 4. Создание Web-сайта

Презентация «Создание Web-сайта»

Презентация «Создание Web-сайта» (Open Document Format)

Интерактивные тесты

  • Онлайн тест «Создание web-сайта». Вариант 1
  • Онлайн тест «Создание web-сайта». Вариант 2

Интерактивный тест «Коммуникационные технологии»

Тест 4

Ссылки на ресурсы ЕК ЦОР:

  • тренировочный тест по курсу 9 класса (128626)
  • итоговый тест по курсу 9 класса (128632)
  • тренировочный тест по курсу информатики за 8-9 кл. (128616)
  • итоговый тест по курсу информатики за 8–9 класс (128633)

Тест по Информатике 9 класс, I полугодие

Тест по Информатике 9 класс, I полугодие

  1. Пространственная дискретизация – это процесс

  1. преобразования графической информации из аналоговой формы в дискретную

  2. преобразования графической информации из цифровой формы в аналоговую

  1. В палитре цветов 32 цвета. Чему равна глубина цвета?

  1. 1 бит

  2. 4 бита

  3. 5 битов

  1. Перечислите форматы графических изображений:

  1. .doc, .bmp, .sys

  2. .bmp, .gif, .jpeg

  3. .jpeg, .txt, .gif

4. В одной из кодировок Unicode каждый символ кодируется 16 битами. Определите размер следующего предложения в данной кодировке. Я к вам пишу – чего же боле? Что я могу ещё сказать?

1) 52 байт

2) 832 бит

3) 104 бит

  1. Статья, набранная на компьютере, содержит 32 страницы, на каждой странице 40 строк, в каждой строке 48 символов. Определите размер статьи в кодировке КОИ-8, в которой каждый символ кодируется 8 битами.

1) 120 Кбайт

2) 480 байт

3) 960 байт

4) 60 Кбайт

  1. Переведите число 126 из десятичной системы счисления в двоичную систему счисления.

Укажите двоичное число.

  1. 111

  2. 101010

  3. 1111110

  1. Пользователь находился в каталоге Расписание. Сначала он поднялся на один уровень вверх, затем спустился на один уровень вниз, потом ещё раз спустился на один уровень вниз и ещё раз спустился на один уровень вниз. В результате он оказался в каталоге

С:\учёба\информатика\ОГЭ.

Укажите полный путь каталога, с которым пользователь начинал работу.

1) С:\учёба\Расписание

2) С:\Расписание

3) С:\учёба\2016\Расписание

  1. От чьего имени произошло слово «алгоритм»?

  1. Блез Паскаль

  2. Альберт Эйнштейн

  3. Аль-Хорезми

  1. Представление алгоритмов на языке программирования называется
  1. команда

  2. блок – схема

  3. программа

  1. Перечислите виды алгоритмов
  1. линейный, циклический, круговой
  2. циклический, разветвляющийся, линейный
  3. линейный, разветвляющийся, циклический, вспомогательный
  1. К основным свойствам алгоритма относятся
  1. массовость, понятность, случайность
  2. конечность, результативность, массовость, дискретность, детерминированность, понятность
  3. дискретность, актуальность, результативность, понятность
  1. Язык программирования Pascal создал

  1. Б. Паскаль

  2. Н. Вирт

  3. М. Фортран

  1. Перечислите основные геометрические фигуры, которые используются для создания алгоритмов с помощью блок – схем
  1. трапеция, прямоугольник, треугольник
  2. прямоугольник, овал, ромб, параллелограмм
  3. треугольник, параллелограмм, овал, ромб
  1. Файл Рисунок.bmp находится в папке 9 класс, которая вложена в папку Мои рисунки на диске С: Укажите путь к файлу:

1) С:\Мои рисунки\9 класс\Рисунок.bmp

2) Мои рисунки\9 класс\РРисунок.bmp

3) С:\9 класс\Мои рисунки\Рисунок.gif

15. Группу ячеек, образующих прямоугольник в электронных таблицах называют:

  1. прямоугольником ячеек

  2. диапазоном ячеек

  3. интервалом ячеек

КЛЮЧ

вопроса

Правильный

1

1

2

3

3

2

4

2

5

4

6

3

7

2

8

2

9

3

10

2, 3

11

2

12

2

13

2

14

1

15

2

Оценка «5» -15 правильных ответов

Оценка «4» — 14-13 Оценка «3» — 12-9 Оценка «2» — меньше 9

Информатика 9 Глава 1. Ответы на тест

Информатика 9 Глава 1. Ответы на тест 1. Ответы на вопросы тестового задания после главы 1 в учебнике Босова Информатика 9 класс. Это пособие для родителей для проверки правильности ответов обучающихся детей на «Тестовые вопросы для самоконтроля», указанные в учебнике Информатики в конце 1-й главы. Как утверждают авторы учебника (Л.Л.Босова, А.Ю.Босова) в конце каждой главы приведены тестовые задания, которые помогут оценить, хорошо ли учащиеся освоили теоретический материал и могут ли они применять свои знания для решения возникающих проблем. Ответы на вопросы помогут родителям оперативно проверить выполнение указанных заданий.

Вернуться к Списку тестов по информатике в 9 классе.


Информатика 9 класс. Глава 1


Ответы на тест 1 (Босова)

 

  1. Совокупность знаков, с помощью которых записываются числа, называется:

а) системой счисления
б) цифрами системы счисления
в) алфавитом системы счисления
г) основанием системы счисления

Правильный ответ: в) алфавитом системы счисления
  1. Чему равен результат сложения двух чисел, записанных римскими цифрами: МСМ + LXVIII?

а) 1168
б) 1968
в) 2168
г) 1153

Правильный ответ: б) 1968
  1. Число 301011 может существовать в системах счисления с основаниями:

а) 2 и 10
б) 4 и 3
в) 4 и 8
г) 2 и 4

Правильный ответ: в) 4 и 8
  1. Двоичное число 100110 в десятичной системе счисления записывается как:

а) 36
б) 38
в) 37
г) 46

Правильный ответ: б) 38
  1. В классе 110010
    2% девочек и 10102 мальчиков. Сколько учеников в классе?

а) 10
б) 20
в) 30
г) 40

Правильный ответ: б) 20
  1. Сколько цифр 1 в двоичном представлении десятичного числа 15?

а) 1
б) 2
в) 3
г) 4

Правильный ответ: г) 4
  1. Чему равен результат сложения чисел 110
    2 и 128?

а) 610
б) 1010
в) 100002
г) 178

Правильный ответ: в) 10000
2
  1. Ячейка памяти компьютера состоит из однородных элементов, называемых:

а) кодами
б) разрядами
в) цифрами
г) коэффициентами

Правильный ответ: б) разрядами
  1. Количество разрядов, занимаемых двухбайтовым числом, равно:

а) 8
б) 16
в) 32
г) 64

Правильный ответ: б) 16
  1. В знаковый разряд ячейки для отрицательных чисел заносится:

а) +
б) —
в) 0
г) 1

Правильный ответ: г) 1
  1. Вещественные числа представляются в компьютере в:

а) естественной форме
б) развёрнутой форме
в) экспоненциальной форме с нормализованной мантиссой
г) виде обыкновенной дроби

Правильный ответ: в) экспоненциальной форме с нормализованной мантиссой
  1. Какое предложение не является высказыванием?

а) Никакая причина не извиняет невежливость.
б) Обязательно стань отличником.
в) Рукописи не горят.
г) 10112 = 1 • 23 + 0 • 22 + 1 • 21 + 1 • 20

Правильный ответ: б) Обязательно стань отличником
  1. Какое высказывание является ложным?

а) Знаком v обозначается логическая операция ИЛИ.
б) Логическую операцию ИЛИ также называют логическим сложением.
в) Дизъюнкцию также называют логическим сложением.
г) Знаком v обозначается логическая операция конъюнкция.

Правильный ответ: г) Знаком v обозначается логическая операция конъюнкция.
  1. Для какого из указанных значений числа X истинно высказывание ((X < 5) v (X < 3)) ˄ ((X < 2) v (X < 1)) ?

а) 1
б) 2
в) 3
г) 4

Правильный ответ: а) 1
  1. Для какого символьного выражения верно высказывание: «НЕ (Первая буква согласная) И НЕ (Вторая буква гласная)»?

а) abcde
б) bcade
в) babas
г) cabab

Правильный ответ: а) abcde
  1. Некоторый сегмент сети Интернет состоит из 1000 сайтов. Поисковый сервер в автоматическом режиме составил таблицу ключевых слов для сайтов этого сегмента. Вот её фрагмент:
Ключевое словоКоличество сайтов, для которых данное слово является ключевым
Сканер 200
Принтер 250
Монитор 450

Сколько сайтов будет найдено по запросу принтер | сканер \ монитор, если по запросу принтер \ сканер было найдено 450 сайтов, по запросу принтер & монитор — 40, а по запросу сканер & монитор— 50?

а) 900
б) 540
в) 460
г) 810

Правильный ответ: г) 810
  1. Какому логическому выражению соответствует следующая таблица истинности?
Правильный ответ: в)
  1. Когда сломался компьютер, его хозяин сказал: «Оперативная память не могла выйти из строя».
    Сын хозяина компьютера предположил, что вышел из строя процессор, а жёсткий диск исправен. Пришедший специалист по обслуживанию сказал, что, скорее всего, с процессором всё в порядке, а оперативная память неисправна. В результате оказалось, что двое из них сказали всё верно, а третий — всё неверно. Что же сломалось?

а) оперативная память
б) процессор
в) жёсткий диск
г) процессор и оперативная память

Правильный ответ: б) процессор
  1. На перекрёстке произошло дорожно-транспортное происшествие, в котором участвовали автобус (А), грузовик (Г), легковой автомобиль (Л) и маршрутное такси (М). Свидетели происшествия дали следующие показания. Первый свидетель считал, что первым на перекрёсток выехал автобус, а маршрутное такси было вторым. Другой свидетель полагал, что последним на перекрёсток выехал легковой автомобиль, а вторым был грузовик. Третий свидетель уверял, что автобус выехал на перекрёсток вторым, а следом за ним — легковой автомобиль. В результате оказалось, что каждый из свидетелей был прав только в одном из своих утверждений. В каком порядке выехали машины на перекрёсток? В вариантах ответов перечислены подряд без пробелов первые буквы названий транспортных средств в порядке их выезда на перекрёсток:

а) АМЛГ
б) АГЛМ
в) ГЛМА
г) МЛГА

Правильный ответ: б) АГЛМ
  1. Какое логическое выражение соответствует следующей схеме?
  
Правильный ответ: г)

Вы смотрели Информатика 9 Глава 1. Ответы на тест 1. Ответы на вопросы тестового задания после главы 1 учебника Босова Информатика 9 класс.

Вернуться к Списку тестов по информатике в 9 классе.

Экзамены по Информатике




Также смотрите разделы связанные с разделом ГИА, экзаменационные билеты по Информатике:
  • Обучение информатике, презентации по информатике
  • Книги, задачники и учебники по информатике
  • Книги и учебники по программированию
  • Обучение пользованию Интернет
  • Полезные сайты, ссылки, утилиты, программы
  • Уроки и советы по PHP, HTML, CSS, JavaScript, Java, JSP, Servlet
  • Уроки и советы по CSS
  • Обучение компьютерным программам
  • Решебники и ГДЗ по Информатике
  • Словари по информатике и компьютерам
  • Книги по Веб-дизайну, CSS, HTML, создание сайтов и верстка
  • ЕГЭ по информатике
  • Все книги по информатике


Ниже Вы можете бесплатно скачать электронные книги и учебники и читать статьи и уроки к разделу Экзамены и экзаменационные билеты по Информатике:
  • Демонстрационный вариант контрольных измерительных материалов основного государственного экзамена 2021 года по ИНФОРМАТИКЕ
  • ОГЭ 2021, Информатика и ИКТ, 9 класс, Кодификатор, Проект
  • ОГЭ 2021, Информатика и ИКТ, 9 класс, Спецификация, Проект
  • ОГЭ 2021, информатика, 10 вариантов, типовые варианты экзаменационных заданий от разработчиков ОГЭ, Ушаков Д. М., 2021
  • ОГЭ 2021, Информатика, 9 класс, Спецификация, Кодификатор, Проект
  • ОГЭ, Информатика, Готовимся к итоговой аттестации, Лещинер В.Р., Путимцева Ю.С., 2021
  • Самостоятельные, проверочные и контрольные работы по информатике, 5 класс, К учебнику Л.Л. Босовой, А.Ю. Босовой «Информатика. 5 класс», Лещинер В.Р., 2021
  • Самостоятельные, проверочные и контрольные работы по информатике, 6 класс, К учебнику Л.Л. Босовой, А.Ю. Босовой «Информатика. 6 класс», Лещинер В.Р., 2021

  • ГВЭ, Информатика и ИКТ, 11 класс, Спецификация, Письменная форма, Проект, 2020
  • ГВЭ, Информатика и ИКТ, 11 класс, Спецификация, Устная форма, Проект, 2020
  • ГВЭ, Информатика, 9 класс, Спецификация, Письменная форма, Проект, 2020
  • ГВЭ, Информатика, 9 класс, Спецификация, Устная форма, Проект, 2020
  • Демонстрационный вариант контрольных измерительных материалов для проведения в 2020 году основного государственного экзамена по ИНФОРМАТИКЕ и ИКТ
  • Демонстрационный вариант контрольных измерительных материалов основного государственного экзамена 2020 года по ИНФОРМАТИКЕ
  • Демонстрационный вариант контрольных измерительных материалов основного государственного экзамена 2020 года по ИНФОРМАТИКЕ
  • Информатика, 9 класс, Итоговая контрольная работа, Босова Л.Л., Босова А.Ю., Аквилянов Н.А., 2020
  • Информатика, Основной Государственный Экзамен, Готовимся к итоговой аттестации, Лещинер В.Р., Путимцева Ю.С., 2020
  • Кодификатор проверяемых требований к результатам освоения основной образовательной программы основного общего образования и элементов содержания для проведения основного государственного экзамена по ИНФОРМАТИКЕ и ИКТ, 2020
  • Кодификатор проверяемых требований к результатам освоения основной образовательной программы основного общего образования и элементов содержания для проведения основного государственного экзамена по ИНФОРМАТИКЕ, 2020
  • Логика и программирование, 5-6 лет, Пархоменко С. В., 2019
  • Логика и программирование, 7-8 лет, Пархоменко С.В., 2020
  • Логика и программирование, 9-10 лет, Пархоменко С.В., 2020
  • Методические рекомендации обучающимся по организации индивидуальной подготовки к ОГЭ, Информатика, Крылов С.С., 2020
  • МЕТОДИЧЕСКИЕ РЕКОМЕНДАЦИИ обучающимся по организации индивидуальной подготовки к ОГЭ, ИНФОРМАТИКА, Крылов С.С., 2020
  • ОГЭ 2020, Информатика и ИКТ, 9 класс, Демонстрационный вариант, Кодификатор, Спецификация, Проект
  • ОГЭ 2020, Информатика и ИКТ, 9 класс, Спецификация, Кодификатор, Проект
  • ОГЭ 2020, Информатика, 10 вариантов, Типовые варианты экзаменационных заданий от разработчиков ОГЭ, Ушаков Д.М.
  • ОГЭ 2020, Информатика, 10 вариантов, Типовые варианты экзаменационных заданий от разработчиков ОГЭ, Ушаков Д.М., 2020
  • ОГЭ 2020, Информатика, 10 тренировочных вариантов, Ушаков Д.М., 2019
  • ОГЭ 2020, Информатика, Досрочный вариант
  • ОГЭ 2021, Информатика и ИКТ, 20 тренировочных вариантов по демоверсии, 9 класс, Евич Л.Н., 2020
  • ОГЭ, Информатика, Методические рекомендации по проверке заданий с развернутым ответом, Кириенко Д.П., Крылов С.С., Лещинер В.Р., Путимцева Ю.С., 2020
  • Подготовка к ОГЭ по информатике, 9 класс, Босова Л.Л., Тарапата В.В, Босова А.Ю., 2020
  • Подготовка к ОГЭ по информатике, 9 класс, Босова Л.Л., Тарапата В.В., Босова А.Ю., 2020
  • Сборник заданий формативного оценивания, Информатика, Естественно-математическое направление, 11 класс, 2020
  • Сборник заданий формативного оценивания, Информатика, Общественно-гуманитарное направление, 11 класс, 2020
  • Спецификация контрольных измерительных материалов для проведения в 2020 году основного государственного экзамена по ГЕОГРАФИИ
  • Спецификация контрольных измерительных материалов для проведения в 2020 году основного государственного экзамена по ИНФОРМАТИКЕ
  • Спецификация контрольных измерительных материалов для проведения в 2020 году основного государственного экзамена по ИНФОРМАТИКЕ
  • Спецификация контрольных измерительных материалов для проведения в 2020 году основного государственного экзамена по ИНФОРМАТИКЕ и ИКТ
  • Спецификация экзаменационных материалов для проведения в 2020 году государственного выпускного экзамена по ИНФОРМАТИКЕ и ИКТ, 11 класс, Письменная форма
  • Спецификация экзаменационных материалов для проведения в 2020 году государственного выпускного экзамена по ИНФОРМАТИКЕ и ИКТ, 11 класс, Устная форма
  • Спецификация экзаменационных материалов для проведения в 2020 году государственного выпускного экзамена по ИНФОРМАТИКЕ, Письменная форма
  • Спецификация экзаменационных материалов для проведения в 2020 году государственного выпускного экзамена по ИНФОРМАТИКЕ, Устная форма
  • Тесты по информатике, К учебнику Л. Л. Босовой, А.Ю. Босовой «Информатика 5 класс», Лешинер В.Р., 2020
  • Тесты по информатике, К учебнику Л.Л. Босовой, А.Ю. Босовой «Информатика. 6 класс», Лещинер В.Р., 2020

  • ГВЭ 2019, Информатика и ИКТ, 11 класс, Письменная форма, Спецификация, Проект
  • ГВЭ 2019, Информатика и ИКТ, 11 класс, Устная форма, Спецификация, Проект
  • ГВЭ 2019, Информатика и ИКТ, 9 класс, Письменная форма, Спецификация
  • ГВЭ 2019, Информатика и ИКТ, 9 класс, Письменная форма, Спецификация, Проект
  • ГВЭ 2019, Информатика и ИКТ, 9 класс, Устная форма, Спецификация
  • ГВЭ 2019, Информатика и ИКТ, 9 класс, Устная форма, Спецификация, Проект
  • ГИА 2019, Информатика и ИКТ, 9 класс, Демонстрационный вариант
  • ГИА 2019, Информатика и ИКТ, 9 класс, Кодификатор
  • ГИА 2019, Информатика и ИКТ, 9 класс, Спецификация
  • Информатика и ИКТ, Методические рекомендации по оцениванию выполнения заданий ОГЭ с развернутым ответом, Кириенко Д.П., Лещинер В.Р., Путимцева Ю.С., 2019
  • Информатика и ИКТ, Подготовка к ОГЭ в 2019 году, Диагностические работы, Путимцева Ю.С., 2019
  • Информатика, 7 класс, Итоговая контрольная работа, Босова Л.Л., Босова А.Ю., Аквилянов Н.А., 2019
  • Информатика, 8 класс, Итоговая контрольная работа, Босова Л.Л., Босова А.Ю., Аквилянов Н.А., 2019
  • Информатика, Основной Государственный Экзамен, Готовимся к итоговой аттестации, Лещинер В.Р., Путимцева Ю.С., 2019
  • Информатика, Сборник заданий с решениями и ответами для подготовки к ОГЭ, Ушаков Д.М., 2019
  • Информатика, Сборник заданий с решениями и ответами для подготовки к основному государственному экзамену, Ушаков Д.М., 2019
  • ОГЭ 2019, Информатика и ИКТ, 9 класс, Демонстрационный вариант, Проект
  • ОГЭ 2019, Информатика и ИКТ, 9 класс, Кодификатор, Проект
  • ОГЭ 2019, Информатика и ИКТ, 9 класс, Спецификация, Кодификатор, Проект
  • ОГЭ 2019, Информатика и ИКТ, 9 класс, Спецификация, Проект
  • ОГЭ 2019, Информатика, 10 тренировочных вариантов, Ушаков Д. М., 2018
  • ОГЭ 2019, Информатика, 9 класс, 10 вариантов, Типовые тестовые, Ушаков Д.М.
  • ОГЭ 2019, Информатика, 9 класс, 10 вариантов, Ушаков Д.М., 2019
  • ОГЭ 2019, Иформатика и ИКТ, 9 класс, Методические рекомендации, Кириенко Д.П., Лещинер В.Р., Путимцева Ю.С.
  • ОГЭ 2020, Информатика, тематические тренировочные задания, Зорина Е.М., Зорин М.В., 2019
  • ОГЭ-2019, Информатика, 20 тренировочных вариантов экзаменационных работ, Ушаков Д.М., 2019

  • ГВЭ 2018, Информатика и ИКТ, 9 класс, Устная форма, Спецификация
  • Информатика и ИКТ, Подготовка к ОГЭ в 2018 году, Диагностические работы, Путимцева Ю.С., 2018
  • Информатика, 10 класс, Базовый уровень, Самостоятельные и контрольные работы, Босова Л.Л., 2018
  • Информатика, 11 класс, Базовый уровень, Самостоятельные и контрольные работы, Босова Л.Л., Босова А.Ю., Аквилянов Н.А., 2018
  • Информатика, 5 класс, Итоговая контрольная работа, Босова Л.Л., Босова А.Ю., Аквилянов Н.А., 2018
  • Информатика, 5-7 классы, Занимательные задачи, Босова Л.Л., Босова А.Ю., Бондарева И.М., 2018
  • Информатика, 6 класс, Итоговая контрольная работа, Босова Л.Л., Босова А.Ю., Аквилянов Н.А., 2018
  • Информатика, 7 класс, Рабочая тетрадь, Часть 1, Семакин И.Г., Ромашкина Т.В., 2018
  • Информатика, 7 класс, Рабочая тетрадь, Часть 1, Угринович Н.Д., Серёгин И.А., 2018
  • Информатика, 7 класс, Рабочая тетрадь, Часть 2, Семакин И.Г., Ромашкина Т.В., 2018
  • Информатика, 7 класс, Рабочая тетрадь, Часть 2, Угринович Н.Д., Серёгин И.А., 2018
  • Информатика, 7-9 классы, Сборник задач и упражнений, Босова Л.Л., Босова А.Ю., Аквилянов Н.А., 2018
  • Информатика, 8 класс, Контрольные работы, Угринович Н.Д., 2018
  • Информатика, 8 класс, Самостоятельные и контрольные работы, Босова Л.Л., Босова А.Ю., 2018
  • Информатика, Большой сборник тематических заданий для подготовки к основному государственному экзамену, Ушаков Д.М., 2018
  • Информатика, Рабочая тетрадь, 7 класс, Часть 1, Босова Л. Л., Босова А.Ю., 2018
  • Информатика, Рабочая тетрадь, 7 класс, Часть 2, Босова Л.Л., Босова А.Ю., 2018
  • Материалы заданий командной инженерной олимпиады школьников Олимпиада Национальной технологической инициативы по профилю Электронная инженерия, Умный дом, 2017/2018 учебный год
  • ОГЭ 2018, Информатика и ИКТ, 9 класс, Кодификатор
  • ОГЭ 2018, Информатика и ИКТ, 9 класс, Спецификация
  • ОГЭ 2018, Информатика и ИТК, 9 класс, Спецификация, Кодификатор
  • ОГЭ 2018, Информатика, 10 вариантов, Типовые тестовые задания от разработчиков ОГЭ, Ушаков Д.М., 2018
  • ОГЭ 2018, Информатика, 20 тренировочных вариантов, Ушаков Д.М., 2017
  • ОГЭ 2018, Информатика, 9 класс, 10 вариантов, Типовые тестовые задания, Ушаков Д.М.
  • ОГЭ 2018, Информатика, Большой сборник тематических заданий, Ушаков Д.М.
  • ОГЭ 2019, Информатика, 10 тренировочных вариантов, Ушаков Д.М., 2018
  • ОГЭ 2019, информатика, тематические тренировочные задания, Зорина Е.М., Зорин М.В., 2018
  • ОГЭ, Информатика и ИКТ, Типовые экзаменационные варианты, 10 вариантов, Крылов С.С., Чуркина Т.Е., 2018
  • ОГЭ, Информатика, 10 вариантов, Типовые тестовые задания, Ушаков Д.М., 2018
  • ОГЭ, Информатика, 9 класс, Митасова Т.С., Животова Е.Б., 2018
  • ОГЭ, информатика, 9 класс, обучающие проверочные работы, Митасова Т.С., Животова Е.Б., 2018
  • ОГЭ, Информатика, Комплекс материалов для подготовки учащихся, Лещинер В.Р., 2018
  • ОГЭ-2018, Информатика и ИКТ, Рекомендации по оцениванию заданий, Кириенко Д.П., Лещннер В.Р., Путимцева Ю.С., 2018
  • ОГЭ-2018, Информатика, 20 тренировочных вариантов, Ушаков Д.М., 2018
  • Перспективная модель измерительных материалов для государственной итоговой аттестации по программам основного общего образования, ИНФОРМАТИКА И ИКТ, Демонстрационный вариант, 2018
  • Перспективная модель измерительных материалов для государственной итоговой аттестации по программам основного общего образования, Спецификация измерительных материалов по ИНФОРМАТИКЕ, 2018

  • Информатика и ИКТ, ОГЭ, Тематический тренинг, Учебное пособие, Евич Л. Н., 2017
  • Информатика и ИКТ, Подготовка к ОГЭ в 2017 году, Диагностические работы, Путимцева Ю.С.
  • Информатика, 3 класс, Рабочая тетрадь, Часть 1, Матвеева Н.В., Челак Е.Н., Конопатова Н.К., 2017
  • Информатика, 3 класс, Рабочая тетрадь, Часть 2, Матвеева Н.В., Челак Е.Н., Конопатова Н.К., 2017
  • Информатика, 4 класс, Рабочая тетрадь, Часть 1, Матвеева Н.В., Челак Е.Н., Конопатова Н.К., 2017
  • Информатика, 4 класс, Рабочая тетрадь, Часть 2, Матвеева Н.В., Челак Е.Н., Конопатова Н.К., 2017
  • Информатика, 5 класс, Самостоятельные и контрольные работы, Босова Л.Л., Босова А.Ю., 2017
  • Информатика, 6 класс, Самостоятельные и контрольные работы, Босова Л.Л., Босова А.Ю., 2017
  • Информатика, 7 класс, Контрольные и проверочные работы, Залогова Л.А., Русаков С.В., Шеина Т.Ю., Шестакова Л.В., 2017
  • Информатика, 7 класс, Контрольные работы, Угринович Н.Д., 2017
  • Информатика, 7 класс, Самостоятельные и контрольные работы, Босова Л.Л., Босова А.Ю., 2017
  • Информатика, 8 класс, Контрольные и проверочные работы, Залогова Л.А., Русаков С.В., Шеина Т.Ю., Шестакова Л.В., 2017
  • Информатика, 8 класс, Рабочая тетрадь, Часть 1, Босова Л.Л., Босова А.Ю., 2017
  • Информатика, 8 класс, Рабочая тетрадь, Часть 1, Угринович Н.Д., Серёгин И.А., Полежаева О.А., 2017
  • Информатика, 8 класс, Рабочая тетрадь, Часть 2, Босова Л.Л., Босова А.Ю., 2017
  • Информатика, 8 класс, Рабочая тетрадь, Часть 2, Угринович Н.Д., Серёгин И.А., Полежаева О.А., 2017
  • Информатика, 9 класс, Контрольные и проверочные работы, Залогова Л.А., Русаков С.В., Шеина Т.Ю., Шестакова Л.В., 2017
  • Информатика, 9 класс, Рабочая тетрадь, Часть 1, Босова Л.Л., Босова А.Ю., 2017
  • Информатика, 9 класс, Рабочая тетрадь, Часть 1, Угринович Н.Д., Серёгин И.А., Полежаева О.А., 2017
  • Информатика, 9 класс, Рабочая тетрадь, Часть 2, Босова Л.Л., Босова А.Ю., 2017
  • Информатика, 9 класс, Рабочая тетрадь, Часть 2, Угринович Н. Д., Серёгин И.А., Полежаева О.А., 2017
  • Информатика, 9 класс, Самостоятельные и контрольные работы, Босова Л.Л., Босова А.Ю., 2017
  • Информатика, Новый полный справочник для подготовки к ОГЭ, Ушаков Д.М., 2017
  • Информатика, Новый полный справочник для подготовки к ОГЭ, Ушаков Д.М., 2017
  • Информатика, Рабочая тетрадь для 6 класса, Часть 1, Босова Л.Л., Босова А.Ю., 2017
  • Информатика, Рабочая тетрадь для 6 класса, Часть 2, Босова Л.Л., Босова А.Ю., 2017
  • Информатика, Рабочая тетрадь, 2 класс, Ломаковская А.В., 2017
  • Информатика, Рабочая тетрадь, 5 класс, Часть 1, Босова Л.Л., Босова А.Ю., 2017
  • Информатика, Рабочая тетрадь, 5 класс, Часть 2, Босова Л.Л., Босова А.Ю., 2017
  • Информатика, Рабочая тетрадь, 6 класс, Часть 1, Босова Л.Л., Босова А.Ю., 2017
  • Информатика, Рабочая тетрадь, 6 класс, Часть 2, Босова Л.Л., Босова А.Ю., 2017
  • Информатика, Рабочая тетрадь, 7 класс, Часть 1, Босова Л.Л., Босова А.Ю., 2017
  • Информатика, Рабочая тетрадь, 7 класс, Часть 2, Босова Л.Л., Босова А.Ю., 2017
  • Информатика, Рабочая тетрадь, 7 класс, Часть 2, Босова Л.Л., Босова А.Ю., 2017
  • Информатика, Рабочая тетрадь, 8 класс, Часть 1, Босова Л.Л., Босова А.Ю., 2017
  • Информатика, Рабочая тетрадь, 8 класс, Часть 2, Босова Л.Л., Босова А.Ю., 2017
  • Информатика, Рабочая тетрадь, 9 класс, Часть 2, Босова Л.Л., Босова А.Ю., 2017
  • Контрольно-измерительные материалы, информатика, 6 класс, Масленикова О.Н., 2017
  • Контрольно-измерительные материалы, информатика, 7 класс, Масленикова О.Н., 2017
  • Контрольно-измерительные материалы, информатика, 8 класс, Масленикова О.Н., 2017
  • Контрольно-измерительные материалы, информатика, 9 класс, Масленикова О.Н., 2017
  • Методические рекомендации для учителей, подготовленные на основе анализа типичных ошибок участников ЕГЭ 2017 года, Информатика и ИКТ, Крылов С.С., 2017
  • ОГЭ 2017, Информатика и ИКТ, 9 класс
  • ОГЭ 2017, Информатика и ИКТ, 9 класс, Демонстрационный вариант
  • ОГЭ 2017, Информатика и ИКТ, 9 класс, Кодификатор
  • ОГЭ 2017, Информатика и ИКТ, 9 класс, Спецификация
  • ОГЭ 2017, Информатика и ИКТ, 9 класс, Спецификация, Кодификатор
  • ОГЭ 2017, Информатика, 10 тренировочных вариантов, Ушаков Д. М.
  • ОГЭ 2018, Информатика, 20 тренировочных вариантов, Ушаков Д.М., 2017
  • ОГЭ, Информатика и ИКТ, Типовые экзаменационные варианты, 10 вариантов, Крылов С.С., Чуркина Т.Е., 2017
  • ОГЭ, Информатика, Новый полный справочник, Ушаков Д.М., 2017
  • ОГЭ-2017, Информатика, 10 тренировочных вариантов экзаменационных работ для подготовки к основному государственному экзамену, Ушаков Д.М., 2017
  • ОГЭ-2017, Информатика, 10 тренировочных вариантов, Ушаков Д.М., 2017
  • ОГЭ-2018, Информатика, 10 тренировочных вариантов, Ушаков Д.М., 2017
  • ОГЭ-2018, Информатика, 20 тренировочных вариантов экзаменационных работ для подготовки к основному государственному экзамену, Ушаков Д.М., 2017
  • Рабочая тетрадь, Информатика, 2 класс, Часть 1, Матвеева Н.В., 2017
  • Рабочая тетрадь, Информатика, 2 класс, Часть 2, Матвеева Н.В., 2017
  • Тестирование программного обеспечения, Куликов С., 2017
  • Я сдам ОГЭ, информатика и ИКТ, модульный курс, практикум и диагностика, Лещинер В.Р., Путимцева Ю.С., 2017
  • Я сдам ОГЭ, модульный курс, информатика и ИКТ, практикум и диагностика, Лещинер В.Р., Путимцева Ю.С., 2017

  • ГИА 2016, Информатика и ИКТ, 9 класс, Демонстрационный вариант
  • ГИА 2016, Информатика и ИКТ, 9 класс, Кодификатор
  • ГИА 2016, Информатика и ИКТ, 9 класс, Спецификация
  • Информатика и ИКТ, Биология, 10 класс, Диагностическая работа, Спецификация, 2016
  • Информатика и ИКТ, Подготовка к ОГЭ 2016, 9 класс, 14 тренировочных вариантов, Евич Л.Н., Кулабухов С.Ю., 2015
  • Информатика, 7 класс, Рабочая тетрадь, Часть 1, Босова Л.Л., Босова А.Ю., 2016
  • Информатика, 7 класс, Рабочая тетрадь, Часть 2, Босова Л.Л., Босова А.Ю., 2016
  • Информатика, 9 класс, Рабочая тетрадь, Часть 1, Босова Л.Л., Босова А.Ю., 2016
  • Информатика, 9 класс, Рабочая тетрадь, Часть 1, Информационное моделирование, Семакин И.Г., Ромашкина Т.В., 2016
  • Информатика, 9 класс, Рабочая тетрадь, Часть 2, Информационное моделирование, Семакин И.Г., Ромашкина Т.В., 2016
  • Информатика, Подготовка к ОГЭ в 2016 году, Диагностические работы, 2016
  • Информатика, Рабочая тетрадь для 6 класса, Часть 1, Босова Л.Л., Босова А.Ю., 2016
  • Информатика, Рабочая тетрадь для 6 класса, Часть 2, Босова Л.Л., Босова А.Ю., 2016
  • Информатика, Рабочая тетрадь для 7 класса, Часть 1, Босова Л.Л., Босова А.Ю., 2016
  • Информатика, Рабочая тетрадь для 7 класса, Часть 2, Босова Л.Л., Босова А.Ю., 2016
  • Информатика, Рабочая тетрадь для 8 класса, Часть 1, Босова Л.Л., Босова А.Ю., 2016
  • Информатика, Рабочая тетрадь для 8 класса, Часть 2, Босова Л.Л., Босова А.Ю., 2016
  • Информатика, Рабочая тетрадь для 9 класса, Часть 1, Босова Л.Л., Босова А.Ю., 2016
  • Информатика, Рабочая тетрадь для 9 класса, Часть 2, Босова Л.Л., Босова А.Ю., 2016
  • Информатика, Рабочая тетрадь, 5 класс, Часть 1, Босова Л.Л., Босова А.Ю., 2016
  • Информатика, Рабочая тетрадь, 5 класс, Часть 2, Босова Л.Л., Босова А.Ю., 2016
  • Информатика, Рабочая тетрадь, 9 класс, Часть 3, Семакин И.Г., 2016
  • Контрольно-измерительные материалы, информатика, 5 класс, Масленикова О.Н., 2016
  • Методика преподавания информатики, Контрольная работа, 2016
  • ОГЭ 2016, Информатика и ИКТ, 9 класс, Демонстрационный вариант
  • ОГЭ 2016, Информатика и ИКТ, 9 класс, Спецификация, Кодификатор
  • ОГЭ 2016, Информатика и ИКТ, Методические рекомендации по оцениванию заданий, Кириенко Д.П., Лещинер В.Р., Путимцева Ю.С.
  • ОГЭ, Информатика и ИКТ, типовые экзаменационные варианты, 10 вариантов, Крылов С.С., Чуркина Т.Е., 2016
  • ОГЭ, Информатика, Комплекс материалов, Лещинер В.Р., Путимцева Ю.С., 2016

  • ГИА 2015, Информатика и ИКТ, 9 класс, Учебно-методические материалы, Кириенко Д.П.
  • Демонстрационный вариант контрольных измерительных материалов для проведения в 2015 году ОГЭ по Иформатике и ИКТ
  • Информатика и ИКТ, 2 класс, Терадь для самостоятельной работы, Бененсон Е.П., Паутова А.Г., 2015
  • Информатика и ИКТ, 3 класс, Терадь для самостоятельной работы, Бененсон Е.П., Паутова А.Г., 2015
  • Информатика и ИКТ, 4 класс, Терадь для самостоятельной работы, Бененсон Е.П., Паутова А.Г., 2015
  • Информатика и ИКТ, Подготовка к ОГЭ 2016, 9 класс, 14 тренировочных вариантов, Евич Л.Н., Кулабухов С.Ю., 2015
  • Информатика, 2 класс, Контрольные работы, Матвеева Н.В., Челак Е.Н., Конопатова Н.К., 2015
  • Информатика, 2 класс, Рабочая тетрадь, Рудченко Т.А., Семёнов А.Л., 2015
  • Информатика, 2 класс, Рабочая тетрадь, Рудченко Т.А., Семёнов А.Л., 2015
  • Информатика, 2 класс, Рабочая тетрадь, Часть 1, Матвеева Н.В., Челак Е.Н., Конопатова Н.К., 2015
  • Информатика, 2 класс, Рабочая тетрадь, Часть 2, Матвеева Н.В., Челак Е.Н., Конопатова Н.К., 2015
  • Информатика, 3 класс, Задачник, Цветкова М.С., 2015
  • Информатика, 4 класс, Рабочая тетрадь, Часть 1, Матвеева Н.В., Челак Е.Н., Конопатова Н.К., 2015
  • Информатика, 4 класс, Рабочая тетрадь, Часть 2, Матвеева Н.В., Челак Е.Н., Конопатова Н.К., 2015
  • Информатика, 5 класс, Рабочая тетрадь, Босова Л.Л., Босова А.Ю., 2015
  • Информатика, 8 класс, Рабочая тетрадь, Часть 2, Информационное моделирование, Семакин И.Г., Ромашкина Т.В., 2015
  • Информатика, 9 класс, Рабочая тетрадь, Босова Л.Л., Босова А.Ю., 2015
  • Информатика, 9 класс, Рабочая тетрадь, Босова Л.Л., Босова А.Ю., 2015
  • Информатика, Рабочая тетрадь, 8 класс, Часть 3, Семакин И.Г., 2015
  • Информатика, Рабочая тетрадь, 9 класс, Босова Л.Л. Босова А.Ю., 2015
  • Кодификатор элементов содержания и требований к уровню подготовки обучающихся для проведения ОГЭ по Информатике и ИКТ, 2015
  • ОГЭ 2015, Информатика, 9 класс, Демонстрационный вариант
  • ОГЭ 2016, Информатика, 9 класс, Тематические тренировочные задания, Зорина Е.М., Зорин М.В., 2015
  • ОГЭ, информатика и ИКТ, типовые экзаменационные варианты, 10 вариантов, Крылов С.С., Чуркина Т.Е., 2015
  • Рабочая тетрадь, Информатика, 4 класс, Часть 1, Могилев А.В., 2015
  • Рабочая тетрадь, Информатика, 8 класс, Часть 1, Семакин И.Г., 2015
  • Спецификация контрольных измерительных материалов для проведения в 2015 году ОГЭ по Информатике и ИКТ

  • Інформатика, 6 клас, Робочий зошит, Ривкінд Й.Я., Лисенко Т.I., Чернікова Л.А., Шакотько В.В., 2014
  • ГИА 2014, Информатика и ИКТ, 9 класс, Демонстрационный вариант
  • ГИА 2014, Информатика и ИКТ, 9 класс, Кодификатор
  • ГИА 2014, Информатика и ИКТ, 9 класс, Спецификация
  • ГИА 2014, Информатика, 9 класс, Демонстрационный вариант
  • ГИА 2014, Информатика, 9 класс, Диагностическая работа, Варианты 90301-90302, 2013
  • ГИА 2014, Информатика, 9 класс, Кодификатор
  • ГИА 2014, Информатика, 9 класс, Спецификация
  • ГИА 2014, Информатика, 9 класс, Тренировочная работа №1, Варианты 9101-9104, 2013
  • ГИА 2014, Информатика, Тренировочные варианты, Кириенко Д.П., Осипов П.О.
  • Информатика и ИКТ, 7-9 классы, тематические задачи и тесты за курс основной школы, подготовка к ГИА в форме ОГЭ, пособие с электронным приложением (CD-диск), Евич Л.Н., Кулабухов С.Ю., 2014
  • Информатика и ИКТ, 8 класс, Рабочая тетрадь, Босова Л.Л., Босова А.Ю., 2014
  • Информатика и ИКТ, 9 класс, подготовка к ГИА-2015, пособие с электронным приложением (CD-диск), Евич Л.Н., Кулабухов С.Ю., 2014
  • Информатика и ИКТ, 9 класс, Рабочая тетрадь, Босова Л.Л., Босова А.Ю., 2014
  • Информатика и ИКТ, Рабочая тетрадь, 8 класс, Босова Л.Л., Босова А.Ю., 2014
  • Информатика, 3 класс, Рабочая тетрадь, Часть 1, Матвеева Н.В., Челак Е.Н., Конопатова Н.К., 2014
  • Информатика, 3 класс, Рабочая тетрадь, Часть 2, Матвеева Н.В., Челак Е.Н., Конопатова Н.К., 2014
  • Информатика, 4 класс, Контрольные работы, Матвеева Н.В., Челак Е.Н., Конопатова Н.К., 2014
  • Информатика, 5 класс, Рабочая тетрадь, Босова Л.Л., Босова А.Ю., 2014
  • Информатика, 6 класс, Рабочая тетрадь, Босова Л.Л., Босова А.Ю., 2014
  • Информатика, 7 класс, Рабочая тетрадь, Босова Л.Л., Босова А.Ю., 2014
  • Информатика, 8 класс, Рабочая тетрадь, Босова Л.Л., Босова А.Ю., 2014
  • Информатика, 9 класс, Рабочая тетрадь, Гейн А.Г., 2014
  • Информатика, 9 класс, Рабочая тетрадь, Гейн А.Г., 2014
  • Информатика, контрольные работы для 2 класса, Матвеева Н.В., Челак Е.Н., Конопатова Н.К., 2014
  • Информатика, контрольные работы для 2 класса, Матвеева Н.В., Челак Е.Н., Конопатова Н.К., 2014
  • Информатика, контрольные работы для 3 класса, Матвеева Н.В., Челак Е.Н., Конопатова Н.К., 2014
  • Информатика, контрольные работы для 3 класса, Матвеева Н.В., Челак Е.Н., Конопатова Н.К., 2014
  • Информатика, рабочая тетрадь для 2 класса, в 2 частях Часть 2, Матвеева Н.В., Челак Е.Н., Конопатова Н.К., 2014
  • Информатика, рабочая тетрадь для 2 класса, в 2 частях, Часть 1, Матвеева Н.В., Челак Е.Н., Конопатова Н.К., 2014
  • Информатика, рабочая тетрадь для 3 класса, в 2 частях, Часть 1, Матвеева Н.В., Челак Е.Н., Конопатова Н.К., 2014
  • Информатика, рабочая тетрадь для 3 класса, в 2 частях, Часть 2, Матвеева Н.В., Челак Е.Н., Конопатова Н.К., 2014
  • Информатика, рабочая тетрадь для 4 класса, в 2 частях, Часть 1, Матвеева Н.В., Челак Е.Н., Конопатова Н.К., 2014
  • Информатика, рабочая тетрадь для 4 класса, в 2 частях, Часть 2, Матвеева Н.В., Челак Е.Н., Конопатова Н.К., 2014
  • Информатика, Рабочая тетрадь, 7 класс, Босова Л.Л., Босова А.Ю., 2014
  • Информатика, Рабочая тетрадь, 8 класс, Босова Л.Л., 2014
  • Информатика, рабочая тетрадь, 8 класс, Босова Л.Л., Босова А.Ю., 2014
  • Типовые задачи по формированию универсальных учебных действий, 4 класс, Работа с информацией, Хиленко Т.П., 2014
  • Типовые задачи по формированию универсальных учебных действий, Работа с информацией, 4 класс, Хиленко Т.П., 2014

  • ГИА 2013 по информатике, 9 класс, Демонстрационный вариант, Техно
  • ГИА 2013 по информатике, 9 класс, Диагностическая работа №1, 2012
  • ГИА 2013, Информатика и ИКТ, 9 класс, Кодификатор
  • ГИА 2013, Информатика и ИКТ, 9 класс, Спецификация
  • ГИА 2013, Информатика и ИКТ, Типовые экзаменационные варианты, 10 вариантов, Крылов С.С., Чуркина Т.Е.
  • ГИА 2013, Информатика и ИКТ, Типовые экзаменационные варианты, 10 вариантов, Крылов С.С., Чуркина Т.Е., 2013
  • ГИА 2013, Информатика, 9 класс, Демонстрационный вариант
  • ГИА 2013, Информатика, 9 класс, Диагностическая работа №1, Вариант 3-4, 2012
  • ГИА 2013, Информатика, 9 класс, Диагностическая работа №2
  • ГИА 2013, Информатика, 9 класс, Кодификатор
  • ГИА 2013, Информатика, 9 класс, Спецификация
  • ГИА 2013, Информатика, 9 класс, Тренировочная работа №1, 2012
  • ГИА 2013, Информатика, 9 класс, Тренировочная работа №2
  • ГИА 2013, Информатика, 9 класс, Тренировочная работа №3
  • ГИА 2013, Информатика, 9 класс, Тренировочная работа №4
  • ГИА 2013, Информатика, 9 класс, Тренировочные варианты экзаменационных работ, Кириенко Д.П., 2013
  • ГИА 2013, Информатика, 9 класс, Тренировочные варианты экзаменационных работ, Кириенко Д.П., Осипов П.О., Чернов А.В.
  • ГИА 2013, Информатика, Диагностические работы, Вареникова Н.В., Шереметьев В.Э.
  • ГИА 2013, Информатика, Тематические тренировочные задания, Зорина Е.М., Зорин М.В., 2012
  • ГИА 2014, Информатика, 9 класс, Диагностическая работа, Варианты 90301-90302, 2013
  • ГИА 2014, Информатика, 9 класс, Тренировочная работа №1, Варианты 9101-9104, 2013
  • ГИА по информатике 2013, 9 класс, Демонстрационный вариант
  • Занимательные задачи по информатике, Босова Л.Л., Босова А.Ю., Коломенская Ю.Г., 2013
  • Занимательные задачи по информатике, Босова Л.Л., Босова А.Ю., Коломенская Ю.Г., 2013
  • Информатика и ИКТ, 9 класс, Подготовка к ГИА 2014, Евич Л.Н., Кулабухов С.Ю., Лисица С.Ю., 2013
  • Информатика и ИКТ, 9 класс, Рабочая тетрадь, Босова Л.Л., Босова А.Ю., 2013
  • Информатика и ИКТ, рабочая тетрадь для 9 класса, Босова Л.Л., Босова А.Ю., 2013
  • Информатика, 2 класс, Рабочая тетрадь, Часть 1, Матвеева Н.В., Челак Е.Н., Конопатова Н.К., 2013
  • Информатика, 4 класс, Контрольные работы, Матвеева Н.В., Челак Е.Н., Конопатова Н.К., 2013
  • Информатика, 5 класс, Рабочая тетрадь, Босова Л.Л., Босова А.Ю., 2013
  • Информатика, 6 класс, Рабочая тетрадь, Босова Л.Л., 2013
  • Информатика, 7 класс, Рабочая тетрадь, Босова Л.Л., 2013
  • Информатика, 8 класс, Рабочая тетрадь, Гейн А.Г., Гейн А.А., 2013
  • Информатика, 9 класс, Ответы на экзаменационные билеты, Чуркина Т.Е., 2013
  • Информатика, контрольные работы для 4 класса, Матвеева Н.В., Челак Е.Н., Конопатова Н.К., 2013
  • Информатика, контрольные работы, 4 класс, Матвеева Н.В., Челак Е.Н., Конопатова Н.К., 2013
  • Информатика, рабочая тетрадь для 6 класса, Босова Л.Л., Босова А.Ю., 2013
  • Информатика, рабочая тетрадь, 5 класс, Босова Л.Л., Босова А.Ю., 2013
  • Информатика, рабочая тетрадь, 5-й класс, перевод с украинского, Ривкинд И.Я., 2013
  • Информатика, Тетрадь для рисования, Для детей 4-5 лет, Часть 1, 2013
  • Информатика, Тетрадь для рисования, Для детей 4-5 лет, Часть 2, 2013
  • Контрольные и самостоятельные работы по информатике, 4 класс, Тур С.Н., Бокучава Т.П., 2013
  • Решение типовых экзаменационных задач по информатике, Дергачева Л.М., 2013
  • Тесты по информатике, 1 класс, Часть 1, Крылова О.Н., 2013
  • Тесты по информатике, 1 класс, Часть 1, Крылова О.Н., 2013
  • Тесты по информатике, 1 класс, Часть 2, Крылова О.Н., 2013
  • Тесты по информатике, 1 класс, Часть 2, Крылова О.Н., 2013
  • Тесты по информатике, 2 класс, Часть 1, Крылова О.Н., 2013
  • Тесты по информатике, 2 класс, Часть 2, Крылова О.Н., 2013
  • Тесты по информатике, 3 класс, Часть 1, Крылова О.Н., 2013
  • Тесты по информатике, 3 класс, Часть 1, Крылова О.Н., 2013
  • Тесты по информатике, 3 класс, Часть 2, Крылова О.Н., 2013
  • Тесты по информатике, 3 класс, Часть 2, Крылова О.Н., 2013
  • Тесты по информатике, 4 класс, Часть 1, Крылова О.Н., 2013
  • Тесты по информатике, 4 класс, Часть 2, Крылова О.Н., 2013
  • Тренировочная работа № 2 по ИНФОРМАТИКЕ 11 класс, Вариант 1, 15 февраля 2013 года
  • Учебник-тетрадь по информатике, 1 класс, Тур С.Н., Бокучава Т.П., 2013
  • Учебник-тетрадь по информатике, 4 класс, Тур С.Н., Бокучава Т.П., 2013

  • ГИА 2012, Информатика и ИКТ, 9 класс, Демонстрационный вариант
  • ГИА 2012, Информатика и ИКТ, 9 класс, Демонстрационный вариант, 2012
  • ГИА 2012, Информатика и ИКТ, 9 класс, Кодификатор
  • ГИА 2012, Информатика и ИКТ, 9 класс, Кодификатор, 2012
  • ГИА 2012, Информатика и ИКТ, 9 класс, Методические материалы
  • ГИА 2012, Информатика и ИКТ, 9 класс, Методическое пособие, 2011
  • ГИА 2012, Информатика и ИКТ, 9 класс, Спецификация
  • ГИА 2012, Информатика и ИКТ, 9 класс, Спецификация, 2012
  • ГИА 2012, Информатика, 9 класс, Варианты 1201-1203
  • ГИА 2012, Информатика, 9 класс, Демонстрационный вариант
  • ГИА 2012, Информатика, 9 класс, Диагностическая работа №1, 2011
  • ГИА 2012, Информатика, 9 класс, Диагностическая работа №2
  • ГИА 2012, Информатика, 9 класс, Тренировочная работа №1
  • ГИА 2012, Информатика, 9 класс, Тренировочная работа №1, 2011
  • ГИА 2012, Информатика, 9 класс, Тренировочная работа №2
  • ГИА 2012, Информатика, 9 класс, Тренировочная работа №3
  • ГИА 2012, Информатика, 9 класс, Тренировочная работа №4
  • ГИА 2012, Информатика, Контрольные тренировочные материалы для 9 класса, Авдошин С.М., 2012
  • ГИА 2012. Информатика. 9 класс. Демонстрационный вариант. Проект. 2011
  • ГИА 2013 по информатике, 9 класс, Диагностическая работа №1, 2012
  • ГИА 2013, Информатика, 9 класс, Диагностическая работа №1, Вариант 3-4, 2012
  • ГИА 2013, Информатика, 9 класс, Тренировочная работа №1, 2012
  • ГИА 2013, Информатика, Тематические тренировочные задания, Зорина Е.М., Зорин М.В., 2012
  • ГИА, Информатика, 9 класс, Демонстрационный вариант, 2012
  • Информатика и ИКТ, 3 класс, Контрольные работы, Матвеева Н.В., Челак Е.Н., 2012
  • Информатика и ИКТ, 3 класс, Контрольные работы, Матвеева Н.В., Челак Е.Н., Конопатова Н.К., Панкратова Л.П., 2012
  • Информатика и ИКТ, 4 класс, Контрольные работы, Матвеева Н.В., Челак Е.Н., 2012
  • Информатика и ИКТ, 5 класс, Рабочая тетрадь, Босова Л.Л., 2012
  • Информатика и ИКТ, 5 класс, Рабочая тетрадь, Босова, 2012
  • Информатика и ИКТ, 6 класс, Рабочая тетрадь, Босова Л.Л., 2012
  • Информатика и ИКТ, 6 класс, Рабочая тетрадь, Босова, 2012
  • Информатика и ИКТ, 7 класс, Рабочая тетрадь, Босова Л.Л., 2012
  • Информатика и ИКТ, 7 класс, Рабочая тетрадь, Босова, 2012
  • Информатика и ИКТ, 8 класс, Рабочая тетрадь, Босова Л.Л., 2012
  • Информатика и ИКТ, 8 класс, Рабочая тетрадь, Босова, 2012
  • Информатика и ИКТ, 9 класс, Рабочая тетрадь, Босова Л.Л., 2012
  • Информатика и ИКТ, Задачник-практикум, Том 1, Залогова Л.А., 2012
  • Информатика и ИКТ, Задачник-практикум, Том 2, Залогова Л.А., 2012
  • Информатика и ИКТ, Задачник-практикум, часть 1, Семакин И.Г., Хеннер Е.К., 2012
  • Информатика и ИКТ, Контрольные работы, 2 класс, Матвеева Н.В., Челак Е.Н., Конопатова Н.К., 2012
  • Информатика и ИКТ, Контрольные работы, 3 класс, Матвеева Н.В., Челак Е.Н., Конопатова Н.К., 2012
  • Информатика и ИКТ, Контрольные работы, 4 класс, Матвеева Н.В., Челак Е.Н., Конопатова Н.К., 2012
  • Информатика и ИКТ, рабочая тетрадь для 7 класса, Босова Л.Л., 2012
  • Информатика и ИКТ, Рабочая тетрадь, 7 класс, Босова Л.Л., 2012
  • Информатика и ИКТ, Рабочая тетрадь, 9 класс, Босова Л.Л., 2012
  • Информатика и ИКТ. Задачник-практикум, часть 2, Семакин И.Г., Хеннер Е.К., 2012
  • Информатика и информационные технологии, 9 класс, Рабочая тетрадь, Гейн А.Г., Юнерман Н.А., 2012
  • Информатика, 2 класс, Рабочая тетрадь, Рудченко Т.А., Семёнов А.Л., 2012
  • Информатика, 2 класс, Рабочая тетрадь, Рудченко Т.А., Семенов А.Л., 2012
  • Информатика, 2 класс, Рабочая тетрадь, Рудченко Т.А., Семенов А.Л., 2012
  • Информатика, 2 класс, Рабочая тетрадь, Часть 1, Матвеева Н.В., Челак Е.Н., 2012
  • Информатика, 2 класс, Рабочая тетрадь, Часть 2, Матвеева Н.В., Челак Е.Н., 2012
  • Информатика, 2 класс, Рабочая тетрадь, Часть 2, Матвеева, Челак, 2012
  • Информатика, 2 класс, Тетрадь проектов, Рудченко Т.А., Семенов А.Л., 2012
  • Информатика, 2 класс, Тетрадь проектов, Рудченко Т.А., Семенов А.Л., 2012
  • Информатика, 5 класс, Рабочая тетрадь, Босова Л.Л., 2012
  • Информатика, 6 класс, Рабочая тетрадь, Босова Л.Л., 2012
  • Информатика, 7 класс, Рабочая тетрадь, Босова Л.Л., 2012
  • Информатика, 7 класс, Рабочая тетрадь, Гейн А.Г., Гейн А.А., 2012
  • Информатика, 8 класс, Рабочая тетрадь, Босова Л.Л., 2012
  • Информатика, 9 класс, Рабочая тетрадь, Босова Л.Л., 2012
  • Информатика, Контрольные работы, 2 класс, Матвеева Н.В., Челак Е.Н., 2012
  • Информатика, Лабораторный практикум, Грошев А.С., 2012
  • Информатика, рабочая тетрадь для 2-го класса, Часть 1, Матвеева Н.В., Челак Е.Н., 2012
  • Информатика, Рабочая тетрадь, 7 класс, Босова Л.Л., 2012
  • Типовые задачи по формированию универсальных учебных действий, Работа с информацией, 1 класс, Хиленко Т.П., 2012
  • Типовые задачи по формированию универсальных учебных действий, Работа с информацией, 1 класс, Хиленко Т.П., 2012

  • ГИА 2011, Информатика, 9 класс, Демонстрационный вариант
  • ГИА 2011, Информатика, 9 класс, Демонстрационный вариант, 2011
  • ГИА 2011, Информатика, 9 класс, Диагностическая работа
  • ГИА 2011, Информатика, 9 класс, Кодификатор
  • ГИА 2011, Информатика, 9 класс, Методические рекомендации, Кириенко Д.П., 2011
  • ГИА 2011, Информатика, 9 класс, Рекомендации
  • ГИА 2011, Информатика, 9 класс, Спецификация
  • ГИА 2011, Информатика, 9 класс, Тренировочная работа №1, 2011
  • ГИА 2011, Информатика, 9 класс, Тренировочные варианты экзаменационных работ, Кириенко Д.П., 2011
  • ГИА 2011, Информатика, 9 класс, Тренировочные варианты экзаменационных работ, Кириенко Д.П., Осипов П.О., Чернов А.В., 2011
  • ГИА 2011, Информатика, Методические рекомендации, Андреева Е.В., Лещинер В.Р., Самылкина Н.Н., Якушкин П.А., Крылов С.С., 2011
  • ГИА 2012, Информатика и ИКТ, 9 класс, Методическое пособие, 2011
  • ГИА 2012, Информатика, 9 класс, Диагностическая работа №1, 2011
  • ГИА 2012, Информатика, 9 класс, Тренировочная работа №1, 2011
  • ГИА 2012. Информатика. 9 класс. Демонстрационный вариант. Проект. 2011
  • ГИА, Информатика и ИКТ, 9 класс, Учебно-справочные материалы, Авдошин С.М., Ахметсафина Р.З., Максименкова О.В., 2011
  • Информатика и ИКТ, 3 класс, Рабочая тетрадь, Часть 1, Матвеева Н.В., Челак Е.Н., 2011
  • Информатика и ИКТ, 3 класс, Рабочая тетрадь, Часть 2, Матвеева Н.В., Челак Е.Н., 2011
  • Информатика и ИКТ, 4 класс, Рабочая тетрадь, Часть 1, Матвеева Н.В., 2011
  • Информатика и ИКТ, 4 класс, Рабочая тетрадь, Часть 1, Матвеева Н.В., Челак Е.Н., 2011
  • Информатика и ИКТ, 4 класс, Рабочая тетрадь, Часть 2, Матвеева Н.В., 2011
  • Информатика и ИКТ, 4 класс, Рабочая тетрадь, Часть 2, Матвеева Н.В., Челак Е.Н., 2011
  • Информатика и ИКТ, 9 класс, Подготовка к ГИА 2011, Лысенко Ф.Ф., Евич Л.Н.
  • Информатика и ИКТ, 9 класс, Подготовка к ГИА 2011, Лысенко Ф.Ф., Евич Л.Н., 2011
  • Информатика и ИКТ, 9 класс, Подготовка к ГИА, Лысенко Ф.Ф., Евич Л.Н., 2011
  • Информатика и ИКТ, задачник-практикум, в 2 томах Том 1, Залогов Л.А., Семакин И.Г., Хеннер Е.К., 2011
  • Информатика и ИКТ, задачник-практикум, в 2 томах Том 2, Залогова Л.А., Семакин И.Г., Хеннер Е.К., 2011
  • Информатика и ИКТ, Задачник-практикум, Том 1, Семакин И.Г., Хеннер Е.К., 2011
  • Информатика и ИКТ. Задачник-практикум, Том 2, Семакин И.Г., Хеннер Е.К., 2011
  • Информатика, 2 класс, Рабочая тетрадь, Часть 1, Матвеева Н.В., Челак Е.Н., Конопатова Н.К., 2011
  • Информатика, 2 класс, Рабочая тетрадь, Часть 2, Матвеева Н.В., Челак Е.Н., Конопатова Н.К., 2011
  • Информатика, 8 класс, Тематические тестовые задания для подготовки к ГИА, Ярцева О.В., Цикина Е.Н., 2011
  • Информатика, 9 класс, Тематические тесты для подготовки к ГИА, Евич Л.Н., Лысенко Ф.Ф., 2011
  • Информатика, 9 класс, Тематические тесты для подготовки к ГИА-9, Евич Л.Н., Лысенко Ф.Ф., 2011
  • Информатика, ГИА, Учебно-справочные материалы, 9 класс, Авдошин С.М., Ахметсафина Р.З., Максименкова О.В., 2011
  • Итоговые тесты по информатике, 11 класс, Чуркина Т.Е., 2011
  • Итоговые тесты по информатике, 11 класс, Чуркина, 2011
  • Подготовка школьников к олимпиаде по программированию, Казнахмедов Т.Б., 2011
  • Сборник задач по программированию, Златопольский Д.М., 2011
  • Тесты по информатике, 2 класс, Крылова О.Н., 2011
  • Тесты по информатике, 2 класс, Крылова, 2011
  • Тесты по информатике, 3 класс, Крылова О.Н., 2011
  • Тесты по информатике, 4 класс, Крылова О.Н., 2011
  • Учебник-тетрадь по информатике, 2 класс, Тур С.Н., Бокучава Т.П., 2011
  • Учебник-тетрадь по информатике, 3 класс, Тур С.Н., Бокучава Т.П., 2011

  • ГИА 2010, Информатика и ИКТ, 9 класс, Демонстрационный вариант
  • ГИА 2010, Информатика и ИКТ, 9 класс, Кодификатор
  • ГИА 2010, Информатика и ИКТ, 9 класс, Спецификация
  • ГИА 2010, Информатика, 9 класс, Демонстрационный вариант, 2010
  • Информатика и ИКТ, 10-11 класс, Задачник-практикум, Гейн А.Г., 2010
  • Информатика и ИКТ, 2 класс, Рабочая тетрадь, Часть 2, Матвеева Н.В., Челак Е.Н., Конопатова Н.К., Панкратова Л.П., 2010
  • Информатика и ИКТ, задачник-практикум, 10-11 классы, базовый и профильный уровни, Гейн А.Г., 2010
  • Информатика и ИКТ, Тематические тесты, 10 класс, Гейн А.Г., 2010
  • Информатика и ИКТ, Тематические тесты, 11 класс, Гейн А.Г., Юнерман Н.А., 2010
  • Информатика и информационные технологии, 9 класс, Рабочая тетрадь, Гейн А.Г., Юнерман Н.А., 2010
  • Информатика и информационные технологии, 9 класс, Тематические тесты, Гейн А.Г., Юнерман Н.А., 2010

  • ГИА 2009, Информатика, 9 класс, Демонстрационный вариант, 2009
  • ГИА 2009, Информатика, 9 класс, Кодификатор
  • ГИА 2009, Информатика, 9 класс, Спецификация
  • Информатика и информационные технологии, 8 класс, Рабочая тетрадь, Гейн А.Г., Юнерман Н.А., 2009
  • Информатика и информационные технологии, 8 класс, Тематические тесты, Гейн А.Г., Юнерман Н.А., 2009

  • Информатика и ИКТ, 10-11 класс, Часть 1, Профильный уровень, Фиошин М.Е., Ресин А.А., Юнусов С.М., 2008
  • Информатика и ИКТ, 10-11 класс, Часть 2, Профильный уровень, Фиошин М.Е., Ресин А.А., Юнусов С.М., 2008
  • Информатика и информационные технологии — Шпаргалки — Цветкова А.В.
  • Информатика и информационные технологии, 8-9 класс, Задачник-практикум, Гейн А.Г., 2008
  • Информатика: тесты, задания, лучшие методики — Молодцов В.А, Рыжикова Н.Б.
  • Программирование, Козлова И.С., 2008

  • Алгоритмы и программы, Решение олимпиадных задач, Порублев И.Н., Ставровский А.Б., 2007
  • Занимательные задачи по информатике, Задачник, 5-6 класс, Босова Л.Л., Босова А.Ю., Коломенская Ю.Г., 2007
  • Занимательные задачи по информатике, издание 3, Босова Л.Л., Босова А.Ю., Коломенская Ю.Г., 2007
  • Информатика — 7-9 класс — Базовый курс — Практикум-задачник по моделированию — Макарова Н.В.
  • Информатика — Примерные билеты для 9 классов
  • Информатика в примерах и задачах, 10-11 класс, Казиев В.М., 2007
  • Информатика и ИКТ — Задачник по моделированию — 9-11 класс — Базовый уровень — Макарова Н.В., 2007
  • Информатика и ИКТ, Задачник по моделированию, 9-11 класс, Базовый уровень, Макарова Н.В., 2007
  • Итоговые тесты по информатике для 10-11 классов, Кошелев, 2007
  • Итоговые тесты по информатике, 10-11 классы, Кошелев М.В., 2007
  • Лабораторный практикум по курсу “Теория автоматического управления”, Линейные непрерывные динамические системы, Шапкарин А.В., Кулло И.Г., 2007
  • Методика решения задач по информатике, Международные олимпиады, Кирюхин В.М., 2007
  • Методика решения задач по информатике, Международные олимпиады, Кирюхин В.М., Окулов С.М., 2007
  • Методика решения задач по информатике, Международные олимпиады, Кирюхин В.М., Окулов С.М., 2007
  • Сборник задач по программированию — Златопольский Д.М.
  • Сборник задач по программированию — Златопольский Д.М. — 2007



  • 1000 задач по программированию, Часть I, Абрамян М.Э., 2004
  • 1000 задач по программированию, Часть II, Абрамян М.Э., 2004
  • 1000 задач по программированию, Часть III, Абрамян М.Э., 2004
  • Контроль знаний по информатике — тесты, контрольные задания, экзаменационные вопросы, компьютерные проекты — Панкратова Л.П., Челак Е.Н. — 2004
  • Контроль знаний по информатике: тесты, контрольные задания, экзаменационные вопросы, компьютерные проекты — Панкратова Л.П., Челак Е.Н. — 2004
  • Методические указания для подготовки к вступительному экзамену в МГИУ по дисциплине «Информатика» — Роганов Е.А., Роганова Н.А. — 2004
  • Программирование, Теоремы и задачи, Шень А., 2004
  • Тесты — Информатика и информационные технологии. 6 — 11 классы — Анеликова Л.А



  • Информатика, Сборник задач и решений, Есипов А.С., Паньгина H.Н., Громада М.И., 2001
  • Информатика, экзаменационные ответы, Семенов М.В., 2001
  • Школьная информатика, Экзаменационные вопросы и ответы, Радченко Н.П., Козлов О.А., 2001


  • Зошит для контролю знань з інформатики, 6 клас, Морзе Н.В., Барна О.В., Вембер В.П.
  • Информатика — 11 класс — 25 билетов с ответами — Угринович Н.Д.
  • Информатика и ИКТ — Рабочая тетрадь для 6 класса — Босова Л.Л.
  • Информатика и ИКТ — Экзаменационные билеты для 11 классов — Профильное обучение
  • Информатика и ИКТ, 7 класса, Рабочая тетрадь, Босова Л.Л.
  • Информатика и ИКТ, 8 класс, Рабочая тетрадь, Босова Л.Л., Босова А.Ю.
  • Информатика и ИКТ, Рабочая тетрадь, 5 класс, Босова Л.Л.
  • Информатика, Программное обеспечение компьютера, Тест
  • Информатика, Сборник заданий формативного оценивания, 7 класс
  • Информатика, Сборник заданий формативного оценивания, 8 класс
  • Информатика, Сборник заданий формативного оценивания, 9 класс
  • Информатика, Состав и работа компьютерной системы, Тест
  • Информатика, Тестовые задания
  • Информатика, Тесты
  • Ответы на экзаменационные вопросы по Информатике — 11 класс
  • Ответы на экзаменационные вопросы по Информатике — 9 класс
  • Поурочный план по информатике — Проверочная работа по теме «Файлы. Файловая система»
  • Сборник заданий формативного оценивания, Информатика, 5 класс
  • Сборник заданий формативного оценивания, Информатика, 6 класс
  • Сборник заданий формативного оценивания, Информатика, Естественно-математическое направление, 10 класс
  • Сборник заданий формативного оценивания, Информатика, ОПГ, 10 класс
  • Сборник заданий формативного оценивания, Информационно-коммуникационные технологии, 3 класс
  • Сборник заданий формативного оценивания, Информационно-коммуникационные технологии, 4 класс
  • Тест по информатике
  • Тест по информатике — Графический редактор
  • Тест по информатике — компьютерные коммуникации
  • Тест по информатике — электронные таблицы
  • Тест по информатике на тему — Microsoft Word
  • Тест по информатике на тему «Программное обеспечение компьютера»
  • Тесты по информатике — Комьютерная графика
  • Учусь создавать проект, 2 класс, Рабочая тетрадь, Часть 2, Сизова Р.И., Селимова Р.Ф.

Описание раздела «Экзамены по Информатике»

В данном разделе мы предлагаем вам для ознакомления Экзаменационные билеты по информатике, которые обязательно вам помогут хорошо подготовиться к сдаче любого экзамена по данному предмету. Используя данные материалы вы без особого труда сможете выполнить на отлично любую контрольную работу, сдать экзамен ОГЭ 2015.

Для того, чтобы составить план подготовки, смотрите расписание ОГЭ 2015.

     К экзамену по информатике нужно готовиться плодотворно, потому что информатику и ИКТ начинают уже изучать чуть ли не с младших классов. Поэтому в данной категории подобрана литература за 4, 5, 6, 7, 8, 9, 10, 11 классы. Тесты, контрольные задания, компьютерные проекты, экзаменационные вопросы, самые полные издания типовых вариантов реальных заданий ЕГЭ 2014, 2013, 2012, 2011, 2010, 2009, ответы на вопросы ГИА, решения сложных задач, сборники экзаменационных заданий, универсальные раздаточные материалы и многое другое.

     Информатика – это компьютерная наука, наука о способах получения, накопления, хранения, преобразования, передачи, защиты и использования информации. Она включает дисциплины, относящиеся к обработке информации в вычислительных машинах и вычислительных сетях: как абстрактные, вроде анализа алгоритмов, так и довольно конкретные, например, разработка языков программирования.

     Так же трудность сдачи экзамена по информатики заключается в том, что обычно он состоит из нескольких этапов. Это тестирование, в котором собраны вопросы по всему курсу информатики. Устная часть, в которой идет обсуждение основных тем предмета. Это понятие информации, основных устройств компьютера, принципа работы компьютера и многое другое. И самый сложный этап — это построение алгоритмов и написание программ. Мы предлагаем вам билеты и ответы на билеты по информатике по всем темам и видам.

     Подготовившись к экзамену не забудьте посетить раздел Книги по информатике, Книги по программированию, раздел Обучение информатике. Подготовиться к ЭГЭ вам поможет раздел ЕГЭ 2014 по информатике.

 

Алгоритмы и исполнители. Информатика. 9 класс. Разработка урока

Тип урока: изучение нового материала.

Цели:

  • образовательные: познакомить учащихся с понятиями алгоритм, алгоритмизация, исполнители алгоритмов и система команд исполнителя; перечислить и проанализировать свойства алгоритма; познакомить учащихся с формами записи алгоритмов.
  • воспитательные: воспитывать аккуратность, внимательность, точность и дисциплинированность.
  • развивающие: развитие внимания, восприятия, самостоятельного анализа, познавательного интереса у учащихся, умения обобщать и сравнивать

Задачи: организовать и направить познавательную деятельность обучающихся на понимание сути алгоритмов, их свойств, способов описания; показать связь данной темы с практикой; применять знания при создании алгоритмов и оценке существующих алгоритмов; формирование умения четко организовать самостоятельную и групповую работу.

Использованные источники: Ю.А.Быкадоров, Информатика и ИКТ. 9 класс, учебник для общеобразовательных учреждений, М.: Дрофа. 2013 г.; О. Н. Масленикова, Информатика и ИКТ. 9 класс. Методическое пособие к учебнику Ю. А. Быкадорова «Информатика и ИКТ. 9 класс», М.: Дрофа. 2013 г.  

Ход урока

Здравствуйте ребята.

Для того, чтобы узнать тему сегодняшнего урока нужно разгадать ребус.


И так, кто уже готов назвать тему нашего урока? (Ответы учащихся…)

Молодцы! Правильно отгадали!

Тема нашего урока «Алгоритмы и исполнители».

Цель нашего урока – выяснить, что такое алгоритм, познакомится с историей возникновения данного понятия, его свойствами, видами алгоритмов и формами, с помощью которых можно записать тот или иной алгоритм, а также где в реальной жизни мы встречаемся с алгоритмами.

С понятием «алгоритм» вы уже знакомились на других предметах и в своей повседневной деятельности нам постоянно приходится сталкиваться с разнообразными правилами, предписывающими последовательность действий, цель которых состоит в достижении некоторого необходимого результата. Подобные правила очень многочисленны. Например, мы обязаны следовать вполне определенной системе правил, чтобы найти корни квадратного уравнения, приготовить кофе и т.д. Все мы живем в мире алгоритмов. Алгоритмы экономят силы и время человека, так как однажды усвоенным правилом (алгоритмом) вы можете пользоваться всю жизнь.

  • Под алгоритмом понимают понятное и точное предписание исполнителю выполнить конечную последовательность действий для достижения поставленной цели.

А теперь давайте немного поговорим об истории происхождения слова алгоритм.Мой помощник, ваш одноклассник, подготовил историческую справку о происхождении слова «алгоритм», для этого он использовал – учебник, справочники, интернет-ресурсы.

Ученик. Историческая справка. Происхождение слова «алгоритм»

Правила выполнения арифметических действий над целыми числами и простыми дробями в десятичной системе счисления впервые были сформулированы выдающимся средневековым ученым по имени Мухаммед ибн Муса ал-Хорезми (в переводе с арабского это означает «Мухаммед, сын Мусы из Хорезма»), сокращенно Ал-Хорезми.

Ал-Хорезми жил и творил в IX веке. Арабский оригинал его арифметического труда утерян, но имеется латинский перевод XII века, по которому Западная Европа ознакомилась с десятичной позиционной системой счисления и правилами выполнения в ней арифметических действий.

Ал-Хорезми стремился к тому, чтобы сформулированные им правила были понятны для всех грамотных людей. Достичь этого в веке, когда еще не была разработана математическая символика (знаки операций, скобки, буквенные обозначения и т. п.), было очень трудно. Но Ал-Хорезми удалось выработать в своих трудах такой стиль четкого, строгого словесного предписания, который не давал читателю никакой возможности уклониться от предписанного или пропустить какие-нибудь действия.

В латинском переводе книги Ал-Хорезми правила начинались словами «Алгоризми сказал». С течением времени люди забыли, что «Алгоризми» — это автор правил, и стали сами эти правила называть алгоритмами. Постепенно «Алгоризми сказал» преобразовалось в «алгоритм гласит».

Таким образом, слово «алгоритм» происходит от имени ученого Ал-Хорезми. Как научный термин первоначально оно обозначало лишь правила выполнения действий в десятичной системе счисления. С течением времени это слово приобрело более широкий смысл и стало обозначать любые точные правила действий. В настоящее время слово «алгоритм» является одним из важнейших понятий науки информатики.

Процесс создания алгоритмов называется – алгоритмизацией.

Всякий алгоритм составляется в расчете на определенного исполнителя. Им может быть человек, робот, компьютер и др.

  • Исполнитель алгоритма – это человек или автоматическое устройство, которое способно воспринимать и исполнять алгоритм.

Запишите исполнителей для приведённых ниже видов работ:

  • Уборка мусора во дворе – дворник
  • Перевозка пассажиров в поезде – машинист
  • Приём экзаменов в школе – учитель
  • Приготовление еды в ресторане – повар
  • Выполнение домашнего задания – ученик

Чтобы составить алгоритм для исполнителя, нужно знать, какие команды исполнитель может понять и исполнить, а какие нет.

  • Система команд исполнителя (СКИ) – это перечень элементарных предписаний (команд), которые исполнитель может исполнять.

Пример. Алгоритм определения периметра прямоугольника:

Дано: А, В — длины сторон прямоугольника.

Найти: Р – периметр прямоугольника.

Математическая модель: Р = (А + В) 2

  1. Задать числовые значения А, В.
  2. Сложить А и В. Результат обозначить X.
  3. Умножить X на 2. Результат обозначить Р.
  4. Записать в качестве ответа значение Р.
  5. Конец.

Данный алгоритм рассчитан на исполнителя старшеклассника. Первоклассник выполнить этот алгоритм не может, т.к. умножать он еще не научился. Первокласснику можно команду умножения заменить командой сложения (X и X). Вообще, чем меньше запас умений школьника, тем подробнее будет составленный для него алгоритм.

Приведите еще примеры алгоритмов. Ответы учащихся …

Из приведенных вами примеров видно, что мир алгоритмов очень разнообразен. Но, несмотря на это, можно выделить общие свойства, которыми обладает любой алгоритм.

Алгоритм обладает следующими свойствами:

  • Целенаправленность – любой алгоритм направлен на достижение определенной цели. Чаще всего целью алгоритма является получение результата при решении какой-нибудь задачи.
  • Дискретность – алгоритм состоит из элементарных предписаний (команд).
  • Понятность – элементарные предписания (команды) алгоритма должны быть точно сформулированы и однозначно понятны исполнителю, а исполнитель должен быть в состоянии их выполнить.
  • Однозначность – после исполнения очередного элементарного предписания (команды) исполнителю точно определено, что делать дальше.
  • Массовость – алгоритм можно использовать для решения той же задачи при других допустимых исходных данных.

 Формы представления алгоритмов могут быть разными: словесной; графической; на языке программирования.

Рассмотрим их:

1. Словесная форма – это форма описания алгоритма на естественном языке. Если алгоритм предназначен для человека, то в качестве предписаний можно использовать привычные для человека предложения и фразы.

Правила записи алгоритмов в словестной форме просты: предписания записываются одно за другим и нумеруются; в записи алгоритма могут использоваться служебные слова Начало и Конец.

Пример: Алгоритм нахождения большего из двух данных чисел.

  1. Начало.
  2. Из числа А вычесть число В.
  3. Если получилось отрицательное значение, то сообщить, что число В больше.
  4. Если получилось положительное значение, то сообщить, что число А больше.
  5. Если получился ноль, сообщить, что числа равны.
  6. Конец.

Данная форма очень удобна, если нужно приближенно описать суть алгоритма. Однако при словесном описании не всегда удается ясно и точно выразить идею.

2. Для более наглядного представления алгоритма используется графическая форма. Графическая форма – изображение алгоритма в виде последовательности связанных между собой функциональных блоков, каждый из которых соответствует выполнению одного или нескольких действий.

3. При записи алгоритма в словесной и в графической форме допускается определенный произвол при изображении команд. Вместе с тем такая запись точна на столько, что позволяет человеку понять суть дела и исполнить алгоритм. Однако на практике в качестве исполнителей алгоритмов используются специальные автоматы – компьютеры. Поэтому алгоритм, предназначенный для исполнения на компьютере, должен быть записан на понятном ему языке. Такой язык принято называть языком программирования, а форму представления алгоритма — программной. То есть программная форма записи алгоритма – это запись на языке программирования.

Самостоятельная работа в группах по карточкам. Командир группы о результатах сообщает учителю.

Примерные вопросы:

  1. Приведите примеры известных Вам алгоритмов.
  2. Запишите алгоритм заварки чая.
  3. Перечислите основные свойства алгоритмов и проиллюстрируйте их примерами.
  4. Имеются два кувшина ёмкостью 3 л и 8 л. Напишите алгоритм для того, чтобы набрать из реки 7 л воды (можно пользоваться только этими кувшинами).
  5. Какие Вы знаете формы описания алгоритмов?
  6. Нужно поджарить три куска хлеба на сковороде, вмещающей только два таких куска. На поджаривание каждой стороны уходит 2 минуты. Можно ли поджарить хлеб меньше чем за 8 минут? Если да, то составить алгоритм.
  7. Мачеха велела Золушке принести ровно 3 л. воды, а в доме всего два ведра: одно пятилитровое (ведро М), а другое девятилитровое (ведро Б) . Как же быть? Помогите Золушке. Составьте алгоритм решения задачи.

Присутствуют ли в нашей жизни алгоритмы? Теория алгоритмов имеет большое практическое значение. Алгоритмический тип деятельности важен не только как одна из эффективных форм труда человека. Через алгоритмизацию, через разбиение сложных действий на всё более простые, на действия, выполнение которых доступно машинам, пролегает путь к автоматизации различных процессов.

Домашнее задание: Домашним заданием для вас будет изучение §1 на стр. 3–6 и ответы на вопросы на стр. 7. Составить алгоритм старинной русской задачи: некий человек должен перевезти в лодке через реку волка, козу и капусту. За один перевоз он может перевезти только кого-то одного. Составьте алгоритм перевоза так, чтобы никто никого не съел.

Примечание: при изучении нового материала учащиеся делают в тетрадь необходимые записи под руководством учителя.

Медицинская информатика — ответы на ваши вопросы

Наши консультанты по зачислению ответят на несколько действительно интересных вопросов от будущих студентов! В этом посте мы отвечаем на эти вопросы, связанные с информатикой в ​​области здравоохранения.

Если у вас возникнут какие-либо вопросы по поводу UNE Online или онлайн-образования в целом, оставьте их в комментариях!

Требуется ли степень бакалавра для поступления на программу информатики в области здравоохранения в определенной области?

Нет, степень бакалавра не обязательно должна быть в определенной области.В наших программах по информатике здоровья участвуют люди из всех слоев общества. У нас, как правило, больше студентов, занимающихся клинической деятельностью (врачи, медсестры, терапевты и фармацевты), но у нас также есть несколько студентов, работающих в сфере ИТ, а также студенты со страховым опытом.

Сфера информатики настолько велика, что есть люди из самых разных отраслей и секторов, которые хотят либо начать новую карьеру, либо открыть двери на своем нынешнем карьерном пути.Таким образом, вам не нужна конкретная степень, но вы должны иметь степень бакалавра в регионально аккредитованном учебном заведении.

Магистр информатики в области здравоохранения имеет три основных направления — доступны ли эти направления для получения сертификата об окончании высшего образования?

В выпускном аттестате только шесть классов, поэтому, к сожалению, профильные направления не входят в план курса. Если бы вы продолжили и закончили магистратуру, у вас была бы возможность посещать занятия, которые составляют предметную область, которая вас интересует.Или у вас есть возможность пройти эти два тематических класса в дополнение к учебной программе сертификата, но это будет выше требований вашего сертификата об окончании высшего образования.

Является ли учебная программа теоретической по своему характеру или у нас есть электронная медицинская карта, над которой нужно работать?

Ответ — да и нет. Учебная программа включает как теоретические, так и практические элементы. У вас не будет возможности работать с одной конкретной электронной медицинской записью, потому что, хотя электронные медицинские записи — это огромная часть того, о чем мы говорим, электронные медицинские записи — это всего лишь часть того, что вам нужно знать.Каждая медицинская карта сильно отличается, поэтому мы не можем научить вас использовать ее исключительно, но мы преподаем общие концепции и теории, касающиеся качественной реализации EMR.

Что касается практических навыков, мы обучаем работе в сети, SQL и Tableau. Если вас интересует сфера аналитики данных, вы также будете использовать программу под названием «R», которая представляет собой практический опыт.

Как работает выставление оценок? Курсы в основном экзамены или работы?

На протяжении всего курса вас в основном оценивают с помощью работ и проектов.Мы не предлагаем студентам сдавать какие-либо экзамены, потому что на уровне выпускников мы стремимся развить вас как самостоятельных учеников. Обучение на уровне выпускников — это не столько ответы, правильные или неправильные, сколько размышления о том, «почему» вещей. Так что никаких экзаменов.

Студенты сдают достаточное количество баллов PowerPoints за письменные работы, тематические исследования и озвучивание. Мы стараемся сделать задания максимально интересными и динамичными и согласовывать их с работой, которую вы будете выполнять в реальной отрасли.

По окончании магистерской программы предстоит одна большая дипломная работа.А затем, когда вы берете технические классы, есть много практических занятий, особенно во время сетевых лабораторий, визуализации данных в Tableau и программирования SQL. Нам нужно многому научиться, поэтому мы стараемся сделать это по-настоящему увлекательным.

Какие типы карьеры могут вам пригодиться при получении степени магистра информатики в области здравоохранения?

Получить работу в сфере информатики здоровья — это совсем другое дело. Например, предположим, что вы медсестра, которая ищет работу медсестры. Вы вводите «медсестра» в поле поиска Indeed.com и получаете десять тысяч результатов медсестры.

С информатикой здоровья дело обстоит не так. Существует множество названий, относящихся к сфере информатики здоровья. Специалист в области медицинской информатики может искать такие ключевые слова, как клинический аналитик, аналитик данных, специалист по информатике, информатик и специалист по данным — и это лишь некоторые из них. Это также тип должностей, на которые вы можете претендовать с этой степенью.

Какие новые возможности я могу иметь право после получения степени магистра информатики в области здравоохранения?

Получение степени магистра в общем смысле настраивает вас на лидерство.Даже если это не формальное лидерство, это мысленное лидерство. Получив степень магистра, вы получите возможность претендовать на управленческую работу. Лидерские должности открываются после получения степени магистра.

Еще вопросы? Оставляйте их в комментариях, и мы вам сразу же свяжемся!


Если вы хотите сделать карьеру в области информатики здравоохранения или просто хотите обсудить программу, обратитесь к консультанту по регистрации по телефону (855) 751-4445 или по электронной почте informatics @ une .Эду .

Или заполните онлайн-заявку сегодня на сайте online.une.edu/gateway-portal-page — мы с нетерпением ждем вашего ответа!

Теги: Магистерские программы по информатике здравоохранения | Информатика здравоохранения | Магистр наук в области информатики здравоохранения

Практика информатики, класс 12 Важные вопросы Глава 9 HTML и XML

Практика информатики, класс 12 Важные вопросы Глава 9 HTML и XML является частью важных вопросов класса 12 по информатике.Здесь мы задали важные вопросы урока 12 по информатике, главу 9 HTML и XML.

Практика информатики Класс 12 Важные вопросы Глава 9 HTML и XML

1 балл Вопросы

Вопрос 1.
Какие теги HTML используются для
(i) Изменить шрифт на странице
(ii) Добавить строку в таблицу? (Дели, 2014 г.)
Ответ:
(i) Тег используется для изменения шрифта на странице.
(ii) Тег используется для добавления строки в таблицу.

Вопрос 2.
Какие теги / атрибуты HTML используются для
(i) Вставить изображение на веб-страницу.
(ii) Вставить пустую строку на веб-страницу. (Вся Индия, 2014 г.)
Ответ:
(i) Тег используется для вставки изображения на веб-страницу.
(ii) Тег
используется для вставки пустой строки на веб-страницу.

Вопрос 3.
Назовите два тега, которые присутствуют в каждом коде HTML. (Вся Индия 2014C)
Ответ:
Два тега, которые присутствуют в каждом HTML-коде:
(i)
(ii)

Вопрос 4.
Напишите имя тега HTML, используемого для включения изображения на веб-страницу HTML. (Дели, 2012 г.)
Ответ:
Для включения изображения на веб-страницу HTML используется тег . Здесь вместо имени пути мы должны указать место на компьютере, где хранится изображение.

Вопрос 5.
Напишите HTML-код для следующего:
Для предоставления гиперссылки на веб-сайт (HOTS; Дели 2012)

 http://WWW.w3schoo1.com 

Ответ:
Для предоставления гиперссылки на веб-сайт http: // www.w3school.com, мы должны написать следующее заявление:

 
Нажмите здесь  

Вопрос 6.
Напишите имя тега HTML, используемого для включения нумерованного списка на веб-страницу HTML. (Вся Индия, 2012 г.)
Ответ:
Чтобы включить нумерованный список на веб-страницу HTML, мы должны использовать тег

    .

    Вопрос 7.
    Введите HTML-код для следующего:
    Для предоставления гиперссылки на веб-сайт (HOTS; All India 2012)

     http: // www.cbse.nic.in 

    Ответ:
    Чтобы предоставить гиперссылку на сайт http://www.cbse.nic.in, мы должны использовать следующий тег:

     
    Нажмите здесь  

    Вопрос 8.
    Какие теги HTML используются для создания таблицы и добавления строк в документ HTML? (Дели 2011; Вся Индия 2011)
    Ответ:
    Для создания таблицы используется тег

    . Мы можем использовать тег для определения и вставки строки в таблицу, а для указания данных, которые должны быть введены в таблицу, мы используем тег и» используется для определения строки таблицы. Строка состоит либо из заголовков таблицы, либо из ячеек таблицы.Строки таблицы не могут напрямую содержать данные. Вместо этого строки таблицы действуют как контейнер для ячеек «
    .

    Вопрос 9.
    Чем тег

      отличается от тега
        в HTML? (Дели 2011; Вся Индия 2011)
        Ответ:
        Тег
          в HTML используется для определения неупорядоченного или маркированного списка, в котором элементы списка отображаются с использованием маркеров без нумерации. В то время как тег
            используется для определения упорядоченного списка, в котором элементы списка отображаются с использованием чисел в различных числовых форматах.

            Вопрос 10.
            Java пишет HTML-код. Как она может увидеть, как код будет выглядеть как веб-страница? (Дели 2011C)
            Ответ:
            Она должна сохранить файл кода HTML, созданный в текстовом редакторе с расширением.htm или расширение .html. После этого ей нужно открыть файл в веб-браузере, таком как Internet Explorer.

            Вопрос 11.
            Что не так в следующем HTML-коде: (HOTS; Delhi 2011C)

             <тип UL = «a» начало = 3> 

            Ответ:
            Исправленный код:

             

            Вопрос 12.
            Что такое полная форма HTML?
            Ответ:
            HTML означает язык разметки гипертекста.

            Вопрос 13.
            Что такое HTML-тег?
            Ответ:
            Теги имеют простую структуру. Они начинаются с «<» и заканчиваются «>». Между угловой скобкой ’<>” написано имя тега и, возможно, некоторые атрибуты, в зависимости от тега.

            Вопрос 14.
            Как можно использовать картинку в качестве ссылки?
            Ответ:
            Мы можем поместить между и , чтобы пользователь мог щелкнуть изображение, чтобы перейти по ссылке.

            Вопрос 15.
            Какая польза от комментариев на веб-странице HTML?
            Ответ:
            Мы можем помещать в ваш HTML-файл комментарии, которые не будут отображаться на веб-странице.Это позволяет вам объяснить, почему ваш HTML-код определенным образом, любому, кто просматривает ваш исходный HTML-код.

            Вопрос 16.
            Как установить столбец или строку, охватывающую более одной строки или столбца? (HOTS)
            Ответ:
            Иногда нам нужна одна ячейка, чтобы охватить более одного столбца или более одной строки в глубину. В этих случаях мы можем использовать атрибуты colspan и rowspan тега

    .

    Вопрос 17.
    Что такое гипертекстовая ссылка?
    Ответ:
    Гипертекстовая ссылка — это специальный тег, который связывает одну страницу с другой страницей или ресурсом.Если вы щелкнете по ссылке, браузер перейдет к месту назначения данной ссылки.

    Вопрос 18.
    Как выровнять таблицу вправо (или влево)?
    Ответ:
    Вы можете использовать

    для выравнивания таблицы по правому краю. (Используйте align = «left», чтобы выровнять его по левому краю.) Любое содержимое, следующее за закрывающим тегом
    , будет перемещаться по таблице.

    Вопрос 19.
    Как создать ссылку, по которой мне будет отправлено электронное письмо?
    Ответ:
    Воспользуйтесь ссылкой mailto.

    Вопрос 20.
    Как вы можете показать примеры HTML, не интерпретируя их как часть моего документа?
    Ответ:
    В примере HTML сначала замените символ «&» на «& amp;» везде, где он встречается. Затем таким же образом замените символ ‘& lt;’ на ‘<‘, а символ ‘& gt;’ на ‘>’.
    Примечание: При отображении примеров HTML может оказаться целесообразным использовать КОД и / или предварительные элементы.

    2 балла за вопросы

    Вопрос 21.
    Объясните на примере (-ах) разницу между контейнером и пустыми элементами HTML. (Дели, 2014 г.)
    Ответ:
    Различия между контейнером и пустым элементом заключаются в следующем:

    Элемент контейнера Пустой элемент
    Этот тип элементов требует запуска, а также соответствующего закрывающего тега. Для этого типа элементов требуется только начальный тег.
    Эти теги влияют на все, что находится между их начальным и конечным тегами. Пустые элементы просто выполняют свою определенную работу.
     Например, 
      --------  

    ——-

     например 
    ,

    Вопрос 22.
    Укажите два атрибута табличного элемента HTML. (Вся Индия, 2014 г.)
    Ответ:
    Два атрибута элемента таблицы:

    Вопрос 23.
    Напишите две функции XML.(Вся Индия 2014C)
    Ответ:
    Особенности XML

    • XML — это текстовые файлы, которыми можно управлять с помощью любого текстового редактора.
    • XML широко используется в качестве формата для хранения и обработки документов как онлайн, так и офлайн.

    Вопрос 24.
    Различите теги

    в HTML с помощью соответствующего примера. (Дели, 2013 г.)
    Ответ:
    Тег «
    » или заголовков «», которые содержат данные, отображаемые в таблице.

    Вопрос 25.
    Почему cTABLE width = 100% не использует всю ширину браузера? (HOTS)
    Ответ:
    Графические браузеры оставляют узкое поле между краем области отображения и содержимым. Также обратите внимание, что навигатор всегда оставляет место для полосы прокрутки справа, но рисует полосу прокрутки только тогда, когда документ достаточно длинный, чтобы требовать прокрутки.Если документ не требует прокрутки, остается правое «поле», которое нельзя удалить.

    Мы надеемся, что важные вопросы 12-го класса по информатике, глава 9, HTML и XML, помогут вам. Если у вас есть какие-либо вопросы по вопросам, касающимся информатики, класс 12, важные вопросы, глава 9, HTML и XML, оставьте комментарий ниже, и мы свяжемся с вами в ближайшее время.

    OHSU Медицинская информатика по выбору для студентов-медиков

    OHSU по специальности Медицинская информатика по выбору для студентов-медиков

    (4 недели)




    Виртуальный курс от Oregon Health & Научный университет (OHSU) становится доступным в качестве факультатив для студентов-медиков в США.Курс широкий обзор в области информатики и первоначально происходит от вводный курс в нашей биомедицинской Аспирантура по информатике. Этот курс также используется в Предложение OHSU в рамках программы American Medical Программа Ассоциации информатики 10х10 («десять на десять»).

    ВАЖНО : Мы хотели бы, чтобы каждая медицинская школа набор студентов и зачисление самих себя. Другими словами, мы будем сделать курс доступным через нашу систему управления обучением (LMS) в OHSU, но мы попросим каждую школу предоставить нам список студенты будут записаны, и каждый получит доступ к курсу.После курс пройден, мы сообщим школам, прошел курс каждый студент или нет. Мы бы хотели медицинские школы, которые участвуют в предоставлении студентам кредитов (возможно, через какое-то самообучение по выбору). В связи с высокий спрос, мы ограничиваем набор студентов в США. аллопатические или остеопатические медицинские школы.

    Мы реализуем курс в качестве 4-недельного студента-медика. факультатив, который дает 2 кредита в OHSU.В курсе есть около 40 часов лекции, и мы ожидаем, что еще 40 часов потратим на дискуссионные форумы, самооценка с несколькими вариантами ответов для каждого блока, и дополнительные чтения. Курс оценивается как пройденный, и прохождение требует завершения всех 10 единиц и их самооценка за 4 недели курса.

    Мы начали один новый 4-недельный курс каждую неделю с понедельника, 6 апреля 2020 г. На каждый курс мы зачисляем всех студентов на единую раздел и сделайте все содержимое доступным для продолжительность 4 недели.Мы используем дискуссионные форумы встроены в нашу LMS, чтобы ответить на их вопросы и поднять несколько вопросы для обсуждения. По истечении 4 недель каждый курс закончится, и те, кто выполнил всю работу, будут получите проходную оценку, о которой мы сообщим контакту от каждой школы.

    С тех пор, как мы начали и в течение всего курса, начинающегося 11 мая 2020 г., всего 123 студента-медика из 16 медицинских вузов, а также еще 35 студентов-медиков из OHSU поступили в курс.Мы планируем и дальше предлагать новые предложения через конец мая, завершение последнего курса в конце июня.

    ВАЖНО : Мы предпочитаем, чтобы была одна точка контакт для каждой школы, с которой мы общаемся. Чтобы запечатлеть это информации, мы создали онлайн-опрос, который запрашивает контактное лицо (используйте адрес электронной почты университета), ориентировочное количество учеников (изначально до 20 на школу — мы можно будет разместить больше позже) и предпочтительные даты (которые нам может потребоваться изменить баланс нагрузки).После опроса завершено, кто-то из наших сотрудников свяжется со школами для работы подробности. Опрос находится по следующему URL-адресу: https://www.surveymonkey.com/r/ohsu-informatics

    Также обратите внимание, для тех, кто интересуется неполным курсом по информатике у нас есть открытый веб-сайт, на котором некоторых материалов и используется некоторыми школами: http://informatics.health

    Обзор конечно

    Цель этот курс предоставляет подробный обзор биомедицинских и информатика здоровья для студентов-медиков.Это виртуальный курс примерно с 40 часами асинхронных лекций и сопоставимым количество времени, потраченное на тесты для самооценки, онлайн-форум обсуждение и дополнительные чтения. Курс предоставляет актуальные подробности в области информатики, включая электронное здоровье записи, стандарты данных и совместимость, клиническое решение поддержка, аналитика медицинских данных, здоровье населения, пациент вовлечение и телемедицина. Он также описывает и устанавливает контекст для новых технологий, таких как SMART на FHIR, машина обучение и искусственный интеллект, носимые устройства и блокчейн.На курсе также будут обсуждаться вопросы, связанные с Covid-19. пандемия.

    Курс Логистика

    Этот курс адаптация он-лайн Введение в Класс биомедицинской и медицинской информатики в настоящее время преподается в биомедицинском отделении OHSU. программа обучения информатике. Предлагается через 4 недели. блок для студентов-медиков. Курс преподается полностью асинхронный способ, но студенты должны не отставать от курса материалы, чтобы они могли извлечь пользу из интерактивного обсуждения с факультет и другие студенты.В курсе используется следующее обучение модальности:
    • Голосовые лекции через Powerpoint — они читаются с использованием HTML5. (Таким образом, он доступен практически на любой платформе.) Доступ к контенту легко получить при любом типе подключения к Интернет.
    • Интерактивное обсуждение — учащиеся могут участвовать в интерактивном обсуждение важных вопросов на онлайн-форумах.
    • Задания по чтению — В курсе предлагается дополнительное чтение задания (хотя студенты несут ответственность только за содержание лекций).
    • Домашнее задание / викторины — К каждому блоку прилагается 10 вопросов. самооценка с несколькими вариантами ответов, направленная на то, чтобы учащийся применить знания от блока.
    Доступ к курсу через Система управления обучением Sakai (LMS) OHSU. В начале Конечно, OHSU предоставляет каждому студенту логин и пароль. сотрудники дистанционного обучения, которые также обеспечивают техническую поддержку курс. Все онлайн-активности асинхронны, поэтому есть не указано время, когда студент должен быть в сети.Студенты должны пройти все тесты самооценки и принять участие в уроке обсуждения.

    Инструктор

    В инструктор курса — Уильям Херш, доктор медицины. Лучший способ связаться с ним — по электронной почте ([email protected]). Вы можете также найти интересное чтение в его блоге.

    Учебный план


    Следующее В таблице представлена ​​учебная программа с указанием номера раздела и темы, а полный план курса представлен ниже.
    Установка
    Тема
    1
    Обзор поля и Проблемы, мотивирующие это
    2
    Биомедицинские вычисления
    3
    Электронные и личные медицинские карты (EHR, PHR)
    4
    Стандарты и совместимость
    5
    Расширенный уход с помощью EHR
    6
    Наука о данных и аналитика
    7
    Внедрение EHR, безопасность, и оценка
    8
    Поиск информации (поиск)
    9
    Исследования в области информатики
    10
    Другие области информатики

    Показания

    В курсе нет требуется учебник.Есть факультативный учебник (под редакцией преподаватель курса), которые студенты могут захотеть рассмотреть: Hoyt RE, Hersh WR, Eds. (2018). Информатика здравоохранения: Практическое руководство, Седьмое издание , доступно на Lulu.com в бумажном и электронном формате. Задания по чтению из книги не являются обязательными, и нет материал появится в тестах для домашних заданий или заключительном экзамене, который также не охвачены в классе. Но некоторые студенты предпочитают также читайте учебник во время обучения.Соответствующие чтения главы для каждого блока в курсе:

    Unit
    Тема
    Учебник Глава (и)
    1
    Обзор поля и Проблемы, мотивирующие это
    1
    2
    Биомедицинские вычисления 3
    3
    Электронные и личные медицинские карты (EHR, PHR)
    2, 4
    4
    Стандарты и совместимость 5
    5
    Расширенный уход с помощью EHR
    6, 8, 9
    6
    Наука о данных и аналитика 7, 14
    7
    Внедрение EHR, безопасность и оценка 4, 10
    8
    Поиск информации (поиск)
    15
    9
    Исследования в области информатики
    18, 20
    10
    Другие области информатики
    12, 13, 16, 17, 19

    Обучение Цели

    Блок 1 — Обзор Поле и проблемы, его мотивирующие

    1.Определите биомедицинскую и медицинскую информатику, термины, связанные с к нему, и его роль в здравоохранении, здравоохранении, общественном здравоохранении и биомедицинские исследования
    2. Опишите основные исторические события биомедицины и здравоохранения. информатика
    3. Обсудить основные проблемы здравоохранения, мотивирующие использование биомедицинская и медицинская информатика
    4. Сравните и сопоставьте роли разных людей в кадровые ресурсы медицинских информационных технологий
    5. Найдите и получите доступ к основным источникам информации для биомедицинская и медицинская информатика в научной литературе и в Интернете

    Блок 2 — Биомедицинские вычисления

    1.Определить основные принципы биомедицинских вычислений, чтобы иметь возможность информировать об оптимальном выборе оборудования, программного обеспечения и сети связи для данной медицинской или биомедицинской среды.
    2. Опишите основные аспекты разработки программного обеспечения, поскольку они относятся к биомедицинской и медицинской информатике.
    3. Укажите вариант использования биомедицинской информатики и информации о здоровье. задача.

    Блок 3 — Электронные и личные медицинские карты (EHR, PHR)

    1. Перечислите основные категории клинических данных вместе с их содержание и структура.
    2. Определите основные функции электронного здоровья. запись (EHR).
    3. Опишите основные препятствия на пути использования ЭУЗ.
    4. Составьте личную медицинскую карту (PHR) и опишите ее использование, содержание и ценность.

    Раздел 4 — Стандарты и взаимодействие

    1. Объясните важность стандартов и взаимодействия для медицинские и биомедицинские данные и информационные системы
    2. Опишите основные вопросы, связанные со стандартами идентификаторов, включая обсуждение идентификаторов пациентов
    3.Обсудите различные стандарты обмена сообщениями, их явное использование и тип данных, которыми они обмениваются
    4. Обсудите различные системы терминологии, используемые в биомедицине и их происхождение, содержание и ограничения
    5. Опишите роль платформ для взаимодействия, включая SMART на FHIR

    Блок 5 — Расширенное лечение с помощью EHR

    1. Опишите основные угрозы безопасности пациентов и причины медицинская ошибка
    2. Объясните основные принципы качества здравоохранения и объясните, как их можно измерить в EHR
    3.Различайте разные типы поддержки принятия клинических решений. и описать их использование и ограничения в клинической практике.
    4. Обсудить цели обмена медицинской информацией и способы их использования. выполняются
    5. Опишите успехи, проблемы и переходный период Программа рационального использования

    Раздел 6 — Наука о данных и аналитика

    1. Опишите ключевые приложения и ограничения приложений науки о данных и аналитики данных в здравоохранении
    2.Обсудите различные подходы к машинному обучению и их успехи в биомедицине и здравоохранении
    3. Опишите методы и ограничения естественного языка обработка
    4. Определите ключевые принципы доказательной медицины (ДМ)
    5. Составьте клинические вопросы, на которые можно ответить, и критически оценить доказательства, отвечающие на них
    6. Критически оценивать исследования вмешательства и диагностики как а также систематические обзоры этих исследований.
    7. Опишите производство и применение клинической практики. руководящие принципы

    Блок 7 — Внедрение и оценка ЭУЗ

    1.Обсудите анализ рабочего процесса для внедрения EHR
    2. Опишите основные шаги и проблемы при внедрении EHR
    3. Классифицируйте различные типы телемедицины и обсудите их использование
    4. Обсудите основные подходы и проблемы конфиденциальности и безопасность медицинских данных
    5. Обсудите значение основных оценочных исследований EHR

    Блок 8 — Поиск информации (поиск)

    1. Перечислите основные ресурсы биомедицинских и медицинских знаний. в книгах, журналах, электронных базах данных и других источниках.
    2. Опишите основные подходы к индексации. контент, основанный на знаниях.
    3. Применяйте передовые методы поиска в основных биомедицинских и ресурсы знаний о здоровье.
    4. Обсудите основные результаты оценки информационного поиска. исследования.

    Раздел 9 — Исследовательская информатика

    1. Обсудите процесс биомедицинских исследований и предприятие.
    2. Опишите способы, которыми информатика может способствовать развитию клинических и трансляционные исследования.
    3. Определите биоинформатику и связанные с ней области и выделите ее работа из других областей биомедицинской и медицинской информатики.
    4. Обсудите роль генетики и геномики в биологии и лекарство.
    5. Опишите парадигму точной медицины и роль биомедицинская и медицинская информатика в облегчении этого.
    6. Понять проблемы, связанные с геномными данными в EHR и другие информационные системы.

    Раздел 10 — Прочие области информатики

    1.Опишите управление изображениями в клинических условиях, включая использование систем PACS
    2. Обсудите различные способы получения изображений и их получение и использовать в цифровой форме
    4. Опишите способы, которыми биомедицинская информатика позволяет медсестрам практика
    5. Опишите способы, которыми биомедицинская информатика позволяет практика здравоохранения
    6. Определите информатику здоровья потребителей и обсудите методы который использует информатику
    7. Определите здоровье населения и обсудите методы, с помощью которых он использует информатику

    Подробный курс Наброски

    1.Обзор Область и проблемы, ее мотивирующие
    1.1 Что такое биомедицинская и медицинская информатика?
    1.2 Краткая история биомедицинской и медицинской информатики
    1.3 Проблемы здравоохранения, побуждающие к биомедицине и здоровью Информатика
    1.4 Кто занимается биомедицинской и медицинской информатикой?
    1.5 Ресурсы для полевых организаций, информации, образования

    2. Биомедицинские вычисления
    2.1 Типы компьютеров
    2.2 Хранение данных в компьютерах
    2.3 Компьютерное оборудование и программное обеспечение
    2.4 Компьютерные сети
    2.5 Разработка программного обеспечения

    3. Электронные и личные медицинские записи (EHR, PHR)
    3.1 Клинические данные
    3.2 История и перспективы медицинской (медицинской) записи
    3.3 Определения и ключевые атрибуты EHR
    3.4 Преимущества и Проблемы, связанные с EHR
    3.5 Примеры EHR
    3.6 Личные медицинские записи

    4. Стандарты и функциональная совместимость
    4.1 Стандарты и функциональная совместимость: основные концепции
    4.2 Стандарты идентификаторов и транзакций
    4.3 Стандарты обмена сообщениями
    4.4 Стандарты терминологии
    4.5 SMART на FHIR

    5. Расширенное лечение с помощью EHR
    5.1 Безопасность пациентов и медицинские ошибки
    5.2 Поддержка принятия клинических решений (CDS)
    5.3 Измерение и улучшение качества здравоохранения
    5.4 Обмен медицинской информацией ( HIE)
    5.5 От конструктивного использования к повышению функциональной совместимости

    6. Наука о данных и аналитика
    6.1 Наука о данных и аналитика данных
    6.2 Машинное обучение и искусственный интеллект
    6.3 Обработка естественного языка
    6.4 Доказательная медицина
    6.5 Руководство по клинической практике

    7. Внедрение, безопасность и оценка ЭУЗ
    7.1 Анализ и перепроектирование клинического рабочего процесса
    7.2 Выбор и внедрение системы ЭУЗ
    7.3 Телемедицина и телездравоохранение
    7.4 Конфиденциальность и безопасность
    7.5 Оценка EHR

    8. Получение информации (поиск)
    8.1 Поиск информации
    8.2 Информация, основанная на знаниях
    8.3 Содержание
    8.4 Индексирование
    8.5 Извлечение
    8.6 Исследования: оценка и будущие направления

    9. Информатика исследований
    9.1 Информатика клинических исследований
    9.2 Обзор фундаментальной молекулярной биологии
    9.3 Трансляционная биоинформатика
    9.4 От клинической генетики и геномики до точной медицины
    9.5 Геномные данные в EHR и другие информационные системы

    10. Другие области информатики
    10.1 Информатика изображений
    10.2 Информатика медсестер
    10.3 Информатика общественного здравоохранения
    10.4 Информатика здоровья потребителей
    10.5 Здоровье населения

    10×10 с OHSU: Описание курса

    Обзор

    Обзор

    Департамент медицинской информатики и клинической эпидемиологии (DMICE) Орегонского университета здравоохранения и науки предложит еще один курс дистанционного обучения по биомедицинской информатике в рамках программы AMIA 10×10.

    Цель этого курса — предоставить подробный обзор биомедицинской и медицинской информатики тем, кто будет работать на стыке здравоохранения и информационных технологий (ИТ).Курс также направлен на то, чтобы предоставить отправную точку для тех, кто желает продолжить обучение (и / или карьерный рост) в этой области. Несмотря на то, что курс имеет клиническую направленность, многие врачи, не являющиеся клиницистами, работающие в ИТ-среде здравоохранения, сочли курс доступным, а полученные знания бесценными для их профессионального развития.

    Курс обеспечивает широкое понимание данной области с точки зрения тех, кто внедряет, возглавляет и разрабатывает ИТ-решения для улучшения здоровья, здравоохранения, общественного здравоохранения и биомедицинских исследований.Он предоставляет обновленную информацию о текущих событиях в этой области, включая электронные медицинские карты, стандарты данных и функциональную совместимость, поддержку клинических решений, аналитику данных, искусственный интеллект, здоровье населения, взаимодействие с пациентами и телемедицину. Он также описывает и устанавливает контекст для новых технологий, таких как SMART на FHIR, машинное обучение, носимые устройства и блокчейн. Кроме того, представлена ​​роль информатики в развивающейся пандемии Covid-19.

    Учебный план / план курса

    Учебный план

    В следующей таблице представлена ​​учебная программа с указанием номера блока, темы, даты публикации и срока сдачи.Курс обычно состоит из двух недель подряд размещаемых материалов, а затем третьей недели для завершения работы. Срок для каждой единицы наступает при разноске следующего цикла материала. Мы снисходительно относимся к продлениям, но участникам настоятельно рекомендуется не отставать, так как если кто-то слишком сильно отстает, наверстать упущенное.

    Часть Тема Дата публикации Срок оплаты
    1 Обзор поля и проблем, которые его мотивируют 7/4/21 21/4/21
    2 Биомедицинские вычисления 14.04.21 21.04.21
    3 Электронные и личные медицинские карты (EHR, PHR) 28.04.21 12.05.21
    4 Стандарты и совместимость 5/5/21 12.05.21
    5 Расширенный уход с EHR 19.05.21 02.06.21
    6

    Наука о данных и аналитика

    26.05.21 02.06.21
    7 Внедрение, безопасность и оценка EHR 09.06.21 23.06.21
    8 Поиск информации (поиск) 16.06.21 23.06.21
    9 Исследования в области информатики 30.06.21 14.07.21
    10 Другие области информатики 14.07.21 21.07.21

    Подробное описание курса

    1.Обзор области и проблем, которые ее мотивируют
    1.1 Что такое биомедицинская и медицинская информатика?
    1.2 Краткая история биомедицинской и медицинской информатики
    1.3 Проблемы здравоохранения, мотивирующие биомедицинскую и медицинскую информатику
    1.4 Кто занимается биомедицинской и медицинской информатикой?
    1.5 Ресурсы для полевых работ — организации, информация, образование

    2. Биомедицинские вычисления
    2.1 Типы компьютеров
    2.2 Хранение данных в компьютерах
    2.3 Компьютерное оборудование и программное обеспечение
    2.4 Компьютерные сети
    2.5 Разработка программного обеспечения

    3. Электронные и личные медицинские записи (EHR, PHR)
    3.1 Клинические данные
    3.2 История и перспективы медицинской записи
    3.3 Определения и ключевые атрибуты EHR
    3.4 Преимущества и Проблемы с EHR
    3.5 Примеры EHR
    3.6 Личные медицинские записи

    4. Стандарты и взаимодействие
    4.1 Стандарты и взаимодействие: основные концепции
    4.2 Стандарты идентификаторов и транзакций
    4.3 Стандарты обмена сообщениями
    4.4 Стандарты терминологии
    4.5 SMART на FHIR

    5. Расширенное лечение с помощью EHR
    5.1 Безопасность пациентов и медицинские ошибки
    5.2 Поддержка принятия клинических решений (CDS)
    5.3 Измерение и улучшение качества здравоохранения
    5.4 Обмен медицинской информацией ( HIE)
    5.5 От эффективного использования к повышению функциональной совместимости

    6. Наука о данных и аналитика
    6.1 Наука о данных и аналитика данных
    6.2 Машинное обучение и искусственный интеллект
    6.3 Обработка естественного языка
    6.4 Доказательная медицина
    6.5 Руководство по клинической практике

    7. Внедрение, безопасность и оценка ЭУЗ
    7.1 Анализ и перепроектирование клинического рабочего процесса
    7.2 Выбор и внедрение системы ЭУЗ
    7.3 Телемедицина и телемедицина
    7.4 Конфиденциальность и безопасность
    7.5 Оценка EHR

    8. Поиск информации (поиск)
    8.1 Поиск информации
    8.2 Информация, основанная на знаниях
    8.3 Контент
    8.4 Индексирование
    8.5 Извлечение
    8.6 Исследования: оценка и будущие направления

    9. Исследовательская информатика
    9.1 Обзор биомедицинских исследований
    9.2 Информация о клинических исследованиях
    9.3 Обзор фундаментальной молекулярной биологии
    9.4 Трансляционная биоинформатика
    9.5 От клинической генетики и геномики к точной медицине
    9.6 Геномические данные в EHR и других информационных системах

    10. Другие области информатики
    10.1 Информатика изображений
    10.2 Медсестринская информатика
    10.3 Информатика здоровья потребителей
    10.4 Информатика общественного здравоохранения
    10.5 Здоровье населения

    Логистика

    Логистика

    Курс состоит из двух частей:

    1. Веб-компонент, состоящий из 10 единиц, начиная с 7 апреля 2021 г. . Веб-часть предоставляется в виде чтения, онлайн-лекций, интерактивных обсуждений и тестов для самооценки. (Тесты должны быть заполнены, но оценки не учитываются в итоговой оценке.)

    2. Интенсивное однодневное очное заседание, проводимое одновременно со встречей AMIA (см. Ниже). Личная сессия соберет участников вместе для интеграции материала, позволит представить курсовые проекты и встретится с преподавателем, а также с другими студентами лично.

    Крайний срок регистрации на курс — 7 апреля 2021 г. Мы будем принимать участников после этой даты при наличии свободных мест.

    Этот курс является адаптацией онлайн-курса Введение в биомедицинскую информатику и информатику здравоохранения , который в настоящее время преподается в рамках образовательной программы OHSU по биомедицине и информатике здоровья.Этот обзорный курс дает широкий обзор области, выделяя ключевые вопросы и проблемы для этой области. Курс преподается полностью асинхронно, т.е. нет никаких «запланированных» занятий. Однако студенты должны не отставать от материалов курса, чтобы они могли извлечь пользу из интерактивного обсуждения с преподавателями и другими студентами. В курсе используются следующие формы обучения:

    • • Лекции с озвучкой в ​​PowerPoint — основной материал доставляется с использованием Flash, HTML 5 или специального проигрывателя iPad.Таким образом, к контенту легко получить доступ при любом типе подключения к Интернету.

    • • Интерактивное многопоточное обсуждение — учащиеся участвуют в обсуждении важных вопросов, используя сетевые дискуссионные форумы. Он-лайн модератор факультета помогает поддерживать обсуждение в нужном русле.

    • • Задания по чтению — в курсе используются различные виды чтения, доступные студентам.

    • • Домашнее задание / контрольные вопросы — Каждый из блоков сопровождается самооценкой из 10 вопросов с несколькими вариантами ответов, цель которой состоит в том, чтобы ученик применил знания из блока.

    Доступ к онлайн-части курса осуществляется через систему управления обучением Sakai (LMS) OHSU. В начале курса каждый студент получает логин и пароль от сотрудников дистанционного обучения OHSU, которые также обеспечивают техническую поддержку курса. В курсе нет необходимого учебника; со всеми назначенными показаниями либо в свободном доступе в режиме онлайн, либо предоставляемыми OHSU. Ожидается, что студенты будут в курсе материалов и потратят на курс 4-8 часов на каждый блок.Все онлайн-мероприятия асинхронны, поэтому нет определенного времени, когда студент должен быть онлайн.

    Студенты должны пройти все тесты для самооценки, проект курса и участвовать в обсуждениях в классе, чтобы получить сертификат AMIA 10×10 об окончании. Врачи имеют право на получение до 46,5 часов AMA PRA Category I CME Credit (s) . ™ Поскольку курс является продолжением обучения, в нем не используются академические буквенные оценки (например, A, B и т. Д.). Тем не менее, тем, кто хочет получить академический балл после сдачи необязательного заключительного экзамена (см. Ниже), будет присвоена буквенная оценка на основе их баллов на экзамене.

    При возникновении проблем очень важно связаться с соответствующим лицом :

    • • По основным проблемам Sakai (невозможно войти в систему, что-то, по-видимому, не работает) и вопросам курса (например, блок или дискуссионный форум не опубликованы, когда это должно быть), обратитесь в службу поддержки Sakai по телефону 877-972-5249 или sakai @ ohsu .edu. Служба поддержки Sakai работает с 8:00 до 22:00. Тихий океан, пн-пт и 12-17.00 Тихий океан по выходным.

    • • По вопросам о содержании курса (напр.g., не понимаете тему или не согласны с ответом на домашнее задание), обратитесь к Ассистенту преподавателя (TA), о котором будет объявлено в начале курса.

    При необходимости все вопросы будут переданы доктору Хершу. Хотя доктор Херш не поддерживает запланированные рабочие часы, с ним легко связаться по электронной почте, и он ответит в течение 24-48 часов. Встречи для обсуждения вопросов курса по телефону или лично могут быть назначены по электронной почте.

    Курсовой проект

    Студенты должны выполнить курсовой проект, чтобы получить сертификат об окончании AMIA 10×10.Цель проекта — выявить проблему информатики в вашей местности (например, там, где вы занимаетесь или работаете или иным образом имеете доступ) и предложить решение, основанное на том, что известно из исследований в области информатики и передовой практики. Отчет о проекте должен быть оформлен до 14 июля 2021 г. (Если у вас нет доступа к медицинскому учреждению, вы можете выполнить проект в другом месте, например, в компании или организации. Инструктор может помочь, если у вас есть Проблема и решение должны быть записаны в краткие 2–3 страницы (пожалуйста, больше не надо!), которые должны включать ссылки, которые оправдывают формулировку проблемы и предлагаемые решения.Он представлен в документе Word, загруженном в Sakai.

    Студенты представят свой проект своим коллегам на очном занятии, которое они посещают. В комнате очного занятия есть круглые столы, и студенты разбиваются на небольшие группы вокруг столов. Каждая группа выбирает одного человека, чтобы представить обзор обсуждения в группе. Остальные участники группы выступают в качестве участников дискуссии на короткой (10-15-минутной) панельной презентации на сессии.

    Цели обучения

    Цели обучения

    После участия в этом упражнении учащийся должен уметь:

    • • Опишите биомедицинскую информатику и ее роль в здравоохранении, здравоохранении, общественном здравоохранении и биомедицинских исследованиях.

    • • Сравните и сопоставьте роли разных людей в кадрах медицинских информационных технологий.

    • • Определите основные принципы биомедицинских вычислений, чтобы обеспечить оптимальный выбор оборудования, программного обеспечения и сетевых подключений для заданных условий.

    • • Идентификация основных функций электронной медицинской карты (EHR) и личной медицинской карты (PHR), а также препятствий для их использования.

    • • Объясните важность стандартов и функциональной совместимости клинических данных, а также основные инициативы, предпринимаемые для их реализации.

    • • Опишите принципы безопасности пациентов и то, как информационные технологии здравоохранения способствуют и усугубляют ее.

    • • Объясните основные принципы оценки качества медицинской помощи, включая оплату программ производительности, и то, как ЭУЗ позволяет их реализовать.

    • • Различайте различные типы поддержки принятия клинических решений и их ограничения в клинической практике.

    • • Объясните процесс компьютеризированного ввода заказов поставщика и проблемы, связанные с его использованием.

    • • Обсудите обмен медицинской информацией, его посредников и препятствия.

    • • Опишите программу значимого использования стимулов ЭУЗ в США.

    • • Обсудите внедрение EHR, включая анализ рабочего процесса и перепроектирование.

    • • Опишите результаты оценочных исследований по оценке внедрения и результатов ЭУЗ.

    • • Обсудите применение информатики в сестринском деле, общественном здравоохранении и других конкретных аспектах системы здравоохранения.

    • • Определите разницу между конфиденциальностью и безопасностью, а также их роль в правилах HIPAA.

    • • Продемонстрировать способность применять доказательную медицину и критическую оценку к клиническим вопросам.

    • • Обсудите базовое применение аналитики данных здравоохранения.

    • • Понимать основные медицинские ресурсы и уметь их эффективно искать.

    • • Опишите управление изображениями в клинических условиях, включая использование систем PACS.

    • • Классифицируйте различные типы телемедицины и телездравоохранения и их эффективность, как показано в клинических исследованиях.

    • • Обсудите роль биомедицинской и медицинской информатики в клинических и трансляционных исследованиях.

    • • Опишите ключевые проблемы и результаты трансляционной биоинформатики.
    Материалы курса

    В курсе нет учебника.

    Показаний

    Студентам предоставляется задание для чтения 1-3 ключевых статей или отчетов по каждому блоку.Студентам также предоставляются исчерпывающие списки ссылок по темам, затронутым в лекциях.

    Существует дополнительный учебник (под редакцией преподавателя курса), который студенты могут захотеть рассмотреть: Hoyt RE, Hersh WR, Eds. (2018). Health Informatics: Practical Guide, Seventh Edition , который доступен на Lulu.com в электронной и бумажной версиях. Задания по чтению из книги не являются обязательными, и в тестах для домашних заданий или заключительном экзамене не будет отображаться никаких материалов, которые также не рассматриваются в классе.Но некоторые студенты предпочитают также читать учебник во время обучения. Соответствующие показания главы для каждого раздела в курсе следующие:

    Установка Тема Учебник Раздел (ы)
    1 Обзор поля и проблем, которые его мотивируют 1
    2 Биомедицинские вычисления 3
    3 Электронные и личные медицинские карты (EHR, PHR) 2, 4
    4 Стандарты и совместимость 5
    5 Расширенный уход с EHR 6, 8, 9
    6 Наука о данных и аналитика 7, 14
    7 Внедрение, безопасность и оценка EHR 4, 10
    8 Поиск информации (поиск) 15
    9 Исследования в области информатики 18, 20
    10 Другие области информатики 12, 13, 16, 17, 19
    Дополнительная информация

    Дополнительный заключительный экзамен и получение кредита OHSU

    Курс 10×10 не имеет заключительного экзамена, и те, кто завершит все работы по онлайн-курсу, получат сертификат AMIA 10×10 об окончании.В конце курса для тех, кто имеет на это право и желает получить зачетные баллы за курс от OHSU, сдается дополнительный выпускной экзамен. Экзамен представляет собой заключительный экзамен с открытой книгой, который длится в течение одной недели. Кредит обычно запрашивают те, кто желает продолжить обучение в области биомедицины и информатики здоровья, или те, кому требуется академическая справка для возмещения стоимости обучения. Дополнительная информация о выпускном экзамене и о том, как поступить в OHSU для получения академического кредита, предоставляется после начала курса.

    Те, кто запрашивает компенсацию за обучение у работодателей или других лиц, должны проверить условия и сроки выплаты компенсации. Некоторые работодатели требуют от OHSU официальную стенограмму с итоговой оценкой, прежде чем возмещать плату за обучение. Транскрипт и зачетные баллы не доступны до конца академического семестра, следующего за завершением курса 10х10.

    За пределами 10×10

    Цель программы AMIA 10×10 — обучить врачей и других специалистов информатике, чтобы они могли стать хорошо осведомленными участниками внедрения ИТ в своих местных условиях.Сама по себе программа 10×10 не сделает человека профессиональным специалистом в области информатики (точно так же, как семестр медицины или медсестры сделает человека врачом или медсестрой!). Тем не менее, программа структурирована таким образом, чтобы позволить тем, кто закончил курс, перенести зачетные единицы в другие программы аспирантуры по информатике. Детали должны быть согласованы с каждой индивидуальной программой.

    Поскольку этот курс представляет собой адаптацию вводного курса образовательной программы OHSU по биомедицине и информатике здоровья, те, кто завершит курс 10×10, смогут получить кредит за курс в программе OHSU.Этот кредит берется путем сдачи факультативного заключительного экзамена в конце курса 10×10. При зачислении на программу получения аттестата выпускника OHSU или программу магистратуры, студенты, сдавшие заключительный экзамен, получат три кредита в программе магистратуры OHSU. (OHSU работает по системе академических кварталов, где каждый квартал состоит из 11 недель обучения. Курс с тремя кредитами сопоставим с курсом с тремя контактными часами в неделю плюс дополнительная работа для чтения заданий, домашних заданий и проектов.) Большинство курсов информатики OHSU преподаются на территории кампуса и в Интернете, и каждый курс считается равнозначным, независимо от того, преподается он вживую или дистанционно.

    Более подробная информация об отдельных программах на получение степени доступна на веб-сайте Департамента медицинской информатики и клинической эпидемиологии OHSU, но в следующей таблице представлен обзор программ.

    Название программы Описание Требования к поступающим Требования к выпускным
    Свидетельство о высшем образовании в области биомедицинской информатики Базовые курсы по информатике Степень бакалавра по специальности 24 кредита (обычно 8 курсов по 3 кредита)
    Магистр биомедицинской информатики Магистратура «Профессионал» с завершающим проектом Степень бакалавра в любой области плюс вводные курсы по информатике и анатомии и физиологии 52 кредита (46 часов обучения плюс 6 часов проекта Capstone)
    Магистр биомедицинской информатики «Исследовательская» степень магистра с кандидатской диссертацией Степень бакалавра в любой области плюс вводные курсы по информатике и анатомии и физиологии 60 кредитов (48 часов обучения плюс 12 часов магистерской работы)
    Доктор философии (PhD) по биомедицинской информатике Программа PhD для передовых лидеров и исследований в этой области Степень бакалавра в любой области плюс вводные курсы по информатике и анатомии и физиологии 135 кредитов, в том числе диссертация

    На веб-сайте также есть информация о различных программах стипендий OHSU, финансируемых Национальной медицинской библиотекой США и другими организациями.

    Американская ассоциация медицинской информатики аккредитована Советом по аккредитации непрерывного медицинского образования для обеспечения непрерывного медицинского образования врачей.

    Американская ассоциация медицинской информатики присваивает этому долговечному материалу максимум 46,5 AMA PRA Category 1 Credit (s) ™ . Врачи должны претендовать только на заслугу, соизмеримую со степенью их участия в этой деятельности.

    Завершение этого прочного материала демонстрируется участием во всех онлайн-сессиях, завершением проекта Capstone, завершением опроса участников, посещение очного занятия для участников класса с преподавателями не является обязательным.

    Стремясь поддерживать принципы и практику академической честности, преподаватели OHSU могут использовать системы проверки оригинальности, такие как Turnitin , для сравнения представленных студентами работ с несколькими источниками. Чтобы защитить конфиденциальность учащихся в этом процессе, учащимся необходимо удалить всю личную информацию, то есть имя учащегося, адрес электронной почты, u-номер учащегося или любую другую личную информацию, из своих документов ДО подачи .

    Никакой коммерческой поддержки по этой деятельности получено не было.

    Как провайдер, аккредитованный ACCME, AMIA требует, чтобы каждый, кто имеет возможность контролировать содержание образовательной деятельности, раскрыл все соответствующие финансовые отношения с любыми коммерческими интересами за 12 месяцев до образовательной деятельности.

    ACCME рассматривает отношения лица, участвующего в деятельности CME, включая финансовые отношения супруга или партнера.

    Преподаватели и специалисты по планированию, которые отказываются раскрывать соответствующие финансовые отношения, будут лишены права участвовать в мероприятиях CME.Для лица, не имеющего соответствующих финансовых отношений, участники должны быть проинформированы об отсутствии конфликта интересов или финансовых отношений.

    AMIA использует ряд методов для разрешения потенциальных конфликтов интересов, в том числе: ограничение содержания презентации тем, что было проверено одним или несколькими рецензентами; обеспечение соответствия всех упомянутых научных исследований общепринятым стандартам экспериментального дизайна, сбора и анализа данных; проведение обзора образовательной деятельности рецензентом содержания для оценки потенциальной предвзятости, сбалансированности изложения, доказательного содержания или других показателей добросовестности и отсутствия предвзятости; мониторинг образовательной деятельности на предмет коммерческой предвзятости в презентации; и / или изучение отзывов участников для оценки коммерческой предвзятости в их деятельности.

    OHSU стремится обеспечить равный доступ для квалифицированных студентов, которые имеют инвалидность, в соответствии с разделом 504 Закона о реабилитации 1973 года, Закона об американцах с ограниченными возможностями (ADA) 1990 года и Закона о поправках ADA (ADA-AA) 2008. Если у вас есть инвалидность или вы считаете, что у вас может быть инвалидность (физическая, сенсорная, хроническая, психологическая или обучающаяся), пожалуйста, свяжитесь с Офисом доступа студентов по телефону (503) 494-0082 или [email protected], чтобы обсудить право на участие. для академических помещений.Информация также доступна на сайте www.ohsu.edu/student-access. Поскольку для внедрения приспособлений может потребоваться время и они не могут быть применены задним числом, важно провести это обсуждение как можно скорее. Вся информация об инвалидности студента хранится в соответствии с соответствующими законами штата и федеральными законами.

    Телефон: (503) 494-0082
    Электронная почта: [email protected]
    Веб-сайт: www.ohsu.edu/student-access

    Уильям Херш, доктор медицины, FACMI, FACP, сообщает, что ни он, ни его супруга не имеют отношения к делу. финансовые отношения с коммерческими интересами.

    Возможности информатики для вовлечения пациентов в безопасность больниц: концептуальная модель | Журнал Американской ассоциации медицинской информатики

    Аннотация

    Цель

    Стационарные пациенты могут сыграть важную роль в выявлении, предотвращении и сообщении о проблемах, связанных с качеством и безопасностью их лечения. Чтобы эффективно поддерживать их в этой роли, решения в области информатики должны соответствовать их опыту. Таким образом, мы намеревались понять, как стационарные пациенты переживают нежелательные события (НС), и выявить возможности для этих информационных решений.

    Материалы и методы

    Мы провели опрос 242 пациентов и лиц, осуществляющих уход, во время их пребывания в больнице, задав открытые вопросы об их опыте работы с UE. На основе нашего качественного анализа мы разработали концептуальную модель, отражающую их опыт, и определили возможности информатики для поддержки пациентов.

    Результаты

    Наша четырехэтапная концептуальная модель иллюстрирует стационарный опыт, с момента, когда они впервые сталкиваются с UE, когда они могут вмешаться, когда возникает вред, какие типы вреда они испытывают и что они делают в ответ на вред.

    Обсуждение

    Существующие решения в области информатики обращаются к первому этапу опыта стационарных пациентов, повышая их осведомленность о потенциальных UE. Однако будущие исследователи могут изучить новые возможности для заполнения пробелов в поддержке, которые пациенты получают на последующих этапах, особенно в критических моментах принятия решений, таких как вмешательство в UE и реагирование на причиненный вред.

    Выводы

    Наша концептуальная модель раскрывает сложный стационарный опыт работы с UE и возможности для новых информационных решений для поддержки их на всех этапах их опыта.Изучение этих новых возможностей могло бы способствовать участию стационарных пациентов и их вовлечению в обеспечение качества и безопасности их лечения, помогло бы системам здравоохранения учиться на опыте стационарных пациентов и уменьшить количество этих пагубных событий.

    ВВЕДЕНИЕ

    На протяжении двух десятилетий безопасность пациентов была признана важнейшей проблемой в системах здравоохранения во всем мире. Только в Соединенных Штатах врачебные ошибки являются третьей по значимости причиной смерти, ежегодно унося более 200 000 жертв. 1 В качестве основных свидетелей получаемой ими помощи пациенты имеют разные точки зрения от поставщиков медицинских услуг на проблемы, возникающие во время их пребывания в больнице, и могут играть значительную роль в выявлении, предотвращении и сообщении об этих проблемах. 2–5 Действительно, пациенты часто первыми обнаруживают недостатки в своей безопасности и успешно вмешиваются в проблемы. 6–8 Несмотря на то, что пациенты играют важную роль в повышении безопасности больниц, понимание пациентом всего спектра проблем качества и безопасности, далее именуемых нежелательными событиями (НБ), недостаточно изучено, поскольку они часто не регистрируются, не признаются и не принимаются во внимание. недостаточно поддерживаются системами информатики. 9 , 10

    Исследователи призвали к решениям, которые поощряют участие пациентов в безопасности. 11 Однако большинство решений, реализуемых в больницах, не предназначены для преодоления препятствий, с которыми пациенты сталкиваются, когда они высказываются, 12 , 13 и не затрагивают широкий спектр UE, с которыми сталкиваются пациенты. 14 , 15 Исследования в области информатики здравоохранения привели к появлению нескольких стационарных технологий, в том числе информационных дисплеев и порталов для пациентов, чтобы пациенты могли оказывать им помощь. 16 Тем не менее, лишь немногие стационарные технологии ставят на первое место участие пациентов в их безопасности. Следовательно, существует значительный разрыв между потенциалом пациентов активно участвовать в повышении своей безопасности и вмешательствами, которые им доступны в настоящее время.

    Понимание того, как пациенты сталкиваются с проблемами качества и безопасности в больнице, является важным шагом на пути к разработке информационных решений, которые пациенты могут использовать для выявления и предотвращения таких проблем. В этой статье мы представляем концептуальную модель, которая представляет опыт пациентов, начиная с момента, когда они впервые сталкиваются с UE, до того, какие действия они предпринимают в ответ на вред.Наша модель основана на результатах опроса, проведенного нами с участием 242 взрослых и педиатрических пациентов, а также лиц, осуществляющих уход за ними. Эта модель открывает новые возможности для информационных решений для поддержки участия пациентов в повышении качества и безопасности их больничной помощи.

    МАТЕРИАЛЫ И МЕТОДЫ

    Мы провели исследование, чтобы понять опыт стационарных пациентов и лиц, осуществляющих уход, с UE, которые мы определяем как удовлетворяющие следующим трем критериям: (1) небольшая или серьезная проблема, (2) что-то неприятное или причиняющее вред, и 3) чего-то, чего можно было бы избежать с точки зрения пациента и опекуна.Нашей целью было уловить точки зрения пациентов и лиц, осуществляющих уход, а не поставщиков услуг. Поэтому мы не сравнивали наши данные с медицинскими записями, официальными отчетами или отчетами поставщиков о событиях. Этическое одобрение было получено от наших институциональных наблюдательных советов.

    Инструмент для обследования

    Наш веб-инструмент анонимного опроса состоял из 30 закрытых и открытых вопросов о UE участника, общем опыте работы в больнице и демографических данных.Мы включили 3 вопроса с произвольным текстом, в которых участникам задавались вопросы о (1) событии их собственными словами, (2) о том, что вызвало событие с их точки зрения, и (3) о результате мероприятия. Наша статья посвящена ответам на эти открытые вопросы, чтобы понять точки зрения участников. Участники могли описать события, которые произошли во время любого предыдущего пребывания в больнице, не обязательно в месте исследования. Инструмент прошел пилотные испытания, чтобы обеспечить читаемость для людей разного возраста и уровня образования.

    Набор персонала и процедуры

    Чтобы охватить различные точки зрения, мы набрали участников в педиатрическую больницу и больницу для взрослых в городском районе США.Мы получили удобную выборку медицинских и хирургических услуг. К пациентам и лицам, осуществляющим уход, обращались, если они соответствовали следующим критериям: если они провели в больнице хотя бы 1 ночь во время их текущего пребывания, были ли им исполнилось 7 лет, могли ли они общаться на английском языке и были ли они достаточно хорошо себя чувствовали, чтобы дать информированное согласие. Мы спросили их, испытывали ли они UE, основываясь на нашем определении. Для тех, кто хотел уточнить это определение, мы предоставили примеры (например, разногласия с поставщиком услуг, неправильное место проведения хирургической операции или что-то среднее между ними).Пациенты и лица, осуществляющие уход, которые испытали UE, согласились на участие и предоставили информированное согласие, прошли опрос на iPad. Члены исследовательской группы ответили на вопросы и при необходимости помогли участнику пройти опрос.

    Анализ данных

    Мы провели многоэтапный качественный анализ открытых вопросов нашего опроса. 17 Открытое кодирование использовалось для разработки предварительных кодовых книг для типов UE, вреда и других ключевых аспектов опыта участников.Во время набора 2 кодировщика встретились с исследовательской группой для обсуждения и итеративного редактирования кодовых книг. Этот анализ продолжался до тех пор, пока не было достигнуто насыщение и не завершился набор. Валидность проверялась путем дедуктивного присвоения кодов случайной выборке из 20% ответов. После пересмотра кодовых книг были закодированы еще 10 случайно выбранных ответов, была достигнута межэкспертная надежность (каппа Коэна = 0,815, процентное соответствие = 96,3%), и весь набор данных был перекодирован с использованием исправленных кодовых книг.Затем мы определили паттерны совпадающих кодов и разработали концептуальную модель, характеризующую отношения между ними.

    Участники

    Мы обратились к 606 госпитализированным пациентам и лицам, осуществляющим уход, в наших детских и взрослых исследовательских центрах. Из них 312 (51,5%) были признаны подходящими для участия, 70 из которых отказались. Остальные 242 пациента и лица, осуществляющие уход, согласились участвовать и заполнили опрос (процент ответивших 77,6%). Мы получили в общей сложности 246 ответов, так как некоторые участники сообщили о нескольких различных UE.В таблице 1 приведены демографические данные наших участников.

    Таблица 1.

    Обзор демографических данных участников

    8321 ) 902,4 / Афроамериканец 902 d или менее
    . Педиатрический участок . Сайт для взрослых . Итого .
    Заполненные ответы 146 (59,3) 100 (40,7) 246 (100)
    Пациенты 56 (22.8) a 79 (32,1) 135 (54,9)
    Помощники по уходу 90 (36,6) 21 (8,5) 111 (45,1)
    902

    Пол

    Внутренняя часть 108 (43,9) 53 (21,6) 161 (65,5)
    Наружная часть 37 (15,0)
    Другое / нет ответа 1 (0.4) 1 (0,4) 2 (0,8)
    Возраст
    7–18 лет 48 (19,5) 902 48 (0,0) 902 (19,5)
    19–24 яркости 12 (4,9) 2 (0,8) 14 (5,7)
    25–44 яркости 67 (27,2) 17 (6,9) 84 (34.1)
    45–64 y 16 (6.5) 45 (18,3) 61 (24,8)
    65+ y 1 (0,4) 36 (14,6) 37 (15,0)
    Нет ответа 2 (0,8 ) 0 (0,0) 2 (0,8)
    Образование
    8 класс 17 (6,9) 0 (0,0) 17 614 902,9
    Средняя школа / GED 25 (10.2) 19 (7,7) 44 ​​(17,9)
    Некоторые колледжи 18 (7,3) 19 (7,7) 37 (15,0)
    2- или 4-летняя степень 44 ​​(17,9) 36 (14,6) 80 (32,5)
    Выпускник / профессиональный 17 (6,9) 24 (9,8) 41 (16,7)
    Нет ответа / нет ответа 25 (10,2) 2 (0,8) 27 (11.0)
    Раса / этническая принадлежность b
    Белый / европеоид 109 (44,3) 79 (32,1) 902 902 8 (3,3) 5 (2,0) 13 (5,3)
    Латиноамериканец 16 (6,5) 5 (2,0) 21 (8,5)
    Азиатский 17 (6.9) 6 (2,4) 23 (9,3)
    Индеец / жители островов Тихого океана 12 (4,9) 4 (1,6) 16 (6,5)
    Другое / нет ответа 17 (6,9) 7 (2,9) 24 (9,8)
    Ночлег в больнице в течение последних 5 лет c
    Менее чем 84 (34.2) 69 (28,0) 153 (62,2)
    Более 10 пребываний 58 (23,6) 30 (12,2) 88 (35,8)
    Нет ответа 4 1,6) 1 (0,4) 5 (2,0)
    Тип пребывания на момент УП
    Планируемый 45 (18,3) 39 ) 84 (34.2)
    Незапланированный 101 (41,1) 61 (24,8) 162 (65,9)
    Срок пребывания на момент УП
    94 (38,2) 69 (28,1) 163 (66,3)
    8 d или более 45 (18,3) 29 (11,8) 74 (30,1)
    Не уверен 7 (2.9) 2 (0,8) 9 (3,7)
    Женский 902,4 / Афроамериканец 902 d или менее
    . Педиатрический участок . Сайт для взрослых . Итого .
    Заполненные ответы 146 (59,3) 100 (40,7) 246 (100)
    Пациенты 56 (22,8) 1) 135 (54,9)
    Опекуны 90 (36,6) 21 (8,5) 111 (45,1)
    Пол 108 (43,9) 53 (21,6) 161 (65,5)
    Мужской 37 (15,0) 46 (18,7) 83 (33,7)
    Другое / нет ответа 1 (0.4) 1 (0,4) 2 (0,8)
    Возраст
    7–18 лет 48 (19,5) 902 48 (0,0) 902 (19,5)
    19–24 яркости 12 (4,9) 2 (0,8) 14 (5,7)
    25–44 яркости 67 (27,2) 17 (6,9) 84 (34.1)
    45–64 y 16 (6.5) 45 (18,3) 61 (24,8)
    65+ y 1 (0,4) 36 (14,6) 37 (15,0)
    Нет ответа 2 (0,8 ) 0 (0,0) 2 (0,8)
    Образование
    8 класс 17 (6,9) 0 (0,0) 17 614 902,9
    Средняя школа / GED 25 (10.2) 19 (7,7) 44 ​​(17,9)
    Некоторые колледжи 18 (7,3) 19 (7,7) 37 (15,0)
    2- или 4-летняя степень 44 ​​(17,9) 36 (14,6) 80 (32,5)
    Выпускник / профессиональный 17 (6,9) 24 (9,8) 41 (16,7)
    Нет ответа / нет ответа 25 (10,2) 2 (0,8) 27 (11.0)
    Раса / этническая принадлежность b
    Белый / европеоид 109 (44,3) 79 (32,1) 902 902 8 (3,3) 5 (2,0) 13 (5,3)
    Латиноамериканец 16 (6,5) 5 (2,0) 21 (8,5)
    Азиатский 17 (6.9) 6 (2,4) 23 (9,3)
    Индеец / жители островов Тихого океана 12 (4,9) 4 (1,6) 16 (6,5)
    Другое / нет ответа 17 (6,9) 7 (2,9) 24 (9,8)
    Ночлег в больнице в течение последних 5 лет c
    Менее чем 84 (34.2) 69 (28,0) 153 (62,2)
    Более 10 пребываний 58 (23,6) 30 (12,2) 88 (35,8)
    Нет ответа 4 1,6) 1 (0,4) 5 (2,0)
    Тип пребывания на момент УП
    Планируемый 45 (18,3) 39 ) 84 (34.2)
    Незапланированный 101 (41,1) 61 (24,8) 162 (65,9)
    Срок пребывания на момент УП
    94 (38,2) 69 (28,1) 163 (66,3)
    8 d или более 45 (18,3) 29 (11,8) 74 (30,1)
    Не уверен 7 (2.9) 2 (0,8) 9 (3,7)
    Таблица 1.

    Обзор демографических данных участников

    8321 ) 902,4 / Афроамериканец 902 d или менее
    . Педиатрический участок . Сайт для взрослых . Итого .
    Заполненные ответы 146 (59,3) 100 (40,7) 246 (100)
    Пациенты 56 (22.8) a 79 (32,1) 135 (54,9)
    Помощники по уходу 90 (36,6) 21 (8,5) 111 (45,1)
    902

    Пол

    Внутренняя часть 108 (43,9) 53 (21,6) 161 (65,5)
    Наружная часть 37 (15,0)
    Другое / нет ответа 1 (0.4) 1 (0,4) 2 (0,8)
    Возраст
    7–18 лет 48 (19,5) 902 48 (0,0) 902 (19,5)
    19–24 яркости 12 (4,9) 2 (0,8) 14 (5,7)
    25–44 яркости 67 (27,2) 17 (6,9) 84 (34.1)
    45–64 y 16 (6.5) 45 (18,3) 61 (24,8)
    65+ y 1 (0,4) 36 (14,6) 37 (15,0)
    Нет ответа 2 (0,8 ) 0 (0,0) 2 (0,8)
    Образование
    8 класс 17 (6,9) 0 (0,0) 17 614 902,9
    Средняя школа / GED 25 (10.2) 19 (7,7) 44 ​​(17,9)
    Некоторые колледжи 18 (7,3) 19 (7,7) 37 (15,0)
    2- или 4-летняя степень 44 ​​(17,9) 36 (14,6) 80 (32,5)
    Выпускник / профессиональный 17 (6,9) 24 (9,8) 41 (16,7)
    Нет ответа / нет ответа 25 (10,2) 2 (0,8) 27 (11.0)
    Раса / этническая принадлежность b
    Белый / европеоид 109 (44,3) 79 (32,1) 902 902 8 (3,3) 5 (2,0) 13 (5,3)
    Латиноамериканец 16 (6,5) 5 (2,0) 21 (8,5)
    Азиатский 17 (6.9) 6 (2,4) 23 (9,3)
    Индеец / жители островов Тихого океана 12 (4,9) 4 (1,6) 16 (6,5)
    Другое / нет ответа 17 (6,9) 7 (2,9) 24 (9,8)
    Ночлег в больнице в течение последних 5 лет c
    Менее чем 84 (34.2) 69 (28,0) 153 (62,2)
    Более 10 пребываний 58 (23,6) 30 (12,2) 88 (35,8)
    Нет ответа 4 1,6) 1 (0,4) 5 (2,0)
    Тип пребывания на момент УП
    Планируемый 45 (18,3) 39 ) 84 (34.2)
    Незапланированный 101 (41,1) 61 (24,8) 162 (65,9)
    Срок пребывания на момент УП
    94 (38,2) 69 (28,1) 163 (66,3)
    8 d или более 45 (18,3) 29 (11,8) 74 (30,1)
    Не уверен 7 (2.9) 2 (0,8) 9 (3,7)
    Женский 902,4 / Афроамериканец 902 d или менее
    . Педиатрический участок . Сайт для взрослых . Итого .
    Заполненные ответы 146 (59,3) 100 (40,7) 246 (100)
    Пациенты 56 (22,8) 1) 135 (54,9)
    Опекуны 90 (36,6) 21 (8,5) 111 (45,1)
    Пол 108 (43,9) 53 (21,6) 161 (65,5)
    Мужской 37 (15,0) 46 (18,7) 83 (33,7)
    Другое / нет ответа 1 (0.4) 1 (0,4) 2 (0,8)
    Возраст
    7–18 лет 48 (19,5) 902 48 (0,0) 902 (19,5)
    19–24 яркости 12 (4,9) 2 (0,8) 14 (5,7)
    25–44 яркости 67 (27,2) 17 (6,9) 84 (34.1)
    45–64 y 16 (6.5) 45 (18,3) 61 (24,8)
    65+ y 1 (0,4) 36 (14,6) 37 (15,0)
    Нет ответа 2 (0,8 ) 0 (0,0) 2 (0,8)
    Образование
    8 класс 17 (6,9) 0 (0,0) 17 614 902,9
    Средняя школа / GED 25 (10.2) 19 (7,7) 44 ​​(17,9)
    Некоторые колледжи 18 (7,3) 19 (7,7) 37 (15,0)
    2- или 4-летняя степень 44 ​​(17,9) 36 (14,6) 80 (32,5)
    Выпускник / профессиональный 17 (6,9) 24 (9,8) 41 (16,7)
    Нет ответа / нет ответа 25 (10,2) 2 (0,8) 27 (11.0)
    Раса / этническая принадлежность b
    Белый / европеоид 109 (44,3) 79 (32,1) 902 902 8 (3,3) 5 (2,0) 13 (5,3)
    Латиноамериканец 16 (6,5) 5 (2,0) 21 (8,5)
    Азиатский 17 (6.9) 6 (2,4) 23 (9,3)
    Индеец / жители островов Тихого океана 12 (4,9) 4 (1,6) 16 (6,5)
    Другое / нет ответа 17 (6,9) 7 (2,9) 24 (9,8)
    Ночлег в больнице в течение последних 5 лет c
    Менее чем 84 (34.2) 69 (28,0) 153 (62,2)
    Более 10 пребываний 58 (23,6) 30 (12,2) 88 (35,8)
    Нет ответа 4 1,6) 1 (0,4) 5 (2,0)
    Тип пребывания на момент УП
    Планируемый 45 (18,3) 39 ) 84 (34.2)
    Незапланированный 101 (41,1) 61 (24,8) 162 (65,9)
    Срок пребывания на момент УП
    94 (38,2) 69 (28,1) 163 (66,3)
    8 d или более 45 (18,3) 29 (11,8) 74 (30,1)
    Не уверен 7 (2.9) 2 (0,8) 9 (3,7)

    РЕЗУЛЬТАТЫ

    Наш анализ выявил 4 различных стадии, которые характеризуют опыт пациентов и лиц, осуществляющих уход, с НЯ и вредами. Основываясь на наших выводах, мы разработали концептуальную модель, которая отображает события, вред, вмешательства и реакции пациентов, а также взаимосвязь между этими элементами (рис. 1). В следующих разделах мы описываем каждый из этих этапов, а также типы UE, вреда и ответных действий.Цитаты и примеры снабжены уникальным идентификатором S #.

    Рисунок 1.

    Наша четырехэтапная концептуальная модель, которая описывает опыт пациента с нежелательными событиями (НС).

    Рисунок 1.

    Наша четырехэтапная концептуальная модель, которая описывает опыт пациента с нежелательными событиями (НС).

    Этап 1: появление UE

    На первом этапе пациенты и лица, осуществляющие уход, признали, что у них возникли 1 или более UE.Эти UE в основном делятся на 2 категории: клинические и доклинические. Клинические UE относятся к проблемам при оказании помощи пациенту в больнице и включают использование сломанного медицинского оборудования, неправильные диагнозы и неожиданные отклонения от согласованных протоколов лечения. С другой стороны, доклинические события влияют на качество пребывания пациента в больнице, что может негативно повлиять на клиническую помощь. Доклинические события состояли из 3 подтипов, которые присутствовали в нашем анализе: недопонимание, институциональные барьеры и отсутствие уважения.Таблицы 2 и 3 содержат сводку подтипов, определений и примеров как клинических, так и доклинических UE. Хотя сочетания клинических и доклинических событий были уникальными, мы обнаружили, что их подтипы взаимосвязаны и усугубляют возникновение друг друга.

    Таблица 2.

    клинических UE, процент от общего числа ответов, в которых они были упомянуты, их определения и репрезентативные цитаты

    клинические UE .% от общего количества ответов . Определение . Представительское предложение .
    Лекарства 24,0 Ошибки при назначении или назначении лекарств пациентам [Пациенту] была назначена внутривенная химиотерапия с более чем вдвое большей скоростью . (S162)
    Медицинское оборудование 17,9 Когда медицинское оборудование, используемое для ухода за пациентами, неисправно или неправильно используется Высокий поток не работает должным образом из-за ослабленных соединений и неисправных деталей. (S33)
    Протокол лечения 10,2 Неожиданные отклонения в протоколах лечения Медсестра не забыла надеть перчатки или продезинфицировать руки перед выдачей лекарства. (S198)
    Отсроченный или неправильный диагноз 6,9 Неопределенность или неправильное суждение относительно диагноза пациента [поставщик], не знакомый со сложной историей болезни моего ребенка, поставил неправильный диагноз относительно абсолютного низкого уровня нейтрофилов моего сына. (S103)
    Естественная причина 8,9 Естественные, а иногда и неожиданные реакции пациентов на лечение Лечение вызвало неизвестные реакции и ночную поездку в отделение неотложной помощи. (S80)
    Условия, приобретенные в больнице 6,5 Дополнительные медицинские условия, с которыми пациент сталкивается в связи с их пребыванием в больнице (например, падения, специфические для больницы инфекции) Инфекция стафилококка после операции… требуется дополнительная операция по зашивке бурсы на плече. (R227)
    Неадекватная диета или питание 3,3 Недостаток в питании или гидратации во время пребывания пациента или опекуна в больнице Я откусил [прием пищи], и это было похоже на поедание чистой соли … диета с ограничением количества соли и воды. Я не ел всю ночь. (S287)
    Клинические UE .% от общего количества ответов . Определение . Представительское предложение .
    Лекарства 24,0 Ошибки при назначении или назначении лекарств пациентам [Пациенту] была назначена внутривенная химиотерапия с более чем вдвое большей скоростью . (S162)
    Медицинское оборудование 17,9 Когда медицинское оборудование, используемое для ухода за пациентами, неисправно или неправильно используется Высокий поток не работает должным образом из-за ослабленных соединений и неисправных деталей. (S33)
    Протокол лечения 10,2 Неожиданные отклонения в протоколах лечения Медсестра не забыла надеть перчатки или продезинфицировать руки перед выдачей лекарства. (S198)
    Отсроченный или неправильный диагноз 6,9 Неопределенность или неправильное суждение относительно диагноза пациента [поставщик], не знакомый со сложной историей болезни моего ребенка, поставил неправильный диагноз относительно абсолютного низкого уровня нейтрофилов моего сына. (S103)
    Естественная причина 8,9 Естественные, а иногда и неожиданные реакции пациентов на лечение Лечение вызвало неизвестные реакции и ночную поездку в отделение неотложной помощи. (S80)
    Условия, приобретенные в больнице 6,5 Дополнительные медицинские условия, с которыми пациент сталкивается в связи с их пребыванием в больнице (например, падения, специфические для больницы инфекции) Инфекция стафилококка после операции… требуется дополнительная операция по зашивке бурсы на плече. (R227)
    Неадекватная диета или питание 3,3 Недостаток в питании или гидратации во время пребывания пациента или опекуна в больнице Я откусил [прием пищи], и это было похоже на поедание чистой соли … диета с ограничением количества соли и воды. Я не ел всю ночь. (S287)
    Таблица 2.

    Клинические UE, процент от общего числа ответов, в которых они были упомянуты, их определения и репрезентативные цитаты

    Клинические UE .% от общего количества ответов . Определение . Представительское предложение .
    Лекарства 24,0 Ошибки при назначении или назначении лекарств пациентам [Пациенту] была назначена внутривенная химиотерапия с более чем вдвое большей скоростью . (S162)
    Медицинское оборудование 17,9 Когда медицинское оборудование, используемое для ухода за пациентами, неисправно или неправильно используется Высокий поток не работает должным образом из-за ослабленных соединений и неисправных деталей. (S33)
    Протокол лечения 10,2 Неожиданные отклонения в протоколах лечения Медсестра не забыла надеть перчатки или продезинфицировать руки перед выдачей лекарства. (S198)
    Отсроченный или неправильный диагноз 6,9 Неопределенность или неправильное суждение относительно диагноза пациента [поставщик], не знакомый со сложной историей болезни моего ребенка, поставил неправильный диагноз относительно абсолютного низкого уровня нейтрофилов моего сына. (S103)
    Естественная причина 8,9 Естественные, а иногда и неожиданные реакции пациентов на лечение Лечение вызвало неизвестные реакции и ночную поездку в отделение неотложной помощи. (S80)
    Условия, приобретенные в больнице 6,5 Дополнительные медицинские условия, с которыми пациент сталкивается в связи с их пребыванием в больнице (например, падения, специфические для больницы инфекции) Инфекция стафилококка после операции… требуется дополнительная операция по зашивке бурсы на плече. (R227)
    Неадекватная диета или питание 3,3 Недостаток в питании или гидратации во время пребывания пациента или опекуна в больнице Я откусил [прием пищи], и это было похоже на поедание чистой соли … диета с ограничением количества соли и воды. Я не ел всю ночь. (S287)
    Клинические UE .% от общего количества ответов . Определение . Представительское предложение .
    Лекарства 24,0 Ошибки при назначении или назначении лекарств пациентам [Пациенту] была назначена внутривенная химиотерапия с более чем вдвое большей скоростью . (S162)
    Медицинское оборудование 17,9 Когда медицинское оборудование, используемое для ухода за пациентами, неисправно или неправильно используется Высокий поток не работает должным образом из-за ослабленных соединений и неисправных деталей. (S33)
    Протокол лечения 10,2 Неожиданные отклонения в протоколах лечения Медсестра не забыла надеть перчатки или продезинфицировать руки перед выдачей лекарства. (S198)
    Отсроченный или неправильный диагноз 6,9 Неопределенность или неправильное суждение относительно диагноза пациента [поставщик], не знакомый со сложной историей болезни моего ребенка, поставил неправильный диагноз относительно абсолютного низкого уровня нейтрофилов моего сына. (S103)
    Естественная причина 8,9 Естественные, а иногда и неожиданные реакции пациентов на лечение Лечение вызвало неизвестные реакции и ночную поездку в отделение неотложной помощи. (S80)
    Условия, приобретенные в больнице 6,5 Дополнительные медицинские условия, с которыми пациент сталкивается в связи с их пребыванием в больнице (например, падения, специфические для больницы инфекции) Инфекция стафилококка после операции… требуется дополнительная операция по зашивке бурсы на плече. (R227)
    Неадекватная диета или питание 3,3 Недостаток в питании или гидратации во время пребывания пациента или опекуна в больнице Я откусил [прием пищи], и это было похоже на поедание чистой соли … диета с ограничением количества соли и воды. Я не ел всю ночь. (S287)
    Таблица 3.

    Сводка доклинических UE, процент от общего числа ответов, в которых они были упомянуты, определения и репрезентативные цитаты

    Доклинические UE . Подтипы UE .% от общего количества ответов . Определение . Представительское предложение .
    Непонимание Разрывы между поставщиками 19,5 Неадекватное общение между отдельными членами группы по уходу Больница рекламирует концепцию « командной медицины », но иногда члены команды не следуют одному и тому же плану игры . (S171)
    Отсутствие информации 14,2 Пациентам и лицам, осуществляющим уход, не передается информация, связанная с уходом Никакой реальной информации о [моем сыне] предоставлено не было… Существенное отсутствие связи между родителями и поставщиками услуг. (S105)
    Неточная информация 7,7 Информация неверна или представлена ​​как более надежная, чем на самом деле Я думал, что собираюсь пройти простую операцию … но на самом деле это было сложнее, чем они позволяли на. (S141)
    Отсутствие сочувствия 28,1 То, как поставщик общается, отражает плохую манеру поведения у постели больного или отсутствие сострадания Поставщик был скупым, и она пыталась ослабить мою боль. Она торопилась и не проявляла ничего личного и сострадательного. (S219)
    Институциональные барьеры Отсутствие координации помощи 17,9 Конфликтующие или несвоевременные графики; логистические сбои Запланированное испытание отложено с 9:30 до 17:00, но откладывается на час без объяснения причин. (S179)
    Плохое использование EHR 4.1 Дизайн, удобство использования, доступ и зависимости EHR, которые негативно влияют на уход Медперсонал много времени уделял электронной документации и очень мало внимания уделял пациенту. (S175)
    Обременительная политика 5,7 Организационные правила, негативно влияющие на пациентов и лиц, осуществляющих уход Мне было отказано в услугах в течение 2–3 дней, прежде чем они начнут [ред] на мне из-за страховки. (S269)
    Отсутствие уважения Способность принимать решения 2,4 Способность пациентов или лиц, осуществляющих уход, принимать решения об оказании неотложной помощи ограничена или не рассматривается командой помощи Мне дали слишком много болеутоляющее средство. Я не был компетентен после процедуры, чтобы быть надежным, когда меня спросили: «Ваша боль все еще 7?» (S96)
    Комфортная среда в больнице 8,1 Физическое окружение больницы отрицательно влияет на пациентов и лиц, осуществляющих уход Опыт Все время было неудобно, плохие одеяла, плохие кровати. (S18)
    Личная конфиденциальность 2,4 Пациенты и лица, осуществляющие уход, изо всех сил пытаются установить личные границы с персоналом больницы [Я] просто не люблю, когда люди прикасаются ко мне, не спрашивая … они думают, потому что я ребенок, которого не нужно спрашивать… (S37)
    Доклинические UE . Подтипы UE .% от общего количества ответов . Определение . Представительское предложение .
    Непонимание Разрывы между поставщиками 19,5 Неадекватное общение между отдельными членами группы по уходу Больница рекламирует концепцию « командной медицины », но иногда члены команды не следуют одному и тому же плану игры . (S171)
    Отсутствие информации 14.2 Информация, связанная с уходом, не передается пациентам и лицам, осуществляющим уход. Не было предоставлено никакой реальной информации о [моем сыне]… Существенное отсутствие связи между родителями и поставщиками услуг. (S105)
    Неточная информация 7,7 Информация неверна или представлена ​​как более надежная, чем на самом деле Я думал, что собираюсь пройти простую операцию … но на самом деле это было сложнее, чем они позволяли на. (S141)
    Отсутствие сочувствия 28.1 То, как медработник общается, отражает плохое поведение у постели больного или недостаток сострадания Медработник был скуп и пытался ослабить мою боль. Она торопилась и не проявляла ничего личного и сострадательного. (S219)
    Институциональные барьеры Отсутствие координации помощи 17,9 Конфликтующие или несвоевременные графики; логистические сбои Запланированное испытание отложено с 9:30 до 17:00, но откладывается на час без объяснения причин. (S179)
    Плохое использование EHR 4.1 Дизайн, удобство использования, доступ и зависимости EHR, которые негативно влияют на уход Медперсонал много времени уделял электронной документации и очень мало внимания уделял пациенту. (S175)
    Обременительная политика 5,7 Организационные правила, негативно влияющие на пациентов и лиц, осуществляющих уход Мне было отказано в услугах в течение 2–3 дней, прежде чем они начнут [ред] на мне из-за страховки. (S269)
    Отсутствие уважения Способность принимать решения 2,4 Способность пациентов или лиц, осуществляющих уход, принимать решения об оказании неотложной помощи ограничена или не рассматривается командой помощи Мне дали слишком много болеутоляющее средство. Я не был компетентен после процедуры, чтобы быть надежным, когда меня спросили: «Ваша боль все еще 7?» (S96)
    Комфортная среда в больнице 8,1 Физическое окружение больницы отрицательно влияет на пациентов и лиц, осуществляющих уход Опыт Все время было неудобно, плохие одеяла, плохие кровати. (S18)
    Неприкосновенность частной жизни 2,4 Пациенты и лица, осуществляющие уход, изо всех сил пытаются установить личные границы с персоналом больницы [Я] просто не люблю, когда люди прикасаются ко мне, не спрашивая … они думают, потому что я ребенок, которого не нужно спрашивать… (S37)
    Таблица 3.

    Сводка доклинических UE, процент от общего числа ответов, в которых они были упомянуты, определения и репрезентативные цитаты

    Доклинические UE . Подтипы UE .% от общего количества ответов . Определение . Представительское предложение .
    Непонимание Разрывы между поставщиками 19,5 Неадекватное общение между отдельными членами группы по уходу Больница рекламирует концепцию « командной медицины », но иногда члены команды не следуют одному и тому же плану игры . (S171)
    Отсутствие информации 14,2 Пациентам и лицам, осуществляющим уход, не передается информация, связанная с уходом Никакой реальной информации о [моем сыне] предоставлено не было… Существенное отсутствие связи между родителями и поставщиками услуг. (S105)
    Неточная информация 7,7 Информация неверна или представлена ​​как более надежная, чем на самом деле Я думал, что собираюсь пройти простую операцию … но на самом деле это было сложнее, чем они позволяли на. (S141)
    Отсутствие сочувствия 28,1 То, как поставщик общается, отражает плохую манеру поведения у постели больного или отсутствие сострадания Поставщик был скупым, и она пыталась ослабить мою боль. Она торопилась и не проявляла ничего личного и сострадательного. (S219)
    Институциональные барьеры Отсутствие координации помощи 17,9 Конфликтующие или несвоевременные графики; логистические сбои Запланированное испытание отложено с 9:30 до 17:00, но откладывается на час без объяснения причин. (S179)
    Плохое использование EHR 4.1 Дизайн, удобство использования, доступ и зависимости EHR, которые негативно влияют на уход Медперсонал много времени уделял электронной документации и очень мало внимания уделял пациенту. (S175)
    Обременительная политика 5,7 Организационные правила, негативно влияющие на пациентов и лиц, осуществляющих уход Мне было отказано в услугах в течение 2–3 дней, прежде чем они начнут [ред] на мне из-за страховки. (S269)
    Отсутствие уважения Способность принимать решения 2,4 Способность пациентов или лиц, осуществляющих уход, принимать решения об оказании неотложной помощи ограничена или не рассматривается командой помощи Мне дали слишком много болеутоляющее средство. Я не был компетентен после процедуры, чтобы быть надежным, когда меня спросили: «Ваша боль все еще 7?» (S96)
    Комфортная среда в больнице 8,1 Физическое окружение больницы отрицательно влияет на пациентов и лиц, осуществляющих уход Опыт Все время было неудобно, плохие одеяла, плохие кровати. (S18)
    Личная конфиденциальность 2,4 Пациенты и лица, осуществляющие уход, изо всех сил пытаются установить личные границы с персоналом больницы [Я] просто не люблю, когда люди прикасаются ко мне, не спрашивая … они думают, потому что я ребенок, которого не нужно спрашивать… (S37)
    Доклинические UE . Подтипы UE .% от общего количества ответов . Определение . Представительское предложение .
    Непонимание Разрывы между поставщиками 19,5 Неадекватное общение между отдельными членами группы по уходу Больница рекламирует концепцию « командной медицины », но иногда члены команды не следуют одному и тому же плану игры . (S171)
    Отсутствие информации 14.2 Информация, связанная с уходом, не передается пациентам и лицам, осуществляющим уход. Не было предоставлено никакой реальной информации о [моем сыне]… Существенное отсутствие связи между родителями и поставщиками услуг. (S105)
    Неточная информация 7,7 Информация неверна или представлена ​​как более надежная, чем на самом деле Я думал, что собираюсь пройти простую операцию … но на самом деле это было сложнее, чем они позволяли на. (S141)
    Отсутствие сочувствия 28.1 То, как медработник общается, отражает плохое поведение у постели больного или недостаток сострадания Медработник был скуп и пытался ослабить мою боль. Она торопилась и не проявляла ничего личного и сострадательного. (S219)
    Институциональные барьеры Отсутствие координации помощи 17,9 Конфликтующие или несвоевременные графики; логистические сбои Запланированное испытание отложено с 9:30 до 17:00, но откладывается на час без объяснения причин. (S179)
    Плохое использование EHR 4.1 Дизайн, удобство использования, доступ и зависимости EHR, которые негативно влияют на уход Медперсонал много времени уделял электронной документации и очень мало внимания уделял пациенту. (S175)
    Обременительная политика 5,7 Организационные правила, негативно влияющие на пациентов и лиц, осуществляющих уход Мне было отказано в услугах в течение 2–3 дней, прежде чем они начнут [ред] на мне из-за страховки. (S269)
    Отсутствие уважения Способность принимать решения 2,4 Способность пациентов или лиц, осуществляющих уход, принимать решения об оказании неотложной помощи ограничена или не рассматривается командой помощи Мне дали слишком много болеутоляющее средство. Я не был компетентен после процедуры, чтобы быть надежным, когда меня спросили: «Ваша боль все еще 7?» (S96)
    Комфортная среда в больнице 8,1 Физическое окружение больницы отрицательно влияет на пациентов и лиц, осуществляющих уход Опыт Все время было неудобно, плохие одеяла, плохие кровати. (S18)
    Неприкосновенность частной жизни 2,4 Пациенты и лица, осуществляющие уход, изо всех сил пытаются установить личные границы с персоналом больницы [Я] просто не люблю, когда люди прикасаются ко мне, не спрашивая … они думают, потому что я ребенок, которого не нужно спрашивать… (S37)

    Например, одно событие недопонимания может привести к дальнейшему недопониманию между пациентами, лицами, осуществляющими уход, и поставщиками услуг. Нарушения связи между поставщиками услуг привели к тому, что участники не получали информацию о неотложной помощи.Другие считали, что «плохое поведение у постели больного» привело к тому, что поставщики предоставили неточную или вводящую в заблуждение информацию о лечении пациента. S78 ухаживала за своим ребенком, когда врач, которого она никогда раньше не видела, «обсудила варианты лечения по сценарию худшего [sic] случая… без консультации с остальной командой [ухода]. Он не был врачом, отвечающим за принятие решений о лечении, но скорее консультантом, однако он так себя не называл ». Это взаимодействие вызвало у S78 много ненужного «стресса и беспокойства».

    События недопонимания также взаимосвязаны с институциональными барьерами — UE, возникающими на организационном уровне больницы, от которых пациенты и лица, осуществляющие уход, испытывают последующие эффекты. Эти барьеры состоят из логистических сбоев или сбоев в расписании между отделениями больниц, плохого использования или дизайна электронных медицинских карт (ЭУЗ) и обременительной политики для пациентов и лиц, осуществляющих уход. Например, S246 засвидетельствовала запланированное обновление больничной системы EHR, которое пошло не так, что привело к сбоям связи между ее поставщиками.Она сказала: «[N] уроков и врачей не смогли составить график информации о пациентах, и их общение друг с другом и с другими отделами было серьезно затруднено… это было возмутительно».

    Случаи недопонимания и институциональные барьеры могут привести к тому, что пациенты и лица, осуществляющие уход, не будут чувствовать себя уважаемыми поставщиками или медицинскими организациями. Иногда участники чувствовали себя вынужденными принимать решения, несовместимые с их ценностями, считали, что их физическое окружение лишает их возможностей, и сталкивались с нежелательными ограничениями — или отсутствием внимания — к своей способности заниматься их заботой.Медработники S184 не общались друг с другом, и оба позвонили ей, чтобы принять решение об уходе во время ее внутривенного (внутривенного) введения. Событие «сделало процесс запутанным и разочаровывающим в то время, когда я действительно не был в состоянии принимать важные решения». S26, ухаживала за своим ребенком в отделении интенсивной терапии новорожденных, когда ее семью попросили переехать и освободить место для другого пациента посреди ночи. S26 объяснил: «Этот переход был бы намного более уважительным и вежливым с некоторым предупреждением… когда мы не спим и восстанавливаемся после тяжелой травмы.

    В некоторых случаях доклинические события могут вызвать клинические события (например, S30, педиатрический пациент, сообщил о недопонимании между поставщиками медицинских услуг, что, по его мнению, привело к его неправильному диагнозу). Когда пациенты и лица, осуществляющие уход, испытывают как доклинические, так и клинические события, они усугубляют воспринимаемую тяжесть друг друга. Например, S21 испытывала неуважение во время доклинического UE, когда ее просьба о мануальной терапии, которую она получала в другой больнице, была отклонена из-за политики больницы.Этот негативный опыт ухудшил ее впечатление о ряде более поздних клинических событий, которые она пережила («ежедневные ингаляционные лекарства были пропущены», «уровень сахара в крови проверялся редко»). По мере того, как эти проблемы накапливались, S21 чувствовала, что ее помощь все больше игнорируется ее командой по уходу.

    Этап 2: возможность вмешательства

    После распознавания UE некоторые пациенты не могли вмешаться, потому что они чувствовали себя слишком уязвимыми, чтобы говорить, они не знали о своих правах в данной ситуации или быстро принимаемые решения по уходу означали, что «ущерб был нанесен» до того, как они увидели возможность предотвратить это.Те, у кого была возможность вмешаться, сделали это, напрямую и неоднократно выражая свои опасения поставщикам медицинских услуг, запрашивая обоснование решений по уходу или запрашивая участие высокопоставленных сотрудников и сторонних представителей (например, представителей семьи пациентов) для посредничества. конфликты. Лица, осуществляющие уход, также от имени пациента рекомендовали сообщать о проблемах поставщикам.

    Вмешательство пациента или опекуна иногда признается поставщиками услуг, побуждая каждую сторону к эффективному сотрудничеству и достижению решения, позволяющего избежать потенциального вреда.В некоторых случаях пациентам и лицам, осуществляющим уход, может потребоваться несколько попыток вмешательства для достижения разрешения. S131 испытал это, когда находился в «агонии» из-за проблемы с задержкой мочи и неоднократно просил провести сканирование мочевого пузыря и установить катетер. В конце концов медсестра подтвердила его настойчивость и в течение нескольких минут приняла необходимые меры для катетеризации, уменьшив дальнейший ненужный вред.

    Однако в других случаях попытки вмешательства пациента и лица, осуществляющего уход, не слышны. Несколько участников выразили обеспокоенность или поставили под сомнение решение поставщика медицинских услуг, которое противоречило их собственному опыту в отношении их здоровья.Педиатрические пациенты, обладающие навыками оказания медицинской помощи, просили изменить лечение, чтобы предотвратить ожидаемый вред, но эти опасения часто отвергались или не принимались во внимание. S34 была воспитательницей, которая рассказала о том, что ее сын получает неправильные лекарства: «Несмотря на мои опасения по поводу отсутствия улучшений… [поставщики] пренебрегают моим собственным опытом и историей с моим ребенком, оставляя огромный простор для ошибок». Что касается S50, ее нерешенные проблемы привели к дополнительным клиническим НЯ, когда она запросила альтернативу внутривенной линии ее дочери.Запрос был отклонен, и пациент «перенес несколько неудачных попыток установить новые линии капельницы».

    Этап 3: возникновение вреда

    Если опасения пациентов и лиц, осуществляющих уход, остаются незамеченными, возникает вред различной степени тяжести. Например, S285 пытался убедить своих поставщиков в симптомах, которые он неожиданно испытал после операции на желчном пузыре. Когда его опасения не были услышаны, он посетил отделение неотложной помощи и обнаружил серьезные повреждения из-за хирургических ошибок, которые потребовали месяцев реконструктивной хирургии.

    Дополнительный уход, в котором нуждается S285, представляет собой вид видимого вреда — ощутимые клинические последствия. Однако пациенты и лица, осуществляющие уход, также могут испытывать невидимый вред, который часто остается незамеченным или не сообщается медицинскому персоналу. Например, многие участники не согласны с «пустой тратой времени», вызванной задержками в процедурах и сбоями связи, «дополнительным счетом», вызванным необходимостью получать больше больничной помощи после UE, и тревогой из-за того, что они не получают адекватных объяснений по поводу их помещения под протоколы изоляции.В таблице 4 приведены дополнительные примеры невидимого и видимого вреда.

    Таблица 4.

    Типы невидимого и видимого вреда, процент от общего числа ответов, в которых они упоминались, определения и репрезентативные цитаты

    вред . Подвиды вреда .% от общего количества ответов . Определение . Представительское предложение .
    Невидимый Отрицательные эмоции 26,8 Чувства, которые испытывают пациенты и лица, осуществляющие уход, после НЯ (например, страх, тревога, одиночество, беспомощность) Я был в состоянии крайней паники и [ ] заставил меня опасаться других хирургических процедур. (S129)
    Потеря доверия 3,3 Доверие пациентов и лиц, осуществляющих уход, к своим поставщикам и системе здравоохранения подорвано Я не нервничал, пока они не надели маску … Меня бросили .Все врачи говорили разное. Кому можно доверять ??? (S156)
    Дополнительное жизненное бремя 11,8 Работа или обязанности, добавленные к жизни пациентов или опекунов из-за UE (например, финансовый стресс, поиск ухода за детьми для маленьких братьев и сестер пациентов во время более длительного пребывания в больнице ) «Дренажный« баллон »[дочери] открылся… У меня на рубашке, свитере, штанах и даже на значке родителя есть физиологические жидкости. Как только мы вошли в двухместный номер, нам сказали, чтобы мы не использовали ту ванную комнату (или решали это с соседями).Итак, теперь мне нужно выяснить, где убирать ванную в коридоре… (S36)
    Видимый Боль или дискомфорт 27,6 Плохое обезболивание и физический дискомфорт, которого можно было бы избежать или уменьшился (например, синяк, недостаток сна) Я пошел, чтобы поставить порт, и [медсестра] воткнула его, и это было так больно… Мне было больно, и остался синяк. (S181)
    Ухудшение состояния здоровья 24.8 Общее состояние здоровья пациента ухудшается Состояние моего отца, по моему мнению, ухудшилось из-за отсутствия медсестринского наблюдения за пациентом. (S175)
    Задержка в оказании помощи или выписке 17,1 Отсроченное введение срочной неотложной помощи; сбои в процессе выписки В аптеке возникли трудности с получением моего инсулина … наконец, были введены лекарства, но это заняло как минимум 24 часа. (S223)
    Дополнительный уход или повторная госпитализация 16,7 Пациенту требуется больший уход, чем первоначально ожидалось, или он повторно госпитализирован из-за проблем, связанных с UE. Инфекция в шунтирующей жидкости… им потребовалась экстренная операция, чтобы победить меня. инфекционное заболевание. (S223)
    Нарушение диеты или питания 3,7 Диетические последствия из-за лечения или неправильного питания Пациенту не давали еду до 1:30 утра … в результате он был голоден и расстроен. (S218)
    Хармс . Подвиды вреда .% от общего количества ответов . Определение . Представительское предложение .
    Невидимый Отрицательные эмоции 26,8 Чувства, которые испытывают пациенты и лица, осуществляющие уход, после НЯ (например, страх, тревога, одиночество, беспомощность) Я был в состоянии крайней паники и [ ] заставил меня опасаться других хирургических процедур. (S129)
    Потеря доверия 3,3 Доверие пациентов и лиц, осуществляющих уход, к своим поставщикам и системе здравоохранения подорвано Я не нервничал, пока они не надели маску … Меня бросили . Все врачи говорили разное. Кому можно доверять ??? (S156)
    Дополнительное жизненное бремя 11,8 Работа или обязанности, добавленные к жизни пациентов или опекунов из-за UE (например, финансовый стресс, поиск ухода за детьми для маленьких братьев и сестер пациентов во время более длительного пребывания в больнице ) «Дренажный« баллон »[дочери] открылся… У меня на рубашке, свитере, штанах и даже на значке родителя есть физиологические жидкости.Как только мы вошли в двухместный номер, нам сказали, чтобы мы не использовали ту ванную комнату (или решали это с соседями). Итак, теперь мне нужно выяснить, где убирать ванную в коридоре… (S36)
    Видимый Боль или дискомфорт 27,6 Плохое обезболивание и физический дискомфорт, которого можно было бы избежать или уменьшился (например, синяк, недостаток сна) Я пошел, чтобы поставить порт, и [медсестра] воткнула его, и это было так больно… Мне было больно, и остался синяк. (S181)
    Ухудшение состояния здоровья 24,8 Общее состояние здоровья пациента ухудшается Состояние моего отца, по моему мнению, ухудшилось из-за отсутствия медсестринского наблюдения за пациентом. (S175)
    Задержка в оказании помощи или выписке 17,1 Отсроченное введение срочной неотложной помощи; сбои в процессе выписки В аптеке возникли трудности с получением моего инсулина … наконец, были введены лекарства, но это заняло как минимум 24 часа. (S223)
    Дополнительный уход или повторная госпитализация 16,7 Пациенту требуется больший уход, чем первоначально ожидалось, или он повторно госпитализирован из-за проблем, связанных с UE. Инфекция в шунтирующей жидкости… им потребовалась экстренная операция, чтобы победить меня. инфекционное заболевание. (S223)
    Нарушение диеты или питания 3,7 Диетические последствия из-за лечения или неправильного питания Пациенту не давали еду до 1:30 утра … в результате он был голоден и расстроен. (S218)
    Таблица 4.

    Типы невидимого и видимого вреда, процент от общего числа ответов, в которых они упоминались, определения и репрезентативные цитаты

    Вред . Подвиды вреда .% от общего количества ответов . Определение . Представительское предложение .
    Невидимый Отрицательные эмоции 26.8 Чувства, которые пациенты и лица, осуществляющие уход, испытывают после НЯ (например, страх, тревога, одиночество, беспомощность) Я был в состоянии крайней паники, и [это событие] заставило меня опасаться других хирургических процедур. (S129)
    Потеря доверия 3,3 Доверие пациентов и лиц, осуществляющих уход, к своим поставщикам и системе здравоохранения подорвано Я не нервничал, пока они не надели маску … Меня бросили . Все врачи говорили разное.Кому можно доверять ??? (S156)
    Дополнительное жизненное бремя 11,8 Работа или обязанности, добавленные к жизни пациентов или опекунов из-за UE (например, финансовый стресс, поиск ухода за детьми для маленьких братьев и сестер пациентов во время более длительного пребывания в больнице ) «Дренажный« баллон »[дочери] открылся… У меня на рубашке, свитере, штанах и даже на значке родителя есть физиологические жидкости. Как только мы вошли в двухместный номер, нам сказали, чтобы мы не использовали ту ванную комнату (или решали это с соседями).Итак, теперь мне нужно выяснить, где убирать ванную в коридоре… (S36)
    Видимый Боль или дискомфорт 27,6 Плохое обезболивание и физический дискомфорт, которого можно было бы избежать или уменьшился (например, синяк, недостаток сна) Я пошел, чтобы поставить порт, и [медсестра] воткнула его, и это было так больно… Мне было больно, и остался синяк. (S181)
    Ухудшение состояния здоровья 24.8 Общее состояние здоровья пациента ухудшается Состояние моего отца, по моему мнению, ухудшилось из-за отсутствия медсестринского наблюдения за пациентом. (S175)
    Задержка в оказании помощи или выписке 17,1 Отсроченное введение срочной неотложной помощи; сбои в процессе выписки В аптеке возникли трудности с получением моего инсулина … наконец, были введены лекарства, но это заняло как минимум 24 часа. (S223)
    Дополнительный уход или повторная госпитализация 16,7 Пациенту требуется больший уход, чем первоначально ожидалось, или он повторно госпитализирован из-за проблем, связанных с UE. Инфекция в шунтирующей жидкости… им потребовалась экстренная операция, чтобы победить меня. инфекционное заболевание. (S223)
    Нарушение диеты или питания 3,7 Диетические последствия из-за лечения или неправильного питания Пациенту не давали еду до 1:30 утра … в результате он был голоден и расстроен. (S218)
    Хармс . Подвиды вреда .% от общего количества ответов . Определение . Представительское предложение .
    Невидимый Отрицательные эмоции 26,8 Чувства, которые испытывают пациенты и лица, осуществляющие уход, после НЯ (например, страх, тревога, одиночество, беспомощность) Я был в состоянии крайней паники и [ ] заставил меня опасаться других хирургических процедур. (S129)
    Потеря доверия 3,3 Доверие пациентов и лиц, осуществляющих уход, к своим поставщикам и системе здравоохранения подорвано Я не нервничал, пока они не надели маску … Меня бросили . Все врачи говорили разное. Кому можно доверять ??? (S156)
    Дополнительное жизненное бремя 11,8 Работа или обязанности, добавленные к жизни пациентов или опекунов из-за UE (например, финансовый стресс, поиск ухода за детьми для маленьких братьев и сестер пациентов во время более длительного пребывания в больнице ) «Дренажный« баллон »[дочери] открылся… У меня на рубашке, свитере, штанах и даже на значке родителя есть физиологические жидкости.Как только мы вошли в двухместный номер, нам сказали, чтобы мы не использовали ту ванную комнату (или решали это с соседями). Итак, теперь мне нужно выяснить, где убирать ванную в коридоре… (S36)
    Видимый Боль или дискомфорт 27,6 Плохое обезболивание и физический дискомфорт, которого можно было бы избежать или уменьшился (например, синяк, недостаток сна) Я пошел, чтобы поставить порт, и [медсестра] воткнула его, и это было так больно… Мне было больно, и остался синяк. (S181)
    Ухудшение состояния здоровья 24,8 Общее состояние здоровья пациента ухудшается Состояние моего отца, по моему мнению, ухудшилось из-за отсутствия медсестринского наблюдения за пациентом. (S175)
    Задержка в оказании помощи или выписке 17,1 Отсроченное введение срочной неотложной помощи; сбои в процессе выписки В аптеке возникли трудности с получением моего инсулина … наконец, были введены лекарства, но это заняло как минимум 24 часа. (S223)
    Дополнительный уход или повторная госпитализация 16,7 Пациенту требуется больший уход, чем первоначально ожидалось, или он повторно госпитализирован из-за проблем, связанных с UE. Инфекция в шунтирующей жидкости… им потребовалась экстренная операция, чтобы победить меня. инфекционное заболевание. (S223)
    Нарушение диеты или питания 3,7 Диетические последствия из-за лечения или неправильного питания Пациенту не давали еду до 1:30 утра … в результате он был голоден и расстроен. (S218)

    Видимый и невидимый вред иногда возникают одновременно и влияют друг на друга. Например, боль или дискомфорт привели к отрицательным эмоциям (например, страху, беспомощности). У S267 возникли задержки с получением лекарств, и она почувствовала «огромную физическую и эмоциональную боль, а также потерю уверенности и доверия к моей команде по уходу». Нарушение связи S18 с поставщиками услуг привело к невидимому ущербу, когда он почувствовал, что его поставщики «отказались» от него. Эти негативные отношения привели к тяжелому выздоровлению и отсрочке выписки на «полтора месяца».”

    Этап 4: реакция и отклик

    После возникновения вреда участникам иногда требовалось время, чтобы обработать или согласовать пережитый опыт, и поэтому они не описывали никаких действий, которые они предприняли в ответ. Другие упоминали ожидание зависимости — например, результата теста или другого мнения — перед тем, как принять решение.

    Участники, решившие отреагировать, используют 2 типа подходов. Во-первых, они переводят больницы или требуют, чтобы к их лечению были привлечены новые поставщики услуг.S201 разочаровалась в «большой матричной организации с занятыми людьми», и это впечатление «снизило доверие пациентов [sic] к модели ухода за пациентами. Пациент перешел в другую больничную систему с улучшенной коммуникацией с пациентами ». Однако в некоторых случаях решение о смене больницы было навязано участникам, а не их выбором. Например, S243 было отказано в дополнительной помощи, когда ее медработники думали, что она «симулирует боль. И меня буквально выгнали из [больницы].”

    Второе действие — это то, что пациенты и лица, осуществляющие уход, предупреждают поставщиков о вреде, который они получают, или сообщают о них в больницу. Хотя персонал больницы побуждал некоторых участников подавать официальные жалобы, они столкнулись с препятствиями при этом. S19 испытывала недостаток сочувствия со стороны медсестры, которая игнорировала крики своего ребенка о помощи. Ей сказали, что ее врач поможет ей составить отчет об УП, но «этого не произошло».

    В подмножестве случаев поставщики, участвующие в UE, будут использовать отчет участника как возможность инициировать улучшения.Например, из-за недостатка внимания и общения у ребенка S124 остановилось сердце. После этого мероприятия «общение между врачами улучшилось, и командам было сказано слушать родителей». Для других участников улучшения были внесены в результате их письменной жалобы. S135 была пациенткой подросткового возраста, которая рассказала о своем опыте в местный отдел здравоохранения и на новостные каналы, что побудило провести расследование и переподготовить персонал больницы. Однако не всем, кто подал жалобу, было сказано, как был рассмотрен их отзыв, если вообще было.S63 была сиделкой, которая успешно помешала своей команде по уходу назначить лечение не тому пациенту в их общей больничной палате. Несмотря на то, что она подала жалобу, она была разочарована отсутствием обновленной информации: «На данный момент я не знаю, разговаривали ли [с командой] [редактировать]. Было бы неплохо узнать, что дальнейшие действия продолжаются так далеко ».

    ОБСУЖДЕНИЕ

    Наша концептуальная модель показывает, как пациенты испытывают НЯ и причиняют им вред, а также выделяет конкретные действия, которые они предпринимают, и их последствия.Это подробное представление помогает нам понять, как информационные системы могут способствовать обеспечению безопасности пациентов. В следующих разделах мы обсудим, как нынешние и будущие информационные системы, ориентированные на пациентов, могут быть интегрированы на каждом этапе их опыта (рис. 2).

    Рисунок 2.

    Наша четырехступенчатая концептуальная модель с аннотациями для каждого этапа справа. Аннотации резюмируют прошлые исследования и будущие возможности решений в области информатики для привлечения пациентов к обеспечению их безопасности.УП: нежелательное событие.

    Рис. 2.

    Наша четырехступенчатая концептуальная модель с аннотациями для каждого этапа справа. Аннотации резюмируют прошлые исследования и будущие возможности решений в области информатики для привлечения пациентов к обеспечению их безопасности. УП: нежелательное событие.

    Повышение осведомленности UE на этапе 1

    Многие существующие технологии, ориентированные на пациента, относятся к этапу 1 модели. В частности, эти технологии делают информацию, содержащуюся в EHR, доступной для пациентов и тех, кто за ними ухаживает.Эти технологии обеспечивают прозрачность, предлагая пациентам цифровой справочник для наблюдения за их лечением и выявления проблем, которые могут привести к UE и вреду. Например, информационные дисплеи и мобильные приложения помогли пациентам в отделениях неотложной помощи снизить обеспокоенность по поводу обсуждения аллергии и лекарств с поставщиками медицинских услуг. 18 , 19 Наборы инструментов для предотвращения падений и заставки безопасности предоставили пациентам индивидуальные планы безопасности, которые повысили их восприятие факторов риска, которые могут привести к ошибкам в их уходе. 20 , 21 Используя порталы пациентов, 22 OpenNotes, 23 и инструменты согласования лекарств, пациенты 24 смогли отслеживать и выявлять ошибки в своих записях.

    Несмотря на обширные исследования, демонстрирующие преимущества таких технологий с точки зрения безопасности, признание UE — лишь один из аспектов опыта пациентов. Учитывая неизбежность человеческой ошибки и проблемы, с которыми системы здравоохранения сталкиваются при понимании сложной природы безопасности больниц, существуют дополнительные малоизученные возможности для технологий, ориентированных на пациентов, для адаптации к последующим этапам модели.

    Поощрение вмешательства и разрешения на этапе 2

    Когда пациенты проходят стадию 2 модели, им требуются инструменты, которые как поощряют их способность вмешиваться в UE, так и гарантируют, что решение может быть достигнуто с помощью их медицинской бригады. Хотя ориентированные на пациента технологии, которые извлекают информацию из электронной записи пациента, могут быть полезны в предупреждении медработников об ошибках в их записях, пациентам также нужны инструменты, когда их опасения не принимаются во внимание медицинской командой.Одним из примеров такого инструмента является MySafeCare, который позволяет пациентам вмешиваться в UE в режиме реального времени с небольшими нагрузками, не требуя прямого взаимодействия с поставщиками услуг. 25 , 26 Этот неконфликтный формат помогает пациентам высказывать свои опасения по мере их возникновения.

    Будущие исследователи могут продолжить эту работу, исследуя, как теории мотивации и поведения могут быть использованы в технологиях, ориентированных на пациента, чтобы помочь пациентам преодолеть препятствия, с которыми они сталкиваются, чтобы высказаться. 27 , 28 Эти технологии могут предоставить пациентам информацию о серьезности риска невмешательства в UE. Порталы для стационаров могут предлагать пациентам функции, позволяющие запрашивать сторонних посредников (например, группы взаимоотношений между пациентом и семьей) для разрешения конфликтов. Системы могут использовать упражнения по развитию навыков, чтобы научить пациентов и поставщиков вести сложные дискуссии по вопросам безопасности, 29 , 30 , или могут действовать как нейтральная платформа для модерации разговоров, связанных с UE. 31 Сравнимые пациенты также известны тем, что помогают пациентам вмешиваться в UE, 32 и нуждаются в обмене информацией, связанной с безопасностью, друг с другом. 33–35 Системы могут связывать пациентов друг с другом во время их пребывания в больнице, чтобы обмениваться советами и нормализовать их поведение.

    Предоставление таких технологий пациентам во время их пребывания в больнице может подтолкнуть их к решению вмешаться и избежать причинения вреда. Кроме того, технологии, которые поддерживают способность пациентов вмешиваться на этапе 2, могут привести к тому, что больницы обнаружат более важные области для улучшения и предотвращения UE.

    Выявление и устранение невидимого вреда на стадии 3

    Этап 3 модели показывает, что невидимый вред включает в себя как «эмоциональный вред», так и жизненное бремя, которое, в отличие от видимого вреда, является нефизическим и труднодоступным для медработников, или пациентам сложно передать их своим поставщикам. 36 Таким образом, существуют возможности для будущих решений в области информатики для поддержки обнаружения провайдерами этого невидимого вреда, чтобы его можно было признать и устранить.Например, исследователи изучили обработку социальных сигналов как способ для пациентов и поставщиков медицинских услуг в клинике получать в реальном времени обратную связь о тоне и сочувствии, передаваемом во время их разговоров. 37 , 38 Аналогичные решения по обработке социальных сигналов могут быть адаптированы для стационарных условий, чтобы вызвать негативные эмоции или снижение доверия, которое пациенты могут выразить после работы с UE. Это может повысить видимость такого вреда, чтобы поставщики могли инициировать соответствующий ответ.

    Помимо использования инструментов для обнаружения невидимого вреда, медицинские организации могут предоставить ресурсы, которые помогут пациентам справиться с невидимым вредом. В частности, дополнительное жизненное бремя, которое пациенты берут на себя после НЯ, состоит из невидимых усилий в дополнение к их обязанностям по поддержанию своего здоровья. 39 Предыдущая работа показала, как информационные решения, ориентированные на пациентов, включая ресурсы цифровой навигации по раку и консультационные услуги по взаимодействию с пациентами, могут помочь пациентам решить эмоциональные, социальные, финансовые и логистические проблемы, с которыми они сталкиваются во время лечения. 40 , 41 Будущие исследователи могли бы изучить, как эти решения могут помочь пациентам контекстуализировать UE, обработать нанесенный вред и поддержать их способность реагировать на этот вред на стадии 4 таким образом, чтобы стимулировать позитивные изменения в система здравоохранения.

    Снижение барьеров для отчетности на этапе 4

    Испытав вред, участники нашего исследования решили отреагировать на этапе 4, сообщив о своем опыте, чтобы убедиться, что UE не повторится снова, или переключив медицинское учреждение после потери доверия к больнице.Однако, когда пациенты предпочитают не сообщать о своем опыте, больницы упускают возможность поучиться у UE и предотвратить подобные проблемы. Таким образом, больницы должны способствовать прозрачности и устойчивости, а также уменьшать препятствия для отчетности. Хотя программы коммуникации и разрешения проблем (например, структура коммуникации и оптимального разрешения) 42 предназначены для продвижения культуры прозрачного раскрытия ошибок, они традиционно позволяют сообщать о событиях, обнаруженных поставщиками услуг или больницами, а не о событиях, обнаруженных пациентами.В больницах должны быть системы, позволяющие пациентам самостоятельно сообщать о событиях и вреде, а также делать их известными и легко доступными для пациентов. Например, системы могут поддерживать мультимодальную отчетность в виде телефонных звонков, текстовых сообщений, электронной почты, форм, которые можно отправлять через порталы для пациентов, или голосовых технологий, таких как Amazon Alexa или Google Assistant, которые все чаще используются в стационарах. настройки. 43 Предложение нескольких методов отчетности позволит пациентам с ситуативными нарушениями — когнитивными или физическими трудностями из-за болезни или лечения — отправлять отчеты с минимальными усилиями. 44 ​​

    После того, как пациенты отправят свои отчеты, они очень заинтересованы в том, чтобы узнать, как больница обрабатывает их отчеты и какие изменения вносятся в результате. 45 Тем не менее, такие участники, как S63, никогда не были проинформированы об этих изменениях, и отсутствие таких обновлений может усугубить вред, который испытывают пациенты. 46 , 47 Для решения этой проблемы больницы могут предоставить пациентам инструменты, которые отслеживают статус их отчета от начала до конца.Предыдущие исследования предлагали функции отслеживания на стационарных порталах, чтобы держать пациентов в курсе динамической информации об уходе. 48 В контексте сообщения о событии инструмент отслеживания может включать в себя (1) краткое изложение процесса и шагов, которые пользовательское оборудование, о котором сообщается, предпринимает в организации; (2) текущий статус UE, о котором было сообщено; (3) список лиц, ответственных за обработку и проверку UE на каждом этапе, включая их роль, имя и контактную информацию; и (4) любые системные изменения, внесенные в результате процесса проверки.Поскольку процесс рассмотрения отчета может выходить далеко за рамки выписки пациента, такой инструмент отслеживания должен быть доступен пациенту во время и после его пребывания в больнице.

    Ограничения и будущая работа

    Наша концептуальная модель раскрывает возможности систем медицинской информатики, чтобы помочь пациентам на каждом этапе их опыта работы с UE, но мы признаем ограничения нашего исследования. Возможна предвзятость отбора, поскольку участие было добровольным и безвозмездным.Участники, представившие несколько ответов, внесли в наши данные некоторую демографическую избыточность. Поскольку наше исследование проводилось в одной детской больнице и одной больнице для взрослых в городской зоне Соединенных Штатов, наши результаты могут не распространяться на более широкие географические или демографические группы населения.

    В дополнение к изучению возможностей для решений в области информатики, ориентированных на пациента, описанных ранее, наши результаты указывают на несколько направлений будущей работы. Поскольку наша модель представляет точки зрения пациентов, требуется дополнительная работа для учета факторов, влияющих на эти точки зрения, таких как раскрытие UE, неоднозначность определения UE и вреда, а также различные пороговые значения для пациентов, переносящих UE, и соответствующим образом доработать модель.Эта работа может открыть дополнительные возможности для эффективных решений в области информатики.

    ЗАКЛЮЧЕНИЕ

    В этой статье мы описываем концептуальную модель, основанную на результатах опроса об опыте пациентов с UE. Наши результаты, полученные от 242 участников, демонстрируют сложность их опыта и представляют поэтапный подход к пониманию безопасности больниц с их точки зрения. Наша модель выявляет потребность в информационных решениях для поддержки пациентов на всех этапах их жизненного опыта путем повышения осведомленности о UE, поощрения вмешательства и решения проблем, выявления и устранения невидимых повреждений, а также снижения барьеров и обмена обновлениями отчетов.Такие решения в области информатики представляют собой новые возможности обучения для систем здравоохранения, позволяющие уменьшить вредные явления и признать пациентов равноправными партнерами в повышении безопасности больниц.

    ФИНАНСИРОВАНИЕ

    Это исследование было поддержано Агентством медицинских исследований и качества 1R01HS022894, присужденным доктору наук Ванде Пратт, а также грантом T15LM007442 Национальной медицинской библиотеки по биомедицинской и медицинской информатике.

    ВКЛАД АВТОРА

    SH руководил концептуализацией исследования, дизайном, сбором данных, анализом и автором статьи.SRM участвовала в сборе, анализе данных, написала часть статьи, а также предоставила отзывы и исправления по черновикам. AHP и WP участвовали в концептуализации исследования, предоставили отзывы о дизайне, написали часть статьи и предоставили свои отзывы и правки.

    БЛАГОДАРНОСТИ

    Мы хотели бы поблагодарить всех наших участников за их вклад в это исследование. Мы также благодарим за усилия и поддержку следующих членов исследовательской группы: Барри Ааронсона, Кристин Чан, Кейт Никель, Алекса Филипковски, Кори Брауна, Келси Айелло, Рашми Элера, Кэлвина Аподака, Махера Хелифи и Юджонга Кима.

    ЗАЯВЛЕНИЕ О КОНФЛИКТЕ ИНТЕРЕСОВ

    Не объявлено.

    ССЫЛКИ

    1

    Макары

    MA

    ,

    Даниил

    М.

    Медицинская ошибка — третья по значимости причина смерти в США

    .

    BMJ

    2016

    ;

    353

    :

    i2139.

    2

    Longtin

    Я

    ,

    Sax

    H

    ,

    Leape

    LL

    и др..

    Участие пациентов: текущие знания и применимость к безопасности пациентов

    .

    Mayo Clin Proc

    2010

    ;

    85

    (

    1

    ):

    53

    62

    ,3

    Белл

    СК

    ,

    Мартинес

    W.

    Каждому пациенту должно быть разрешено отключать линию

    .

    BMJ Qual Saf

    2019

    ;

    28

    (

    3

    ):

    172

    6

    .4

    Вайнгарт

    SN

    ,

    Пагович

    О

    ,

    Пески

    ДЗ

    и др. .

    Что госпитализированные пациенты могут рассказать нам о побочных эффектах? Изучение инцидентов, о которых сообщили пациенты

    .

    J Gen Intern Med

    2005

    ;

    20

    (

    9

    ):

    830

    6

    ,5

    Эчегарай

    JM

    ,

    Ottosen

    MJ

    ,

    Aigbe

    A

    и др..

    Пациенты как партнеры в обучении на неожиданных событиях

    .

    Health Serv Res

    2016

    ;

    51

    :

    2600

    14

    ,6

    Галлахер

    TH

    ,

    Mazor

    КМ.

    Серьезное отношение к жалобам: использование защитных линз пациента

    .

    BMJ Qual Saf

    2015

    ;

    24

    (

    6

    ):

    352

    5

    .7

    Schwappach

    Д.

    Привлечение пациентов как бдительных партнеров к обеспечению безопасности систематический обзор

    .

    Med Care Res Ред.

    2010

    ;

    67

    (

    2

    ):

    119

    48

    ,8

    Унру

    кт

    ,

    Pratt

    W.

    Пациенты как действующие лица: роль пациента в обнаружении, предотвращении и исправлении медицинских ошибок

    .

    Int J Med Inform

    2007

    ;

    76

    Дополнение 1

    :

    S236

    44

    .9

    Харрисон

    R

    ,

    Walton

    M

    ,

    Manias

    E

    и др. .

    Отсутствующие доказательства: систематический обзор опыта пациентов в отношении побочных эффектов в здравоохранении

    .

    Int J Qual Health Care

    2015

    ; 27:

    1

    19

    .10

    Коллинз

    S

    ,

    Couture

    B

    ,

    Dykes

    PC

    и др. .

    Внедрение, оценка и рекомендации по расширению общих форматов AHRQ для сбора данных о безопасности, генерируемых пациентами и партнерами по уходу

    .

    JAMIA Open

    2018

    ;

    1

    :

    20

    25

    .11

    О’Хара

    JK

    ,

    Lawton

    RJ.

    На распутье? Ключевые проблемы и будущие возможности для участия пациентов в обеспечении безопасности пациентов

    .

    BMJ Qual Saf

    2016

    ;

    25

    :

    565

    8

    .12

    Фишер

    KA

    ,

    Smith

    KM

    ,

    Gallagher

    T

    и др..

    Мы хотим знать: комфорт пациента, рассказывающий о сбоях в уходе и опыте пациентов

    .

    BMJ Qual Saf

    2019

    ; 28 (3): 190–7,13

    Белл

    СК

    ,

    Roche

    SD

    ,

    Mueller

    A

    и др. .

    Высказывание о проблемах, связанных с уходом в отделении интенсивной терапии: отношение к пациенту и семье

    .

    BMJ Qual Saf

    2018

    ; 27 (11): 928–36,14

    Зал

    Дж

    ,

    Торф

    M

    ,

    Биркс

    Y

    и др. .

    Эффективность вмешательств, направленных на вовлечение пациентов в повышение безопасности: систематический обзор

    .

    Qual Saf Health Care

    2010

    ;

    19

    :

    e10.

    15

    Епископ

    AC

    ,

    Макдональд

    м.

    Участие пациентов в обеспечении безопасности пациентов: качественное исследование медперсонала и восприятия пациентов

    .

    J Сейф пациента

    2017

    ;

    13

    :

    82–7

    .16

    Добыча

    JE

    ,

    Шерсть

    J

    ,

    Wilcox

    L

    и др..

    Вовлечение пациентов в стационарных условиях: систематический обзор

    .

    J Am Med Informatics Assoc

    2014

    ;

    21

    (

    4

    ):

    742

    50

    ,17

    Браун

    В

    ,

    Кларк

    В.

    Использование тематического анализа в психологии

    .

    Qual Res Psychol

    2006

    ;

    3

    (

    2

    ):

    77

    101

    .18

    Wilcox

    л

    ,

    Morris

    D

    ,

    Tan

    D

    и др. .

    Создание информационных дисплеев, ориентированных на пациента, для больниц

    . В: материалы 28-й Международной конференции по человеческому фактору в вычислительных системах

    -CHI’10;

    2010

    :

    2123

    ,19

    Пфайфер Вардулакис

    л

    ,

    Karlson

    A

    ,

    Morris

    D

    и др..

    Использование мобильных телефонов для предоставления медицинской информации пациентам больниц

    . В: сборник материалов

    2012 ACM Annual Conference on Human Factors Computing Systems-CHI’12

    ;

    2012

    :

    1411

    ,20

    Дакворт

    м

    ,

    Leung

    E

    ,

    Fuller

    T

    и др. .

    Восприятие медсестрой, пациентом и партнером по уходу персонализированной заставки плана безопасности

    .

    J Gerontol Nurs

    2017

    ;

    43

    (

    4

    ):

    15

    22

    ,21

    Дайки

    ПК

    ,

    Duckworth

    M

    ,

    Cunningham

    S

    и др. .

    Пилотные испытания осенних СОВЕТОВ (индивидуальные меры по обеспечению безопасности пациентов): ориентированный на пациента набор инструментов для предотвращения падений

    .

    Jt Comm J Qual Patient Saf

    2017

    ;

    43

    (

    8

    ):

    403

    13

    .22

    Шерстяное

    Дж

    ,

    Prey

    J

    ,

    Wilcox

    L

    и др. .

    Опыт пациента с использованием личной истории болезни пациента

    .

    Аппл Клин Информ

    2016

    ;

    7

    :

    446

    60

    ,23

    Гроссман

    л

    ,

    Masterson Creber

    R

    ,

    Restaino

    S

    и др..

    Обмен клиническими записями с госпитализированными пациентами через портал неотложной помощи

    .

    AMIA Annu Symp Proc 2017;

    2017

    :

    800

    9

    ,24

    Добыча

    JE

    ,

    Полубрягиноф

    Ф

    ,

    Гроссман

    LV

    и др. .

    Вовлечение пациентов больниц в процесс согласования лекарств с помощью планшетных компьютеров

    .

    J Am Med Inform Assoc

    2018

    ;

    25

    :

    1460

    9

    ,25

    Коллинз

    S

    ,

    Couture

    B

    ,

    Smith

    AD

    и др. .

    Оценка смешанными методами отчетов о безопасности в режиме реального времени госпитализированными пациентами и их партнерами по уходу: приложение MySafeCare

    .

    J Пациент Saf

    2018

    27 апреля [Электронный паб перед печатью].26

    От кутюр

    В

    ,

    Lilley

    E

    ,

    Chang

    F

    и др. .

    Применение методов проектирования, ориентированного на пользователя, для разработки приложения мобильного здравоохранения для использования пациентами и партнерами по уходу в больницах

    .

    Аппл Клин Информ

    2018

    ; 9:

    302

    12

    ,27

    Айзен

    И.

    Теория планового поведения

    .

    Org Behav Hum Decis Process

    1991

    ;

    50

    (

    2

    ):

    179

    211

    ,28

    Бандура

    А.

    Самоэффективность

    . В: Friedman H, ed.

    Энциклопедия психического здоровья.Сан-Диего, Калифорния: Academic Press;

    1998

    ; 4:

    1

    65

    .30

    Галлахер

    TH

    ,

    Mello

    MM

    ,

    Sage

    WM

    и др. .

    Могут ли программы коммуникации и разрешения раскрыть свой потенциал? Пять ключевых вопросов

    .

    Health Aff

    2018

    ;

    37

    (

    11

    ):

    1845

    52

    .31

    Райан

    ,

    Luz

    S

    ,

    Albert

    P

    и др. .

    Использование искусственного интеллекта для оценки коммуникативных навыков врачей

    .

    BMJ

    2019

    ;

    364: L161

    .32

    Duhn

    л

    ,

    Медвес

    J.

    5-гранная структура, описывающая участие пациента в обеспечении безопасности пациента

    .

    Heal Expect

    2018

    ; 21 (6): 1122–33.33

    Халдар

    S

    ,

    Мишра

    SR

    ,

    Хелифи

    M

    и др. .

    Возможности и особенности дизайна для поддержки со стороны сверстников в условиях больницы

    .В: материалы конференции CHI 2017 г. по человеческому фактору в вычислительных системах

    -CHI ’17

    ;

    2017

    :

    867

    79

    ,34

    Халдар

    S

    ,

    Мишра

    SR

    ,

    Хелифи

    M

    и др. .

    Изучение конструкции инструмента поддержки сверстников в стационаре: взгляды взрослых пациентов

    . В:

    AMIA Annu Symp Proc

    2018

    ; 2018:

    1282

    91

    .35

    Халдар

    S

    ,

    Мишра

    SR

    ,

    Хелифи

    M

    и др. .

    Значение поддержки пациентов и сверстников в повышении безопасности больниц

    . Stud Health Technol Inform

    2019

    ;

    257

    :

    152

    6

    ,36

    Сокол-Хесснер

    л

    ,

    Folcarelli

    PH

    ,

    Пески

    K.

    Эмоциональный вред от неуважения: игнорирование предотвратимого вреда

    .

    BMJ Qual Saf

    2015

    ;

    24

    (

    9

    ):

    550

    3

    ,37

    Пател

    R

    ,

    Hartzler

    A

    ,

    Czerwinski

    M

    , et al. .

    Визуальная обратная связь о невербальном общении: исследование дизайна с профессионалами в области здравоохранения

    .В: материалы 7-й Международной конференции по распространенным вычислительным технологиям для здравоохранения и семинаров, 2013 г .;

    2013

    ,38

    Хартцлер

    AL

    ,

    Patel

    RA

    ,

    Czerwinski

    M

    и др. .

    Обратная связь в реальном времени о невербальном клиническом общении

    .

    Methods Inf Med

    2014

    ;

    53

    :

    389

    405

    .39

    Gui

    Х

    ,

    Chen

    Y.

    Обеспечение работы инфраструктуры здравоохранения. В: Материалы конференции CHI 2019 по человеческому фактору в вычислительных системах-CHI’19;

    2019

    :

    1

    14

    ,40

    Джейкобс

    м

    ,

    Clawson

    J

    ,

    Mynatt

    ED.

    Мой путевой компас: предварительное исследование мобильного инструмента для онкологических больных

    .

    В: CHI’14 Труды конференции SIGCHI по человеческому фактору в вычислительных системах. Нью-Йорк: ACM;

    2014

    :

    663

    72

    .41

    Джексон Г.П., Робинсон Дж. Р., Инграм Э. и др. .

    Технологическая консультационная служба по взаимодействию с пациентами и их членами в педиатрической больнице

    .

    J Am Med Inform Assoc

    2017

    ;

    25

    :

    167

    74

    .44

    Моррис

    D

    ,

    Карлсон

    А.

    Требования динамической доступности для пациентов больниц

    .

    В: Семинар CHI 2011 по динамической доступности;

    2011

    ,45

    Мур

    Дж

    ,

    Bismark

    M

    ,

    Mello

    MM.

    Опыт пациентов с программами общения и разрешения проблем после травм

    .

    JAMA Intern Med

    2017

    ;

    177

    (

    11

    ):

    1595

    603

    .46

    Hannawa AF.

    Что представляет собой «компетентное раскрытие ошибок»? Выводы национального исследования фокус-группы в Швейцарии

    .

    Swiss Med Wkly

    2017

    ;

    147

    :

    w14427

    .47

    Wu

    AW

    ,

    McCay

    L

    ,

    Levinson

    W

    и др..

    Сообщение пациентам о нежелательных явлениях: международные нормы и тенденции

    .

    J Сейф пациента

    2017

    ;

    13

    (

    1

    ):

    43

    9

    , 48

    Халдар

    S

    ,

    Мишра

    SR

    ,

    Хелифи

    M

    и др. .

    За пределами портала для пациентов: поддержка госпитализированных пациентов

    .В:

    CHI Conference on Human Factors in Computing Systems Proceedings (CHI 2019)

    ;

    2019

    .

    © Автор (ы) 2019. Опубликовано Oxford University Press от имени Американской ассоциации медицинской информатики.

    Это статья в открытом доступе, распространяемая в соответствии с условиями некоммерческой лицензии Creative Commons Attribution (http://creativecommons.org/licenses/by-nc/4.0/), которая разрешает некоммерческое повторное использование, распространение, и воспроизведение на любом носителе при условии правильного цитирования оригинала.По вопросам коммерческого повторного использования обращайтесь по адресу [email protected].

    Отвечая на вопрос «Почему вы хотите здесь работать?» — SJSU

    Опубликовано: 9 сентября 2020 г. iSchool Career Advisor

    «Почему вы хотите здесь работать?» или «Почему вы подали заявку на эту должность?» Это распространенные вопросы собеседования, над которыми вам стоит подумать перед тем, как заглянуть в свое собеседование. Подумайте, что побудило вас подать заявку на эту конкретную должность и что вам нравится в этой работе? Ваша цель — показать, что вы идеально подходите для этой должности.

    Интервьюер хочет узнать о вас и ваших карьерных целях, а также о том, как эта должность соответствует вашему плану. Они хотят убедиться, что вы искренне заинтересованы в этой работе и что вы полны энтузиазма, даже увлечены ею, и что у вас будет мотивация к ее выполнению.

    Итак, как часть процесса подготовки к собеседованию, придумайте убедительный ответ, который конкретно продемонстрирует, почему вы заинтересованы в этой работе. Вот несколько советов, которые стоит учесть.

    • Уметь сформулировать, что вас волнует в этой должности.
    • Расскажите, как вам подходят ваш прошлый опыт, навыки и сильные стороны.
    • Продемонстрируйте свой энтузиазм и искренний интерес к работе.

    Вот два примера ответов на этот важный вопрос

    «Мне нравится энергия работы в кампусе колледжа. Это вдохновляет, и мне нравится отвечать на рекомендательные вопросы учащихся. Мне также нравится разнообразие покровителей и знание того, что я помогаю людям и оказываю услуги.Во время учебы в аспирантуре у меня развились способности и сильный интерес к социальным сетям, и я был очарован тем, как они могут соединять людей по всему миру с информацией; Я действительно хочу быть частью этого ».

    «Меня интересует эта должность по трем основным причинам. Во-первых, я заинтересован в работе в моей альма-матер, и я нахожусь в том моменте своей жизни, когда теперь могу отдавать и помогать изменить жизнь людей, идущих на моем месте.Во-вторых, мне особенно интересно работать с этой группой людей. Я знаю, что проблемы, вопросы и проблемы, с которыми я буду сталкиваться ежедневно, будут разными и сложными, и я с нетерпением жду, когда смогу продолжить исследования и узнать новое. В-третьих, мне интересна эта работа, потому что весь мой опыт и получение степени MLIS хорошо подготовили меня к следующему шагу в моей карьере. Я обладаю высокой квалификацией в области справочных, инструктивных и технических услуг и с нетерпением жду возможности применить эти навыки в вашей библиотеке.”

    Дополнительные советы по успешному прохождению собеседования можно найти в разделе собеседований на нашем веб-сайте по развитию карьеры и получить уникальный опыт отработки навыков проведения собеседования; попробуйте наш онлайн-инструмент для имитации собеседований Big Interview.

    Образцы документов CBSE для Практики информатики 2 класса 12

    Образцы документов CBSE для Практики информатики 2 класса 12 являются частью Образцов документов CBSE для Практики информатики 12 класса.Здесь мы предоставили образцы документов CBSE для практического использования информатики класса 12 2.

    Образцы документов CBSE для практики информатики класса 12 2

    Board CBSE
    Class XII 902
    Предмет Практика информатики
    Набор образцов бумаги Документ 2
    Категория Образцы материалов CBSE

    для студентов, которые собираются на экзамены в класс 12 рекомендуется практиковать приведенные здесь образцы работ CBSE, разработанные в соответствии с последней программой, и схема выставления оценок, предписанная CBSE, приведена здесь.Документ 2 Solved CBSE Sample Paper for Class 12 Informatics Practices представлен ниже вместе с бесплатными решениями для загрузки в формате PDF.

    Время: 3 часа
    Максимальное количество оценок: 70

    Общие инструкции

    • Все вопросы обязательны.
    • Отвечайте на вопросы, внимательно прочитав текст.

    ВОПРОСЫ

    Вопрос 1.
    (а) Театр «Чалчитра» имеет компьютерную сеть. Сеть находится в одном здании
    (i) Назовите этот тип сети (вне LAN / MAN / WAN).
    (ii) Назовите один канал связи, который можно использовать для быстрой связи между рабочими станциями сети.
    (b) Кратко объясните любые 2 угрозы безопасности компьютерных сетей.
    (c) Напишите преимущества использования Unicode для представления текста.
    (d) Напишите по одному примеру URL и IP-адреса.
    (e) Определите топологию, показанную ниже. Напишите 2 преимущества этой топологии.

    Вопрос 2.
    (a) Во время работы в Netbeans г-жа Соня разработала страницу входа, на которой она хочет отображать сообщение «Добро пожаловать» или «Повторить попытку» в зависимости от пароля, введенного пользователем в текстовое поле с именем ‘jTextFieldl’.Если введен пароль «Индия», должно отображаться сообщение «Добро пожаловать», в противном случае должно отображаться сообщение «Повторить попытку». Помогите ей выбрать более подходящую формулировку из «Если» и «Переключить». Обоснуйте свой выбор.
    (b) Напишите код Java, чтобы присвоить значение 10 переменной x и сохранить его квадратное значение в другой переменной y.
    (c) Дипти работает программистом в туристической компании. Она разработала следующий код для отображения сведений о поездках по выбору пользователя.Помогите ей переписать тот же код с помощью SWITCH CASE:

     if (choice == 1)
    jTextFieldl.setText («Нью-Дели - Гоа»);
    иначе, если (выбор = 2)
    jTextFieldl.setText («Нью-Дели - Париж»);
    иначе если (выбор = 3)
    jTextFieldl.setText («Нью-Дели - Бангкок»);
    еще
    jTextFieldl.setText («ИП. выберите допустимый вариант»); 

    (d) Шамбхави должен разработать две веб-страницы со следующими спецификациями:
    (i) Одна веб-страница должна иметь неупорядоченный список.
    (ii) Другая веб-страница должна иметь «желтый» фон.
    Предложите ей подходящие теги и атрибуты для указанных выше спецификаций.

    (e) Альберт работает разработчиком веб-сайтов в компании Global Website Designers. В настоящее время он создал следующие теги в XML:

      ... 
     ...  

    Являются ли эти теги частью кода HTML или кода XML?
    Они такие же или разные? .

    (f) Сколько раз будет выполняться цикл?

     int значение1 = 7. значение2 = 19;
    делать
    {
    JOptionPane.showMessageDialog (ноль, значение + значение2);
    значение = значение + 2;
    значение2 = значение2-2;
    } while (значение <= значение2); 

    Вопрос 3.
    (a) Рассмотрим таблицу'empsalary '

    Чтобы выбрать кортежи с некоторой зарплатой, Сиддхарт написал следующий ошибочный оператор SQL:

    SELECT ID. Зарплата ОТ empsalary ГДЕ Зарплата = что-то;

    Напишите правильный оператор SQL.

    (б) Рассмотрим таблицу «Сотрудник».

    Напишите команду SQL, чтобы получить следующий результат:

    (c) При создании таблицы Student на прошлой неделе, Ms.Шарма забыл включить столбец GamePlayed. Теперь напишите команду для вставки столбца GamePlayed с типом данных VARCHAR и размером 30 в таблицу Student.

    (d) В столбце «Оценки» таблицы «Студент» для Rollnumber 2 учитель класса ввел оценки как 45. Однако произошла ошибка суммирования, и ученик увеличил свои оценки на 5. Какую команду MySQL следует использовать изменить оценки в таблице «Студент».

    (д) Рассмотрите таблицу «Учитель», приведенную ниже.

    Каким будет результат следующих запросов на основе приведенной выше таблицы:

     (i) Выберите количество (Отдел) от Учителя;
    (ii) Выберите счет (*) от Учителя; 

    (f) Рассмотрим таблицу Stu,

    Следующие SQL-запросы выполняются в приведенной выше таблице

     INSERT INTO Stu VALUES (5.«Гаган»);
    СОВЕРШИТЬ ;
    ОБНОВЛЕНИЕ Stu SET NAME = ‘Abhi’ WHERE ROLLNO = 4
    SAVEPOINT A;
    ВСТАВИТЬ В ЗНАЧЕНИЯ Стю (6, «Крис»);
    SAVEPOINT B;
    ВСТАВИТЬ В СТЮ ЦЕННОСТИ (7, «Бабита»);
    SAVEPOINT C;
    ОТКАТ НА B; 

    Что теперь будет на выходе следующего SQL-запроса:

     SELECT * FROM Stu; 

    (g) Атрибут A типа данных varchar (20) имеет значение «Amit». Атрибут B типа данных char (20) имеет значение «Karanita». Сколько символов занято в атрибуте A? Сколько символов занято в атрибуте B?

    Вопрос 4.
    (a) Ниже приводится список языков программирования:
    BASIC, COBOL, C, JAVA
    Помогите Sandhya определить объектно-ориентированный язык (языки) из приведенного выше списка.
    (b) Сатьям разрабатывает фрейм в Netbeans, содержащий окно списка. Помогите ему написать подходящий оператор Java для извлечения выбранного элемента из заданного списка с именем «jList1».
    (c) Что будет отображаться в jTextFieldl и jTextField2 после выполнения следующего кода:

     int number = 12;
    если (число <15)
    {
    jTextField1.setText ("" + число);
    число ++;
    jTextField2.setText («« + число);
    }
    еще
    {
    jTextFieldl. setText («число 1»);
    число ++;
    jTextFleld2.setText («число 2»);
    } 

    (d) Найдите вывод следующего фрагмента кода Java:

     String City = "Delhi", PinCode = "110001", Str = "";
    Город = Город + "" + PIN-код;
    JOptionPane.showMessageDialog (нуль, город);
    int l = City.length ();
    int я = 0;
    в то время как (i 

    (e) Перепишите следующий код, используя цикл WHILE:

     int x = 100;
    для (int i = 2; i <= 22; i = i + 4)
    {
    jTextAreal.добавить ("\ п" + (я + х));
    х = х-2;
    } 

    (f) В следующем коде есть ошибки. Перепишите правильный код, подчеркнув все внесенные исправления:

     int n = 5, int i = l. f = l;
    делать ;
    {
    f = f * i;
    я + +;
    в то время как (я <= п)
    jTextFieldl.setText ("" + f); 

    (g) Г-н Паван работает программистом в «ABC Marketing Company», где он разработал программное обеспечение для генерации зарплаты, чтобы генерировать зарплату продавца, в которой имя и зарплата вводятся пользователем. Скриншот того же показан ниже:

    Помогите ему в написании кода, чтобы сделать следующее:

    (i) После выбора соответствующей радиокнопки, когда нажата кнопка «Комиссия», комиссия должна отображаться в соответствующем тексте. поле, поскольку каждый продавец получит комиссию, основанную на проданных единицах в соответствии со следующими критериями:

    (ii) При нажатии кнопки «Валовая зарплата» должна быть рассчитана и отображена в соответствующем текстовом поле валовая зарплата в соответствии с заданным формула:
    Заработная плата брутто = Заработная плата + Комиссия

    (iii) После необходимого выбора флажков, когда нажата кнопка «Плата за обслуживание», в соответствующем текстовом поле будут отображаться сборы за обслуживание в соответствии со следующими критериями:

    (iv) Деньги будут вычтены из общей заработной платы в соответствии с условиями, выбранными сотрудником.При нажатии кнопки «Чистая зарплата» должна быть рассчитана и отображена в соответствующем текстовом поле чистая зарплата в соответствии с приведенными формулами:
    Чистая зарплата = вычеты из заработной платы брутто

    Вопрос 5.
    (a) Г-жа Шарма - классный учитель. класса «XII A». Она хочет создать таблицу «Студент» для хранения сведений о своем классе.

    (i) Что из следующего может быть атрибутами таблицы Student?
    (a) RollNo
    (b) «Amit»
    (c) Имя
    (d) 25

    (ii) Назовите первичный ключ таблицы «Student».Укажите причину выбора.

    (b) Запишите вывод следующих SQL-запросов:

     (i) SELECT TRUNCATE (8.975.2);
    (ii) ВЫБРАТЬ MID («ЧЕСТНОСТЬ ПОБЕДЫ», 3,4);
    (iii) ВЫБЕРИТЕ RIGHTCCONCATC «ПРАКТИЧЕСКАЯ ИНФОРМАТИКА»), 5);
    (iv) ВЫБЕРИТЕ DAYOFMONTHA2015-01-16 '); 

    (c) Таблица «Emp» приведена ниже. Напишите команды в SQL для (i) - (iv) и вывод для (v) и (vi)

    (i) Для отображения списка всех сотрудников младше 25 лет.
    (ii) Перечислить имена и соответствующие оклады в порядке убывания оклада.
    (iii) Подсчитать количество сотрудников, имена которых начинаются с буквы «K».
    (iv) Перечислить имена и адреса тех лиц, у которых в адресе есть слово «Дели».

     (v) ВЫБЕРИТЕ Имя. Заработная плата ОТ Emp, ГДЕ Заработная плата МЕЖДУ 50000 И 70000;
    (vi) ВЫБЕРИТЕ Имя, Телефон ИЗ Emp, ГДЕ ТЕЛЕФОН КАК '99% '; 

    Вопрос 6.
    (a) Напишите SQL-запрос для создания таблицы Player со следующей структурой:

    (b) Рассмотрите таблицы, приведенные ниже.
    Продавец

    Заказы

    (i) Столбец «Продавец» в таблице «Продавец» является КЛЮЧОМ ___.Столбец «Продавец» в таблице «Заказы» является КЛЮЧОМ ___.
    (ii) Может ли «Продавец» быть установлен в качестве первичного ключа в таблице «Заказы». Обоснуйте.

    (c) Со ссылкой на приведенные выше таблицы (в Q6 b), Запишите команды в SQL для (i) и (ii) и вывод для (iii) ниже:
    (i) Для отображения Продавца, имен, заказов и сумма заказа всех продавцов.
    (ii) Отображать имена, идентификаторы продавцов и идентификаторы заказов тех продавцов, имена которых начинаются с буквы A, а сумма продаж составляет от 15000 до 20000.
    (iii) ВЫБЕРИТЕ продавца, имя, возраст, сумму от продавца, заказы, ГДЕ продавец. Продавец = Заказы. Продавец И Возраст ОТ 30 до 45 лет;

    Вопрос 7.
    (a) «Bachpan Toys» - небольшая компания, производящая игрушки. Они решили, что для компании было бы выгодно создать веб-сайт, на котором клиенты могли бы заказывать игрушки онлайн. Укажите, какую пользу принесет «Bachpan Toys» Веб-сайт.

    (b) Что из перечисленного является преимуществом (-ями) электронного управления?

    1. Технологии ускоряют управление.
    2. Компьютерная грамотность и базовое пользование Интернетом не требуется.
    3. Управление стало прозрачным, то есть большая часть информации доступна общественности.

    (c) Лакшми работает в школе. Она хочет создать в форме элементы управления для следующих функций. Выберите соответствующие элементы управления из текстового поля, метки, переключателя, флажка, поля списка, поля со списком, кнопки и напишите в третьем столбце.

    ОТВЕТЫ

    Ответ 1.
    (a) (i) LAN (ii) Satellite

    (b) Две угрозы безопасности компьютерных сетей:

    1. Компьютерный червь Это автономная вредоносная компьютерная программа который копирует себя для распространения на другие компьютеры.
    2. Спам Интернет наводняет множество копий одного и того же сообщения.

    (c) Преимущества использования Unicode для представления текста следующие:

    1. Позволяет использовать многоязычный текст на любом или всех языках по вашему желанию.
    2. Обмен текстами на любом языке возможен во всем мире.

    (d) Пример URL → http: // www. Майкрософт. com
    Пример IP-адреса → 192.168.0.0

    (e) Звездная топология
    Два преимущества звездообразной топологии:

    1. Простота обнаружения неисправностей и их устранения.
    2. Отказ одной системы не приведет к выходу из строя всей сети.

    Ответ 2.
    (a) Оператор If более подходит для этой ситуации, потому что здесь даны только два условия, которые будут выполняться оператором if, в то время как оператор switch используется, когда заданы операторы выбора нескольких ветвей.

     (б) класс Квадрат
    {
    public static void main (String args [])
    {
    int x. у;
    х = 10:
    у = х * х;
    
    (c) переключатель (выбор)
    {
    Случай 1 :
    JTextFieldl. setText («Нью-Дели - Гоа»);
    перемена ;
    Случай 2:
    jTextFieldl.setText («Нью-Дели - Париж»);
    перемена ;
    Случай 3:
    jTextFieldl.setText («Нью-Дели - Бангкок»);
    перемена ;
    дефолт:
    jTextFieldl.setText («P1. выберите допустимый вариант»);
    }
    
    (d) (i) Теги → ,  
      ,
    • Атрибут → тип (ii) Теги → , Атрибут → bgcolor

    (e) Эти теги являются частью кода XML.
    Это разные, потому что XML чувствителен к регистру.

    (f) 4 раза

    Ответ 3.
    (a) Правильный код:

     SELECT ID, Salary FROM empsalary
    ГДЕ Зарплата = 56000; 
     (b) ВЫБРАТЬ ОТЛИЧИТЕЛЬНОЕ (местоположение) ОТ сотрудника;
    (c) ALTER TABLE Student ADD Game_Played VARCHAR (30);
    (d) ОБНОВЛЕНИЕ УКАЗАННЫХ УЧАСТНИКОВ = 50
    ГДЕ Номер рулона = 2; 

     (g) Символы заняты в атрибуте A = 4
    Символы заняты в атрибуте B = 8 

    Ответ 4.
    (a) JAVA, COBOL
    (b) jListl. getSelectedValue ();

    (c) Выходные данные
    jTextFieldl будут содержать значение = 12
    jTextField2 будет содержать значение = 13

    (d) Выходные данные
    Delhi 110001
    10001

    (g) (i) Нам нужно дважды щелкнуть по кнопке Комиссии ( jButtonl) и введите следующий код:

     int Комиссия:
    если (jRadioButtonl. isSelected () = True)
    Комиссия = 500;
    иначе, если (jRadioButton2. isSelected () = True)
    Комиссия = 1000;
    еще
    Комиссия = 2000;
    jTextField3.setText (Комиссия); 

    (ii) Нам нужно дважды щелкнуть кнопку «Gross Salary» (jButton2) и ввести следующий код:

     int Gross_Sal;
    int Salary = Integer, parselnt (jTextField2. getText ());
    Gross_Sal = Заработная плата + Комиссия;
    jTextField4.setText (Валовая_продажа); 

    (iii) Нам нужно дважды щелкнуть кнопку «Плата за услуги» (jButton3) и ввести следующий код:

     int Fac_Charges;
    if (jCheckBox1. is Selected () == True)
    Fac_Charges = 500;
    иначе, если (jCheckBox2.isSelected () = Истина)
    Fac_Charges = 2000;
    еще
    Fac_Charges = 0;
    jTextField5. setText (Fac_Charges); 

    (iv) Нам нужно дважды щелкнуть кнопку «Чистая зарплата» (jButton4) и ввести следующий код:

     int Net_Sal;
    Чистая_продажа = Валовая_продажа-Фактические_платы;
    jTextField6. setText (Net_Sal); 

    Ответ 5.
    (a) (i) RolINo, Имя может быть атрибутами таблицы Student.
    (ii) Первичным ключом таблицы Student является RolINo, потому что RolINo - это поле в таблице Student, которое однозначно идентифицирует каждую строку / запись в таблице базы данных.

    (b) (i) 8.97
    (ii) NEST
    (iii) ATICS
    (iv) 16

     (c) (i) ВЫБРАТЬ * ИЗ Emp
    ГДЕ ВОЗРАСТ <25;
    (ii) ВЫБЕРИТЕ ИМЯ, ЗАРПЛАТУ ИЗ Emp
    ЗАКАЗАТЬ ПО ОПРЕДЕЛЕНИЮ ЗАРПЛАТЫ;
    (iii) ВЫБЕРИТЕ ИМЯ, СЧЕТЧИК (ИМЯ) ИЗ Emp
    ГДЕ ИМЯ как "K%";
    (iv) ВЫБЕРИТЕ ИМЯ, АДРЕС ИЗ Emp
    ГДЕ АДРЕС, например "% Дели%; 

    (v)

    (vi)

    Ответ 6.
    (a) CREATE TABLE Player

    (b) (i) Primary, Foreign
    (ii) Продавцы не могут быть указаны в качестве первичного ключа в таблице Заказы, потому что таблица Заказы уже имеет первичный ключ, и в одной таблице не может быть более одного столбца первичного ключа.

    (c) (i) ВЫБРАТЬ продавцов, имя, заказ, сумму от продавца; Заказы
    ГДЕ Продавец.Salespersonld = Заказы. Продавцы;
    (ii) Выберите Имя, Продавцы, Заказ от продавца, Заказы
    ГДЕ Продавец. Продавцы = Заказы. Продавцы И Имя, например «A%» И Сумма МЕЖДУ 15000 И 20000;

    Ответ 7.
    (a) Веб-сайт дает следующие преимущества:

    1. Покупатели могут покупать игрушки где угодно и когда угодно, потому что Интернет доступен 24 часа в сутки, 7 дней в неделю.

      Добавить комментарий

      ©2024 «Детская школа искусств» Мошенского муниципального района